You are on page 1of 208

Table of Contents

S.No. Chapter Name Pg. No.

1. Limit
• Theory 1-13
(Examples, Concept Builders, Solved Examples)
• Objective Exercise - I 14-16
• Objective Exercise - II 17-22
• Subjective Exercise - I 23-23
• Subjective Exercise – II 24-24
• JEE-Main (Previous Year Questions) 25-28
• JEE-Advanced (Previous Year Questions) 29-30

2. Continuity
• Theory 31-37
(Examples, Concept Builders, Solved Examples)
• Objective Exercise - I 38-40
• Objective Exercise - II 41-42
• Subjective Exercise - I 43-44
• Subjective Exercise – II 45-45
• JEE-Main (Previous Year Questions) 46-49
• JEE-Advanced (Previous Year Questions) 50-50

3. Differentiability
• Theory 51-61
(Examples, Concept Builders, Solved Examples)
• Objective Exercise - I 62-63
• Objective Exercise - II 64-67
• Subjective Exercise - I 68-69
• Subjective Exercise – II 70-71
• JEE-Main (Previous Year Questions) 72-74
• JEE-Advanced (Previous Year Questions) 75-78

4. Methods of Differentiation
• Theory 79-97
(Examples, Concept Builders, Solved Examples)
• Objective Exercise - I 98-101
• Objective Exercise - II 102-104
• Subjective Exercise - I 105-107
• Subjective Exercise – II 108-108
• JEE-Main (Previous Year Questions) 109-112
• JEE-Advanced (Previous Year Questions) 113-114
5. Rate Measure, Tangent and Normal
• Theory 115-124
(Examples, Concept Builders, Solved Examples)
• Objective Exercise - I 125-126
• Objective Exercise - II 127-129
• Subjective Exercise - I 130-130
• Subjective Exercise – II 131-132
• JEE-Main (Previous Year Questions) 133-135
• JEE-Advanced (Previous Year Questions) 136-136

6. Monotonicity
• Theory 137-146
(Examples, Concept Builders, Solved Examples)
• Objective Exercise - I 147-149
• Objective Exercise - II 150-152
• Subjective Exercise - I 153-153
• Subjective Exercise – II 154-155
• JEE-Main (Previous Year Questions) 156-158
• JEE-Advanced (Previous Year Questions) 159-162

7. Maxima - Minima
• Theory 163-175
(Examples, Concept Builders, Solved Examples)
• Objective Exercise - I 176-178
• Objective Exercise - II 179-181
• Subjective Exercise - I 182-183
• Subjective Exercise – II 184-185
• JEE-Main (Previous Year Questions) 186-189
• JEE-Advanced (Previous Year Questions) 190-191

Answer Key 192-205


1 Limit
Introduction
The concept of limit of a function is one of the fundamental ideas that distinguishes calculus from
algebra and trigonometry. We use limits to describe the way a function f varies. Some functions vary
continuously; small changes in x produce only small changes in f(x). Other functions can have values
that jump or vary erratically. We also use limits to define tangent lines to graphs of functions. This
geometric application leads at once to the important concept of derivative of a function.

Definition
Let f f(x) gets arbitrarily close
f(x) approaches the limit L as x
Lim f f
x →a

implies if we can make the value of f(x) arbitrarily close to L (as close to L as we like) by taking x to
be sufficiently close to a (on either side of a) but not equal to a.

Left Hand Limit and Right-Hand Limit of a Function


The value to which f → a ) is called left hand
limit of f(x) at x = a. Symbolically, LHL = Lim− f(x) = Lim f(a h).
x →a h→0

The value to which f → a+) is called right


hand limit of f(x) at x = a. Symbolically, RHL = Lim+ f(x) = Lim f(a + h).
x →a h→0

Limit of a function f(x) is said to exist as, x → a when Lim− f(x) = Lim+ f(x) = Finite quantity.
x →a x →a

Example: Graph of y = f(x)


y
1

1 1 2 x
0

Fig. 1
Lim f(x) = Lim f( 1 + h) = f( 1+) = 1
x →−1+ h→0

= − = =
x →0− h→0
+
= + = =
x →0+ h→0

Limit 1

= − = =−
x →1− h→0
+
= + = =
x →1+ h→0

= − = =
x →2− h→0

= 0 and does not exist.


x →0 x→1

Important Note
In , x → a necessarily implies x  a. That is while evaluating limit at x = a, we are not concerned
x →a

with the value of the function at x = a. In fact, the function may or may not be defined at x = a.
Also, it is necessary to note that if f(x) is defined only on one side of 'x = a', one sided limit is good
enough to establish the existence of limits, and if f(x) is defined on either side of 'a' both sided limits
are to be considered.
As in lim cos 1 x = 0, though f(x) is not defined for x > 1, even in its immediate vicinity.
x→1

Example 1:
Consider the adjacent graph of y = f(x) Find the following:
y

0 x
1 2 3 4 5 6
1
(a) (b) (c)
x →0− x →0+ x → 1−
(d) (e) (f)
x → 1+ x → 2− x → 2+
(g) lim f(x) (h) (i)
x → 3− x → 3+ x → 4−
(j) (k) =2 (l) = 
x → 4+ x→ x →6−

Solution:
(a) As x → 0 : limit does not exist (the function is not defined to the left of x = 0)
(b) As x → 0+: f(x) → 1  = 1. (c) As x → 1 : f(x) → 1  = 1.
x →0+ x → 1−

(d) As x → 1+: f(x) → 2  lim f(x) = 2. (e) As x → 2 : f(x) → 3  lim f(x) = 3.


x → 1+ x → 2−
(f) As x → 2 : f(x) → 3  lim f(x) = 3.
+
(g) As x → 3 : f(x) → 2  = 2.
x → 2− x → 3−

(h) As x → 3+: f(x) → 3  = 3. (i) As x → 4 : f(x) → 4  = 4.


x → 3+ x → 4−
(j) As x → 4+: f(x) → 4  = 4. (k) As x →: f(x) → 2  = 2.
x → 4+ x→

(l) As x → 6 , f(x) →  =  limit does not exist because it is not finite.


x →6−

2 Limit
Concept Builders - 1

(i) Which of the following statements about the function y = f(x) graphed here are true, and which

are false?

y = f(x)

1 0 1 2 3 x

(a) =1 (b) does not exist


x → − 1+ x →2

(c) =2 (d) =2
x →2 x → 1−

(e) does not exist (f) =


x →1 x →0+ x →0−

(g) exists at every c  ( 1, 1) (h) exists at every c  (1, 3)


x →c x →c

(i) =0 (j) does not exist.


x → 1− x → 3+

Fundamental Theorems on Limits

Let =  and lim g(x) = m. If  and m exist finitely then:


x →a x →a

(a) Sum rule: Lim {f(x) + g(x)} =  + m


x →a

(b) Difference rule: Lim {f(x) g(x)} =  m


x →a

(c) Product rule: Lim f(x). g(x) = .m


x →a

(d) Quotient rule: Lim = ,provided m  0


x →a g(x) m

(e) Constant multiple rule: Lim kf(x) = k Lim f(x) ; where k is constant.
x →a x →a

(f) Power rule: If m and n are integers then Lim [f(x)]m/n = m/n provided m/n is a real number.
x →a

Limit 3
Indeterminate Forms
0 
, ,  , 0 × , 1, 00, 0
0 
Initially we will deal with first five forms only and the other two forms will come up after we have gone
through differentiation.
Note: (i) Here 0,1 are not exact, infact both are approaching to their corresponding values.
(ii) We cannot plot  on the paper. Infinity () is a symbol and not a number It does not
obey the laws of elementary algebra,
(a)  +  →  (b)  ×  →  (c) →  (d) 0 → 0

General Methods to Be Used to Evaluate Limits


(A) Factorization:
Important factors:
(i) xn an = (x a) (xn 1 + axn 2
+ ........... + an 1), n  N
(ii) xn + an = (x + a))(xn 1
axn 2
+...............+an 1), n is an odd natural number.
xn − an
Note: Lim = nan 1
x →a x−a
 1 2(2 x − 3) 
Example 2: Evaluate: lim  − 
x →2  x − 2 3
x − 3x2 + 2x 
Solution: We have
 1 2(2 x − 3)   1 2(2 x− 3)   x(x− 1) − 2(2 x − 3) 
lim  −  = lim  −  = lim  
x →2  x − 2 3 2
x − 3x + 2x  x →2  x − 2 x(x− 1)(x− 2)  x →2  x(x − 1)(x − 2) 

 x2 − 5x + 6   (x− 2)(x− 3)   x−3  1


= lim   = lim   = lim   =
x→2  x(x− 1)(x− 2)  x →2  x(x − 1)(x − 2)  x →2  x(x − 1)  2
 

Concept Builders - 2

x−1
(i) Evaluate: lim
x → 1 2x2 − 7x + 5
2
x − 3x + 2
(ii) lim
x→1 x2 − 1
x5 − 25
(iii) lim
x →2 x7 − 27
x3 + 1
(iv) lim
x→−1 x + 1

xm − 1
(v) lim
x →1 x − 1

(B) Rationalization or Double Rationalization:

4 Limit
4 − 15x + 1
Example 3: Evaluate: lim
x→1 2 − 3x + 1

Solution: lim
4 − 15x + 1
= lim
(4 − )( )(
15x + 1 2 + 3x + 1 4 + 15x + 1 )
x→1 2 − 3x + 1 x→1
(2 − 3x + 1 )( 4 + 15x + 1 )(2 + 3x + 1 )

(15 − 15 x) 2 + 3x + 1 5
lim  =
x→1 (3 − 3 x) 4 + 15x + 1 2

 x2 + 8 − 10 − x2 
Example 4: Evaluate: lim  
x→1  2 2 
 x +3 − 5−x 
3−3 0
Solution: This is of the form = if we put x = 1
2−2 0
0
To eliminate the factor, multiply by the conjugate of numerator and the conjugate of
0
the denominator
 Limit = lim
x→1

 x2 + 8 + 10 − x2   x2 + 3 + 5 − x2 
   
 x2 + 8 − 10 − x2    ×  
 
   x2 + 8 + 10 − x2   x2 + 3 + 5 − x2   x2 + 3 − 5 − x2 
    
    

x2 + 3 + 5 − x2 (x2 + 8) − (10 − x2 )  x2 + 3 + 5 − x2 
lim × = lim   × 1 = 2+2 = 2
x→1
x2 + 8 + 10 − x2 (x2 + 3) − (5 − x2 ) x→1  2 2  3+3 3
 x + 8 + 10 − x 

Concept Builders - 3

p+x − p−x
(i) Evaluate: lim
x→0 q+ x − q−x

a + 2x − 3x
(ii) Evaluate: lim ,a  0
x →a 3a + x − 2 x
 G(x) − G(1) 
(iii) If G(x) = 25 − x2 , then find the lim  
x→1  x−1 

1− x
(iv) lim
x→1 1− x
1− 1+ x
(v) lim
x →0 2 − 2+x

(c) Limit When x → :


(i) Divide by greatest power of x in numerator and denominator.
(ii) Put x = 1/y and apply y → 0

Limit 5
Example 5:

x2 + x + 1
Evaluate: Lim
x → 3x2 + 2x − 5

Solution:

x2 + x + 1    1 1 + y + y2 1
Lim  form  Put x = Limit = Lim =
2
x → 3x + 2x − 5    y y →0 3 + 2y − 5y2 3

Example 6:

 x3 + 1 
If lim  − (ax+ b)  = 2, then
x→  x2 + 1 
 

(A) a = 1, b = 1 (B) a = 1, b = 2

(C) a = 1, b = 2 (D) none of these

Solution:

 x3 + 1  x3 (1 − a) − bx2 − ax + (1 − b)
lim  − (ax+ b)  = 2  lim =2
 2
x→ x + 1  x → x2 + 1
 

a (1 − b)
x(1 − a) − b− +
x x2
lim =21 a = 0, b = 2  a = 1, b = 2 Ans. (C)
x → 1
1+
x2

Concept Builders - 4

n+2+ n+ 1
(i) Evaluate: lim
n→ n+2− n+ 1

(ii) Evaluate: lim  n − n2 + n 


n→  

2x3 + 1
(iii) lim
x → 3x3 + x + 2
x
2
(iv) lim  
x →  3 

x 
(v) lim  
x → x

6 Limit
(d) Squeeze Play Theorem (Sandwich theorem):
Statement: If f(x)  g(x)  h (x) ;  x in the neighborhood at x = a and
y

y = x2

y = x2 sin

0 x

y= x2

Lim f(x) =  = Lim h(x) then Lim g(x) = ,


x →a x →a x →a

Example 1:
1
Lim x2sin = 0,
x →0 x
 1
 sin   lies between 1 and 1
x
1 1
 x2 x2 sin  x2  Lim x2 sin = 0 as Lim ( x2) = Lim x2 = 0
x x →0 x x →0 x →0

Example 2:

1
lim x sin =0
x →0 x
 1
 sin   lies between 1 and 1
x
1
 x  x sin  x
x
1
 Lim x sin = 0 as Lim ( x) = Lim x = 0
x →0 x x →0 x →0

Example 7:
[x] + [2 x] + [3 x] + .....[nx]
Evaluate: lim (where [.] denotes the greatest integer function.)
n→ n2
Solution:
We know that x 1 < [x]  x
n
 x + 2x + ......nx n<  [rx]  x+ 2 x+ ....... + nx
r=1

Limit 7
n n
xn x.n(n+ 1) x  1 1 1 x  1

2
(n+ 1) n<  [rx] 
r=1
2
 1+ 
2  n
<
n n2  [rx]  2  1 + n 
r=1

x  1 x x 1 1 x
Now, lim 1+  = and lim  1 +  − =
n→ 2  n  2 n → 2  n  n 2
[x] + [2 x] + ...... + [nx] x
Thus, lim =
n→ n 2 2

Limit of Trigonometric Functions


sin x tan x tan−1 x sin−1 x
Lim = 1 = Lim = Lim = Lim [where x is measured in radians]
x →0 x x →0 x x →0 x x→0 x
sin( nx)
(a) If = 0, then Lim = 1, example Lim =1
x →a x →a x → 1 ( nx)

Example 8:
x3 cot x
Evaluate: l im
x →0 1 − cos x

Solution:
x3 cos x x3 cos x(1 + cosx) x3
lim = lim = lim . cosx(1 + cos x) = 2
x →0 sin x(1 − cosx) x →0 sin x. sin2 x x →0 sin3 x

Example 9:
(2 + x) sin(2 + x) − 2 sin2
Evaluate: lim
x →0 x
Solution:
  x x 
 2.2cos  2 +  sin 
2(sin(2 + x) − sin2) + x sin(2 + x)  2 2
lim = lim  + sin(2 + x) 
x →0 x x →0  x 
 
 
 x x
2cos  2 +  sin
 2  2
= lim + lim sin(2 + x) = 2cos 2 + sin 2
x →0 x x →0

Example 10:
a
sin
Evaluate: lim n
n→ b
tan
n+ 1
Solution:
1 a
As n →, → 0 and also tends to zero
n n
a
sin
a n sin 
sin should be written as so that it looks like lim
n a →0 

8 Limit
 a  b 
 sin   
The given limit = lim  n   n + 1  . a(n+ 1)
n→  a   b  n.b
   tan 
 n  n+ 1
 a  b 
 sin   
= lim  n  n + 1 . a 1 + 1  = 1 × 1 × a × 1 = a
n→  a  
 
b  b n b b
  tan 
 n  n+ 1

Concept Builders - 5

sin x
(i) lim
x →0 tan  x

sin2 x − sin2 y
(ii) lim
x→y x2 − y2

(a+ h)2 sin(a+ h) − a2 sina


(iii) lim
h→0 h
sin5x
(iv) lim
x→0 sin7x

sin x
(v) lim
x → x

Limit of Exponential Functions

ax − 1 ex − 1
(a) Lim = n a (a > 0) In particular Lim = 1.
x→0 x x→0 x

a −1
In general if Lim f(x) = 0, then Lim = n a, a > 0
x →a x →a

Example 11:

etan x − ex
Evaluate: lim
x →0 tanx − x

Solution:

etan x − ex ex  e(tanx −x) − ex


lim = lim
x →0 tanx − x x→0 tanx − x

ex (etan x −x − 1) ex (ey − 1) ey − 1
= lim = lim where y = tan x x and lim =1
x→0 tanx − x x →0y →0 y y →0 y

= e0 × 1 =1×1=1 [as x → 0, tan x x → 0]

Limit 9
Concept Builders - 6

ex − ea
(i) Evaluate: lim
x →a x − a

2x − 1
(ii) Evaluate: lim
x →0 (1 + x)1/2 − 1
e2x − 1
(iii) lim
x →0 sin3x

2sin x − 1
(iv) lim
x→0 x
1
3x − 1
(v) lim
x →  1
sin  
x
x
1/x  1
(b) (i) Lim ( 1 + x ) = e = Lim  1 +  (Note: The base and exponent depend on the
x →0 x →  x
same variable.) In general, if Lim f(x) = 0, then Lim (1 + f(x))1/f(x) = e
x →a x →a
n(1 + x)
(ii) Lim =1
x →0 x
(x)
(iii) If = 1 and Lim (x) = , then ;   = ek
x →a x →a x →a
where k = Lim  (x) [f(x) 1]
x →a

Example 12:
log x 3
Evaluate Lim(log 3 3 x)
x→1
Solution:
log x 3 log x 3 1/log3 x 1
Lim(log 3 3 x) = Lim(log 3 3 + log 3x) = lim(1 + log3x) =e  logb a =
x→1 x→1 x→1 log a b

Example 13:
x n(1 + 2 tanx)
Evaluate: Lim
x →0 1 − cos x
Solution:
x n(1 + 2 tanx) x n(1 + 2 tan x) 2 tan x
Lim = Lim . =4
x →0 1 − cos x x →0 1 − cos x 2 2 tan x
.x
x2

Example 14:
4x2 +2
 2x2 − 1 
Evaluate: lim  2 
x →  2x + 3 
 
Solution:
Since it is in the form of 1
4x2 +2
 2x2 − 1  lim  2x2 − 1 − 2x2 − 3 
lim 


x →  2x2 + 3 

= ex → 


2
2x + 3 

(
 4x2 + 2 = e ) 8

10 Limit
Concept Builders - 7

(i) Evaluate: lim x{n(x + a) nx}


x →
pn+q
 1
(ii) Evaluate: lim  1 + 
n→  n
1
(iii)
x→0
(
Evaluate: lim 1 + tan2 x ) 2x

x+4
x + 6
(iv) Evaluate: lim  
x→  x + 1 

1
(v)
x→
(
lim 2x + 3x + 5x ) x

Limit Using Series Expansion

Expansion of function like binomial expansion, exponential and logarithmic expansion, expansion of

sinx, cosx, tanx should be remembered by heart which are given below:

x na x2 n2a x3 n3a
(a) ax = 1 + + + +........ a > 0
1! 2! 3!

x x2 x3
(b) ex = 1 + + + + ......
1! 2! 3!

x2 x3 x4
(c) m (1 + x) = x + − + ..... for 1<x1
2 3 4

x3 x5 x7
(d) sin x = x + − +......
3! 5! 7!

x2 x4 x6
(e) cos x = 1 + − +.......
2! 4! 6!

x3 2x5
(f) tan x = x + + + .....
3 15

x3 x5 x7
(g) tan 1 x = x + − + ....
3 5 7

12 3 12.32 5 12.32.52 7
(h) sin 1x = x + x + x + x + ....
3! 5! 7!

x2 5x4 6x6
(i) sec 1x = 1 + + + + .....
2! 4! 6!

n(n− 1) 2
(j) (1 + x)n = 1 + nx + x + ................ n  Q
2!

Limit 11
Example 16:
ex − e−x − 2x
lim
x→0 x − sinx
Solution:
x2 x3 x3 x3
x −x 1+ x + + + ....... − 1 − x + − + ...... − 2x
e − e − 2x 2! 3! 2! 3!
lim  lim
x→0 x − sinx x →0  x3 x5 
x − x − + ....... 
 3! 5! 
 

x3 x5 1 1 2 
2. + 2. + ..... x3  + x + .... 
6 5!  3 60  = 1/ 3 = 2
 lim  lim
x →0 x 3
x 5 x →0 3  1 1 2  1/6
+ ...... x  + x + ..... 
6 5!  6 120 

Concept Builders - 8

x − sinx
(i) Evaluate: Lim
x→0 sin(x3 )

x − tan−1 x
(ii) Evaluate: Lim
x →0 x3
x − sin x
(iii) lim
x →0 x3
x − tanx
(iv) lim
x→0 x3
x2
1− − cos x
(v) lim 2
x →0 x4

Miscellaneous Examples
Example 17:

Evaluate lim sin .
x →0 x
Solution:
Again, the function f(x) = sin (/x) is undefined at 0. Evaluating the function for some small
 1
values of x, we get f(1) = sin  = 0, f   = sin 2 = 0,
2
f(0.1) = sin 10 = 0, f(0.01) = sin 100 = 0.

On the basis of this information we might be tempted to guess that lim sin = 0 but this time
x →0 x
our guess is wrong. Note that although f(1/n) = sin n = 0 for any integer n, it is also true that

12 Limit
 
f(x) = 1 for infinitely many values of x that approach 0. [In fact, sin (/x) = 1 when = + 2n
x 2
and solving for x, we get x = 2/(4n + 1) ]. The graph of f is given in following figure:

The dashed line indicates that the values of sin (/x) oscillate between 1 and 1 infinitely
often as x approaches 0. Since the values of f(x) do not approach a fixed number as x
approaches 0,

 lim sin does not exist.
x →0 x

ANSWER KEY FOR CONCEPT BUILDER

1. (i) (a) T (b) F (c) F (d) T (e) T (f) T

(g) T (h) T (i) F (j) T

1 1 5
2. (i) (ii) − (iii) (iv) 3 (v) m
3 2 28
q 2 1 1 1
3. (i) (ii) (iii) (iv) (v) 1+
p 3 3 24 2 2
1 2
4. (i) 1 (ii) (iii) (iv) 0 (v) 1
2 3
 sin2y 5
5. (i) (ii) (iii) 2asin a + a2 cos a (iv) (v) 0
 2y 7
2
6. (i) ea (ii) 2ln2 (iii) (iv) n2 (v) n3
3
1
7. (i) a (ii) ep (iii) e2 (iv) e5 (v) 5

1 1 1 1 1
8. (i) (ii) (iii) (iv) (v)
6 3 6 3 24

Limit 13
Objective Exercise - I

 1 3 
1. lim  −  is equal to:
x→1  1 − x 1 − x3 
(A) 1 (B) 0 (C)1 (D) D.N.E.

1+ x − 1− x
2. lim is equal to:
x →0 2x
1 1
(A) 0 (B) 1 (C) (D)
2 4

1+ 2+ x − 3
3. lim is equal to:
x→2 x−2
1 1 1
(A) (B) 3 (C) (D)
3 4 3 8 3

n
x −1
4. lim (m and n integers) is equal to:
x→1 m x −1
m n
(A) 0 (B) 1 (C) (D)
n m

2x − x2 + 3a2
5. If lim = 2 (where a  R+), then a is equal to
x →a x + a − 2a
1 1 1 1
(A) (B) (C) (D)
3 2 2 3 2 9

(n+ 1)4 − (n− 1)4


6. lim is equal to:
n→ (n+ 1)4 + (n− 1)4
(A) 1 (B) 0 (C) 1 (D) D.N.E.

(x+ 1)10 + (x+ 2)10 + ..... + (x+ 100)10


7. lim is equal to:
x→ x 10 + 1010
(A) 1 (B) 100 (C) 200 (D) 10

8. lim  x2 − 2x − 1 − x2 − 7x + 3  is equal to:


x→−  
5 5
(A) (B) (C) 0 (D) D.N.E.
2 2

14 Limit
cos2 − cos2x
9. lim is equal to:
x→−1 x2 − | x |
(A) 0 (B) cos2 (C) 2sin2 (D) sin1

 sin x   2 sin2x   10 sin 10x 


10. Let f(x) =   +   + ......+   (where [y] is the largest integer  y). The value
 x   x   x 
of lim f(x) equals:
x →0

(A) 55 (B) 164 (C) 165 (D) 375

x
 x2 − 2x + 1 
11. lim   is equal to:
x →   x2 − 4x + 2 
 
1
(A) 1 (B) e (C) (D) e2
e2

12. lim (1 + sin x)cos x is equal to:


x →0

1
(A) 0 (B) e (C) 1 (D)
e

13. lim (cos x + asinbx)1/x is equal to:


x →0

(A) ea (B) eab (C) eb (D) ea/b

1/x
  
14. lim  tan  + x   is equal to:
x →0
 4 
1
(A) e 2
(B) (C) e (D) e2
e

15. lim(4n + 5n )1/n is equal to:


n→

(A) 5 (B) 4 (C) 0 (D) D.N.E.

nx
 11/x + 21/x + 31/x + ..... + n1/x 
16. lim   is equal to:
x→ n 
 
1
(A) n! (B) 1 (C) (D) 0
n!

17. If lim (1 + ax + bx2)2/x = e3 , then:


x →0

3 3
(A) a = and b  R (B) a = and b  R+
2 2
(C) a = 0 and b = 1 (D) a = 1 and b = 0

Limit 15
x(1 + acosx) − bsinx
18. lim = 1 then:
x →0 x3
(A) a = 5/2 (B) a = 3/2, b = 1/2
(C) a = 3/2, b = 5/2 (D) a = 5/2, b = 3/2

1/x
 1
19. lim  2x −1 +  equals:
x →0  2
1
(A) 2 (B) n2 (C) n2 (D) 2
2

20. The value of l im


( x ) n(1 + 3 x)
sin 3

is equal to:
( tan x )  e ( ) − 1
x →0 2 5 3
x
−1

1 3 2 4
(A) (B) (C) (D)
5 5 5 5

16 Limit
Objective Exercise - II

Single Correct Type Questions

3
7 + x 3 − 3 + x2
1. lim is equal to:
x→1 x−1
1 1 1 1
(A) (B) (C) (D)
4 6 4 6

1
2. If lim  2n2 + n −  2n2 − n  = (where  is a real number), then -
n→   2
(A)  = 1 (B)  = 1 (C)  = ±1 (D)   ( , 1)

1/x
 f(x) + x2 
3. If f(x) is a polynomial of least degree, such that lim  1 +  = e2 , then f(2) is -
x →0  x2 
 
(A) 2 (B) 8 (C) 10 (D) 12

en
4. lim equals -
n→ n2
 1
1+ 
 n
1
(A) 1 (B) (C) e (D) e
2

secx  − tanx  − 1
5. The value of lim is equal to-
x →2 x−2
(A) sec . n sec + tan2.n tan
2
(B) sec2.n tan  + tan2. n sec
(C) sec2. n tan tan2.n sec (D) sec2.n sec tan2.n tan

 1 − x, 0  x  1

6. Consider the function f(x) = x + 2, 1  x  2 . Let lim f(f(x)) =  and lim f(f(x)) = m then which
x→1 x →2
4 − x, 2  x  4

one of the following hold good?
(A)  exists but m does not. (B) m exists but  does not.
(C) Both  and m exist (D) Neither  nor m exist.

 
cos  cos2 x 
7. Let f(x) be a quadratic function such that f(0) = f(1) = 0 and f(2) = 1, then lim 2  is
x →0 2

equal to-

(A) (B)  (C) 2 (D) 4
2

Limit 17
One or More Than One Correct Type Questions

3x2 + a2 − x2 + 3a2
8. If  = lim then -
x →a (x− a)
(A)  = 1  a  R (B)  = 1  a > 0

(C)  = 1  a  0 (D)  = D.N.E if a = 0

9. Which of the following limits vanish?


sinx arc tan x
(A) lim (B) lim
x→ x x→ x
x + sinx arcsin x
(C) lim (D) lim
x→ x + cos x x→1 x
tan
2

10. Which of the following statement are true for the function f defined for 1  x  3 in the figure
shown.

(A) lim f(x) = 1 (B) lim f(x) does not exist


x → − 1+ x →2

(C) lim f(x) = 2 (D) lim f(x) = lim f(x)


x → 1− x → 0+ x → 0−

11. Let f(x) = x + x2 + 2x and g(x) = x2 + 2x x, then:


(A) lim g(x) = 1 (B) lim f(x) = 1
x→ x→

(C) lim f(x) = 1 (D) lim g(x) = 1


x→− x→−

sin−1 (sinx) | x |


12. If A = lim and B = lim , then (where [.] denotes greatest integer function):
x → 0 cos−1 (cosx) x →0 x

(A) A = 1 (B) A does not exist


(C) B = 0 (D) B = 1

ex − 1 − x
13. The value (s) of 'n' for which lim exists is / are -
x→1 (x− 1)n
(A) 1 (B) 2 (C) 3 (D) 4

18 Limit
  (tan x)2n + x2 
    lim  ; x  0
14. Let f :  − ,  → R, f(x) = n →   sin2 x + (tan x)2n  , n  N. Which of the following holds
 2 2  

1 ; x=0
good?
 −   +   −   + 
(A) −  =   (B) −  = − 
 4  4  4   4 
       
 −   + 
(C)  =   (D) f(0+) = f(0) = f(0 )
 4  4
   

 tan2 x 
 for x  0
 x2 − x 2 
   
15. Let f(x) =  1 for x = 0 where [x] is the step-up function and {x} is the fractional
 x cot x for x  0
 
 
 
part function of x, then -
2
 
(A) lim f(x) = 1 (B) lim f(x) = 1 (C) cot 1  lim f(x)  = 1 (D) None

x → 0+ x → 0− x→0 

16. Which of the following limits does not exist?


 x 
(A) lim cosec 1   (B) lim sec 1 (sin 1x)
x→  x + 7 x→1

1 cot x
  
(C) lim+ xx (D) lim  tan  + x  
x →0 x →0
 8 

17. Consider following statements and identify correct options:


 2x 8  2x 8
(i) lim  −  = lim lim
x→4  x − 4 x − 4  x → 4 x −4 x→4 x−4

x2 + 6x − 7 (
lim x2 + 6x − 7
x→1
)
(ii) lim =
x→1 x2 + 5x − 6 lim ( x 2
+ 5x − 6 )
x→1

lim ( x − 3)
x−3 x→1
(iii) lim =
x→1 x2 + 2x − 4 (
lim x2 + 2x − 4
x→1
)
(iv) If lim f(x) = 2 and lim g(x) = 0, then lim does not exist.
x →5 x →5 x →5 g (x)

(v) If lim f(x) = 0 and lim g(x) = 2, then lim does not exist.
x →5 x →5 x →5 g (x)
(A) Only one is true. (B) Only two are true.
(C) Only three are false. (D) Only two are false.

Limit 19
x2 + 1 ; x  0,2
sinx; wherex = int eger 
   
18. Let f(x) =   : g(x) =  4 ; x = 0  , then:
0
 ; otherwise    5 ; x=2 
 
(A) lim g(f(x)) = 4
x →0

(B) lim f(g(x)) = 0


x →0

(C) lim f(g(x)) = 0


x→1

(D) lim g(f(x)) = 5


x→1

 2
x , if x is rational
19. If f(x) =  , then:
0 , if x is irrational

(A) lim f(x) = 0
x→ 0

(B) lim f(x) does not exist


x →0

(C) lim f(x) = 4


x →2

(D) lim f(x) does not exist


x →2

sin2  − sin2  


20. Let f() = lim , then f   is greater than -
 →  −2 2
4
3
1 − cos x
(A) lim
x→ 0 x sin2x
cot x − cos x
(B) lim
x → /2 ( − 2 x)3

(
(C) lim cos x + 1 − cos x
x→
)
a + 2x − 3x
(D) lim where a > 0
x →a 3a + x − 2 x

sin x + aex + be− x + cln(1 + x)


21. If has a finite limit L as x → 0, then:
x3
1 1
(A) a = (B) b =
2 2
1
(C) c = 0 (D) L =
3

ax − a −x
22. Let  = lim (a > 0), then:
x→ ax + a −x
(A)  = 1  a > 0 (B)  = 1  a  (0, 1)
(C)  = 0, if a = 1 (D)  = 1  a  

20 Limit
Match the Column Type Questions

23. For the function g(t) whose graph is given, match the entries of List-I to List-II

List-I List-II

(P) lim g(t) + lim g(t) (1) lim g(t)


t → 0+ t →2− t → 2+

(Q) lim g(t) + g(2) (2) does not exist


t → 0−

(R) lim g(t) (3) 0


t →0

(S) lim g(t) (4) lim g (t)


t →2 t→4

(A) (P)→(1,3); (Q)→(1,3); (R)→(2); (S)→(2)

(B) (P)→(2); (Q)→(4); (R)→(3); (S)→(1)

(C) (P)→(3); (Q)→(1); (R)→(2); (S)→(4)

(D) (P)→(4); (Q)→(2); (R)→(3); (S)→(1)

24. List I List-II

     
(P) lim n sin   cos   is equal to (1) 0
n→  4n   4n 

sin x 1
(Q) lim is equal to (2)
x →0 x 2

 1 1  
(R) lim  −  is equal to (3)
x → 0  sin x tan x  4

1 + cos2x 
(S) lim is equal to (4)
x → /2 2
( − 2 x) 180

(A) (P)→(4); (Q)→(1); (R)→(3); (S)→(2)

(B) (P)→(2); (Q)→(3); (R)→(1); (S)→(4)

(C) (P)→(3); (Q)→(4); (R)→(1); (S)→(2)

(D) (P)→(1); (Q)→(2); (R)→(3); (S)→(4)

Limit 21
25. List I List-II

ax
(P) lim (a > 0) can be equal to (1) lim x (e1/x 1)
x→ ax + 1 x→

sin(ex −2 − 1) ax + bx + cx − 3
(Q) lim is equal to (2) lim (a, b, c > 0 and abc = 1)
x →2 log(x − 1) x →0 x

(R) lim
( nx − 1) e
is equal to (3) lim
e4x − e3x
x →e x−e x →0 x

x(5x − 1) 1
(S) lim is equal to (4)
x→ 0 (1 − cosx)4 n5 2
(5) 0
(A) (P)→(5); (Q)→(2,3); (R)→(1,2,4); (S)→(3)
(B) (P)→(1,4); (Q)→(4,5); (R)→(2,3); (S)→(1)
(C) (P)→(4,5); (Q)→(2,4); (R)→(1,2); (S)→(4)
(D) (P)→(1,2,3,4,5); (Q)→(1,3); (R)→(1,3); (S)→(4)

22 Limit
Subjective Exercise - I

x2 − x. n x+ n x− 1
1. lim
x→ 1 x−1

 100 k 

 x  − 100
 K = 1 
2. l im 
x→1 x−1

1 − tan x
3. l im
x→
 1 − 2 sin x
4

2 − cos  − sin 
4. l im
→
 (4 − )2
4

5.
(3x 4
) 1
+ 2x2 sin + | x |3 + 5
x
l im
x→− | x |3 + | x |2 + | x | +1

 x2 + 1 
6. Find a and b if: (i) l im  − ax − b = 0
x→  x + 1 

(ii) l im  x2 − x + 1 − ax − b = 0
x→−  

7. l im [n (1 + sin2x). cot (n2 (1 + x))].


x →0

27x − 9x − 3x + 1
8. l im .
x →0 2 − 1 + cos x

9. Let f(x) = ax3 + bx2 + cx + d and g(x) = x2 + x 2.


c2 + d2
If l im = 1 and l im = 4, then find the value of .
x→1 g(x) x→−2 g(x) a2 + b2

8x2 + 3
 2x2 + 3 
10. l im  2 
x →   2x + 5 
 

x
x +c
11. l im   = 4, then find c:
x→  x − c 

Limit 23
Subjective Exercise - II

Integer Type Questions

8  x2 x2 x2 x2 
1. If L = l im  1 − cos − cos + cos cos  , then value of 32L is
x →0 x8  2 4 2 4 

        
sin  + 4h  − 4 sin  + 3h  + 6 sin  + 2h  − 4 sin  + h  + sin
 3   3   3   3  3
2. If L = l im , then value of 4L2 is:
4
h→0 h

 x+2 x+3
3. If L = l im x2  −3  , then value of 2L is:
x→  x x 

n
  
4. If  = l im
n→
  (r+ 1) sin r + 1 − r sin r  then find 
r =2
{} is

(where { } denotes the fractional part function)

5. Let {an}, {bn}, {cn}be sequences such that:


(i) an + bn + cn = 2n + 1 ; (ii) an bn + bncn + cnan = 2n 1;
(iii) anbncn = 1; (iv) an < bn< cn

Then find the value of 2 L im (nan) is


n→

6. If n  N and an = 22 + 42 + 62 + ....... +(2n)2 and bn = 12 + 32 + 52 + ...... + (2n 1)2.


an − bn
Find the value l im is:
n→ 3
n
2

x(1 + acos x) − bsinx


7. f(x) is the function such that l im = 1. l im = 1, then find the value |a +
x →0 x x →0 3

b| is:

1
2x2n sin +x
8. Let f(x) = l im x , then find:
n→ 1 + x2n
(a) l im xf(x)
x→

(b) l im f(x)
x →0

(c) l im f(x)
x→−

24 Limit
JEE-Main (Previous Year Questions)

1. For each t  R, let [t] be the greatest integer less than or equal to t. Then
  1  2  15  
lim x    +   + .....    [JEE(Main)-2018]
x → 0+ x
    x  x 
(1) does not exist (in R) (2) is equal to 0
(3) is equal to 15 (4) is equal to 120

2. For each x  R, let [x] be the greatest integer less than or equal to x. Then

lim−
( )
x x  + x sin x 
[JEE(Main)-2019]
x→0 x

(1) sin1 (2) sin 1


(3) 1 (4) 0

3. For each t  R, let [t] be the greatest integer less than or equal to t. Then,
 
(1− | x | + sin | 1 − x |) sin  [1 − x] 
 2 
lim [JEE(Main)-2019]
x → 1+ | 1 − x | [1 − x]
(1) equals 0 (2) does not exist
(3) equals 1 (4) equals 1

x cot(4 x)
4. lim is equal to: [JEE(Main)-2019]
x→0 sin2 x cot2 (2 x)
(1) 1 (2) 4 (3) 0 (4) 2

cot3 x − tanx
5. lim is: [JEE(Main)-2019]
x →  cos x + 
4 ( 4 )
(1) 4 2 (2) 8 (3) 4 (4) 8 2

 − 2sin−1 x
6. lim is equal to: [JEE(Main)-2019]
x→1− 1− x
 2 1
(1)  (2) (3) (4)
2  2

sin2 x
7. lim equals [JEE(Main)-2019]
x→0 +
(1) 2 2 (2) 4 (3) 4 2 (4) 2

Limit 25
4 3 3
8. If lim = lim , then k is: [JEE(Main)-2019]
x→1 x →k 2 2

3 4 3 8
(1) (2) (3) (4)
2 3 8 3

x2 − ax + b
9. If lim = 5 , then a + b is equal to: [JEE(Main)-2019]
x →1 x−1
(1) 7 (2) 5 (3) 4 (4) 1

x + 2 sinx
10. lim is: [JEE(Main)-2019]
x →0 2
x + 2 sinx + 1 − sin2 x − x + 1
(1) 3 (2) 2 (3) 1 (4) 6

3x + 33−x − 12
11. lim is equal to ______. [JEE(Main)-2020]
x →2 3−x/2 − 31−x

1/x2
 3x2 + 2 
12. lim   is equal to [JEE(Main)-2020]
x →0  7x2 + 2 
 
1 1
(1) e2 (2) e (3) (4)
e e2

x + x2 + x3 + ... + xn − n
13. If lim = 820, (n  N) then the value of n is equal to ______.
x →1 x−1
[JEE(Main)-2020]
1
   x
14. lim  tan  + x   is equal to: [JEE(Main)-2020]
x →0
 4 
(1) e (2) 2 (3) e2 (4) 1

1 1
(a+ 2 x)3 − (3 x) 3
15. lim (a  0) is equal to: [JEE(Main)-2020]
x→a 1 1
(3a+ x)3 − (4 x)3
4 4 1 1
 2 3  2 3  2   2 3  2   2 3
(1)   (2)   (3)     (4)    
9 3 9 3  39

1 − cos(p(x))
16. If  is the positive root of the equation, p(x) = x2 x 2 = 0, then lim is equal to
x → +
x+−4
[JEE(Main)-2020]
3 1 1 3
(1) (2) (3) (4)
2 2 2 2

26 Limit
1+ x2 + x4 − 1)/x
x(e( − 1)
17. lim [JEE(Main)-2020]
x →0
1 + x2 + x 4 − 1
(1) does not exist (2) is equal to e
(3) is equal to 1 (4) is equal to 0

    
 3 sin  + h  − cos  + h  
18. The value of 2lim 
 6  6   is: [JEE(Main) 2021]

h →0
 3h( 3 cosh− sinh) 
 

3 2 4 2
(1) (2) (3) (4)
4 3 3 3

aex − bcos x + ce− x


19. If lim = 2 , then a + b + c is equal to ______. [JEE(Main) 2021]
x →0 x sinx

cos−1 (x − [x]2 ). sin−1 (x − [x]2 )


20. The value of lim+ , where [x] denotes the greatest integer  x is:
x →0 x − x3
[JEE(Main) 2021]
 
(1)  (2) 0 (3) (4)
4 2

  1 1 

21. lim  tan2 x  (2 sin2 x + 3 sin x + 4) 2 − (sin2 x + 6 sin x + 2) 2   is equal to: [JEE(Main) 2021]
 
x→ 
2  
(1) 1/12 (2) 1/18 (3) 1/2 (4) 1/6

sin(3x2 − 4x + 1) − x2 + 1
22. If lim = −2 , then the value of (a b) is equal to. [JEE(Main) 2022]
x →1 2x3 − 7x2 + ax + b

23. The value of lim


(x 2
)
− 1 sin2 ( x)
is equal to : [JEE(Main) 2022]
x→1 x − 2x3 + 2x − 1
4

2 2 2
(1) (2) (3) (4) 2
6 3 2

24. If lim
n→
( )
n2 − n − 1 + n +  = 0 then 8( + ) is equal to: [JEE(Main) 2022]

(1) 4 (2) 8 (3) 4 (4) 8

 2 − (cos x + sin x)7


25. lim is equal to: [JEE(Main) 2022]
x→

4
2 − 2 sin2x

(1) 14 (2) 7 (3) 14 2 (4) 7 2

Limit 27
 1 1 1 1
26. lim  + + + ... +  is equal to [JEE(Main) 2023]
n→ 1 + n 2+n 3+n 2n 

3 2
(1) loge 2 (2) log e   (3) log e   (4) 0
2 3

sin2 t
 12 1 1

27. lim  1 sin t
+2 sin2 t
+ .... + nsin2 t
 is equal to: [JEE(Main) 2023]
t →0  
 
n ( n + 1)
(1) n2 (2) n2 + n (3) (4) n
2

1 + 2 − 3 + 4 + 5 − 6 + ...... + (3n − 2) + (3n − 1) − 3n


28. The value of lim is: [JEE(Main) 2023]
n→
2n4 + 4n + 3 − n4 + 5n + 4

2+1 3 3
(1) (2) (3) ( 2 + 1) (4) 3( 2 + 1)
2 2 2 2

( ) +( )
6 6
3x + 1 + 3x − 1 3x + 1 − 3x − 1
29. lim x6 [JEE(Main) 2023]
( ) ( )
x → 6 6
2 2
x+ x −1 + x− x −1

27
(1) is equal to (2) is equal to 9 (3) is equal to 27 (4) does not exist
2



1 1
 1 1
  1 1
 
30. Limn→  22 − 23   22 − 25  .  22 − 22n+ 1   is equal to [JEE(Main) 2023]
  

    
1
(1) (2) 1 (3) 2 (4) 0
2

28 Limit
JEE-Advanced (Previous Year Questions)

1− x
 −ax + sin(x− 1) + a  1− x 1
1. The largest value of the non-negative integer a for which lim   = is:
x → 1  x + sin(x − 1) − 1  4
[JEE(Advanced)-2014]
x2 sin( x)
2. Let ,    be such that lim = 1. Then 6( + ) equals: [JEE(Advanced)-2016]
x →0 x − sin x

1 − x(1+ | 1 − x |)  1 
3. Let f(x) = cos   for x  1, Then: [JEE(Advanced)-2017]
| 1−x | 1− x
(A) lim f(x) does not exist
x → 1+

(B) lim f(x) does not exist


x → 1−

(C) lim f(x) = 0


x → 1+

(D) lim f(x) = 0


x → 1−

4. Let f: → be a function. We say that f has [JEE(Advanced)-2019]



PROPERTY 1 if lim exists and is finite, and
h→0 |h|

PROPERTY 2 if lim exists and is finite.
h→0 h2
Then which of the following options is/are correct?
(A) f(x) = x2/3 has PROPERTY 1
(B) f(x) = sin x has PROPERTY 2
(C) f(x) = x| x | has PROPERTY 2
(D) f(x) = | x | has PROPERTY 1

5. Let e denote the base of the natural logarithm. The value of the real number a for which the
1
(1 − x) x − e−1
right-hand limit lim+ is equal to nonzero real number, is ______.
x →0 xa
[JEE(Advanced)-2020]

6. The value of the limit

4 2(sin3 x+ sinx)
lim is______ [JEE(Advanced)-2020]
x→
  3x 5x   3x 
2  2 sin2x sin + cos −
  2 + 2 cos2x + cos 
 2 2   2 

Limit 29
7. For any positive integer n , let Sn : (0, ) → be defined by [JEE(Advanced)-2021]
n  1+k(k+1)x2 
Sn (x) =  cot
k=1
−1


 x



−1  π π
where for any x  ,cot−1 (x)  (0,π) and   . Then which of the following
 2 2
statements is (are) TRUE ?
  1 + 11x2 
(A) S10 (x) = − tan−1   , for all x  0
2  10x 
 
(B) lim cot ( Sn (x) ) = x , for all x  0
n→


(C) The equation S3 (x) = has a root in (0, )
4
1
(D) tan ( Sn (x) )  , for all n  1 and x  0
2

8. Let  be a positive real number. Let 𝑓: ℝ ℝ and 𝑔: (𝛼 ℝ be the functions defined by

 x  2loge ( x −  )
f(x) = sin   and g(x) = .
 12  loge (e x − e  )
Then the value of lim f(g(x)) is__________. [JEE(Advanced)-2022]
x → +

1  1 
3
ex − 1 − x3( ) 3
(
+  1 − x2

) 2 − 1  sinx

9. If  = lim   , Then the value of 6 is __________.
x →0 x sin2 x
[JEE(Advanced)-2022]

30 Limit
2 Continuity
Continuous Functions
A function for which a small change in the independent variable causes only a small change and not a
sudden jump in the dependent variable are called continuous functions. Naively, we say that a function
is continuous at fixed point (in the domain of function) if we can draw the graph of the function around
the point without lifting the pen from the plane of the paper.

Continuity of a function at a point:


A function f(x) is said to be continuous at x = a, if Lim f(x) = f(a). Symbolically f is continuous at x = a
x →a

if Lim f(a h) = Lim f(a + h) = f(a) , h > 0


h→ 0 h→ 0

Example 1:
 x
sin , x  1
If f(x) =  2 then find whether f(x) is continuous or not at x = 1, where [ ]
 [x], x  1

denotes greatest integer function.
Solution:
 x
sin , x  1
f(x) =  2
 [x], x1

For continuity at x = 1, we determine, f(1), l im− f(x) and l im+ f(x).
x→1 x →1

Now, f(1) = [1] = 1


x 
l im− f(x) = l im− sin = sin = 1 and l im+ f(x) = l im+ [x] = 1
x→1 x→1 2 2 x →1 x →1

So f(1) = lim− f(x) = lim+ f(x)


x →1 x →1

 f(x) is continuous at x = 1

Continuity 31
Example 2:

 a(1 − x sin x) + bcos x + 5
 , x0
 x2
Let f(x) =  3, x=0
 1
   cx + dx3   x
  1 +    , x0
   x2 
If f is continuous at x = 0, then find out the values of a, b, c and d.
Solution:
Since f(x) is continuous at x = 0, so at x = 0, both left and right limits must exist and both
must be equal to 3.

a(1 − xsinx) + bcos x+ 5


(a + b + 5) +  −a − b2  x 2
+ ....
Now Lim− = Lim−   =3
x →0 x2 x →0 x2
(By the expansion of sinx and cosx)
b
If lim− f(x) exists then a + b + 5 = 0 and a =3a= 1 and b = 4
x →0 2
1
  cx + dx3   x cx + dx3
since lim+  1 +    exists  lim+ =0c=0
x →0  x  x2
2
x →0
  
d
1
 1

Now lim+ ( 1 + dx ) x = lim+ ( 1 + dx ) dx  = ed
x →0 x →0
 
So ed = 3  d = n 3,
Hence a = 1, b = 4, c = 0 and d = n 3.

Concept Builders - 1

cos x; x  0
(i) If f(x) =  find the value of k if f(x) is continuous at x = 0.
 x + k; x  0
 | x+ 2 |
 ; x  −2
(ii) If f(x) =  tan−1 (x+ 2) then discuss the continuity of f(x) at x = 2
 2 ; x = −2

Continuity of The Function in an Interval


(a) A function is said to be continuous in (a, b) if f is continuous at each and every point
belonging to (a, b).
(b) A function is said to be continuous in a closed interval [a,b] if :
(i) f is continuous in the open interval (a, b)
(ii) f is right continuous at 'a' i.e. Lim+ f(x) = f(a) = a finite quantity
x →a

(iii) f is left continuous at 'b' i.e. Lim− f(x) = f(b) = a finite quantity
x →b

32 Continuity
Note: (i) All polynomials, trigonometrical functions, exponential and logarithmic functions are
continuous in their domains.
(ii) If f(x) and g(x) are two functions that are continuous at x = c then the function defined
by:
F1(x) = f(x) ± g(x) ; F2(x) = Kf(x), where K is any real number ; F 3(x) = f(x). g(x) are also
continuous at x = c.
f(x)
Further, if g (c) is not zero, then F4(x) = is also continuous at x = c.
g(x)

Example 3:
 | x+ 1 | , x  −2

 2x + 3 , −2  x  0
Discuss the continuity of f(x) =  2
x +3 , 0x3
x3 − 15 , x3

Solution:
 −x − 1 , x  −2

 2x + 3 , −2  x  0
We write f(x) as f(x) =  2
x +3 , 0x3
x3 − 15 , x3

As we can see, f(x) is defined as a polynomial function in each of intervals
( , 2), ( 2, 0), (0, 3) and (3, ). Therefore, it is continuous in each of these four open intervals.
Thus, we check the continuity at x = 2, 0, 3.
At the point x = 2
lim f(x) = lim− ( x 1) = + 2 1=1
x → −2− x → −2

lim f(x) = lim+ (2x + 3) = 2. ( 2) + 3 = 1


x → −2+ x → −2

Therefore, lim f(x) does not exist and hence f(x) is discontinuous at x = 2.
x → −2

At the point x = 0
lim f(x) = lim− (2x + 3) = 3
x →0− x →0

lim f(x) = lim+ (x2 + 3) = 3


x →0+ x →0

f(0) = 0 + 3 = 3
2

Therefore f(x) is continuous at x = 0.


At the point x = 3
lim f(x) = lim− (x2 + 3) = 32 + 3 = 12
x → 3− x →3

lim f(x) = lim+ (x3 15) = 33 15 = 12


x → 3+ x →3

f(3) = 33 15 = 12
Therefore, f(x) is continuous at x = 3.
We find that f(x) is continuous at all points in R except at x = 2

Continuity 33
Concept Builders - 2

 x2
 ; 0x1
 a
(i) If f(x) =  −1 ; 1  x  2 then find the value of a and b if f(x) is continuous in [0,)
 2b2 − 4b
 ; 2x
 x2


| x − 3 | ; 0x1

 
(ii) Discuss the continuity of f(x) =  sin x ; 1  x in[0, 3)
 2
 
log  x ; 2
x3
 2

The Intermediate Value Theorem


Suppose f(x) is continuous on an interval I, and a and b are any two points of I. Then if y 0 is a number
between f(a) and f(b), there exists a number c between a and b such that f(c) = y 0

The function f, being continuous on [a, b]


Takes on every value between f(a) and f(b)
Note that a function f which is continuous in [a, b] possesses the following properties:
(i) If f(a) and f(b) possess opposite signs, then there exists at least one root of the equation
f(x) = 0 in the open interval (a, b).
(ii) If K is any real number between f(a) and f(b), then there exists at least one root of the
equation f(x) = K in the open interval (a, b).

Note: In above cases the number of roots is always odd.

Example 4:
a +b
Show that the function, f(x) = (x a)2 (x b)2 + x, takes the value for some x0  (a, b)
2

34 Continuity
Solution:
f(x) = (x a)2 (x b)2 + x
f(a) = a
f(b) = b
a +b
and  (f(a), f(b))
2
a +b
 By intermediate value theorem, there is at least one x0 (a, b) such that f(x0) = .
2

Example 5:

Let f : [0, 1] ⎯⎯⎯


onto
→ [0, 1] be a continuous function, then prove that f(x) = x for atleast one x 

[0, 1]
Solution:
Consider g(x) = f(x) x
g(0) = f(0) 0 = f(0)  0 { 0  f(x)  1 }

g(1) = f(1) 10


 g(0). g(1)  0
 g(x) = 0 has at least one root in [0, 1]
 f(x) = x for at least one x  [0, 1]

Concept Builders - 3

(i) If f(x) = xnx 2, then show that f(x) = 0 has exactly one root in the interval (1, e).

Some Important Points


(a) If f(x) continuous and g(x) is discontinuous at x = a then the product function
(x) = f(x).g(x) will not necessarily be discontinuous at x = a,
 
sin x0
Example: f(x) = x and g(x) =  x

 0 x=0

f(x) is continuous at x = 0 and g(x) is discontinuous at x = 0.


but f(x).g(x) is continuous at x = 0.
(b) If f(x) and g(x) both are discontinuous at x = a then the product function
(x) = f(x).g(x) is not necessarily be discontinuous at x = a,
1 x  0
Example: f(x) = g(x) = 
 −1 x  0
f(x) and g(x) both are discontinuous at x = 0 but the product function f(x).g(x) is still continuous
at x = 0

Continuity 35
(c) If f(x) and g(x) both are discontinuous at x = a then f(x) ± g(x) is not necessarily be discontinuous
at x = a
(d) A continuous function whose domain is closed must have a range also in closed interval.
(e) If f is continuous at x = a and g is continuous at x = f(a) then the composite g[f(x)] is continuous
x sinx
at x = a. eg. f(x) = and g(x) = |x| are continuous at x = 0, hence the composite function
x2 + 2
x sin x
(gof) (x) = will also be continuous at x = 0
x2 + 2

Concept Builders - 4

x if x  0
 3x if x  0

(i) Let f(x) =  and g(x) =  . Show that f + g is continuous at x = 0 even
2 if x = 0
 −2 if x = 0

though f and g are both discontinuous there.

Continuity Over Countable Set


Functions which are continuous only at one point are said to exhibit single point continuity
Example 8:
 x if x  Q
If f(x) =  , find the points where f(x) is continuous
 −x if x  Q
Solution:
Let x = a be the point at which f(x) is continuous.
 lim f(x) = lim f(x)
x →a x →a
through rational through irrational

a= a
 a = 0  function is continuous at x = 0.

Concept Builders - 5

x if x  Q
(i) If g(x) =  , then find the points where function is continuous.
0 if x  Q

 x
2
; x Q
(ii) If f(x) =  , then find the points where function is continuous.
1 − x ; x  Q
2

36 Continuity
ANSWER KEY FOR CONCEPT BUILDER

1. (i) 1 (ii) discontinuous at x = 2



2. (i) a = 1 and b = 1 (ii) Discontinuous at x = 1 and continuous at x =
2
1
5. (I) x=0 (ii) x=±
2

Continuity 37
Objective Exercise - I

 ax + 1 if x  1

1. Let f(x) =  3 if x = 1 . If f(x) is continuous at x = 1 then (a b) is equal to-
bx2 + 1 if x  1

(A) 0 (B) 1 (C) 2 (D) 4

px2 − px + q, x1

2. Let f(x) = x − 1, 1x 3
l x2 + mx + 2, x3

ql − m
If f(x) is continuous  x  R, then the value of is equal to
l
(A) 1 (B) 2 (C) 3 (D) 4

 sin x2
 , x0
 x
2

3. Let f(x) =  .
3 1
 + ,x=0
 4 4
If f(x) is continuous at x = 0, then  can be
(A) 1 (B) 2 (C) 3 (D) 4

4 − x2
4. The function f(x) = , is-
4x − x3
(A) Discontinuous at only one point in its domain.
(B) Discontinuous at two points in its domain.
(C) Discontinuous at three points in its domain.
(D) Continuous everywhere in its domain.

x2 − bx + 25
5. If f(x) = for x  5 and f is continuous at x = 5, then f(5) has the value equal to-
x2 − 7x + 10
(A) 0 (B) 5 (C) 10 (D) 25

x − ex + cos2x
6. If f(x) = , x  0 is continuous at x = 0, then
x2
5
(A) f(0) = (B) [f(0)] = 2
2
(C) {f(0)} = 0.5 (D) [f(0)]. {f(0)} = 1.5
where [.] and {.} denotes greatest integer and fractional part function

38 Continuity
 1

 (1 + tanx) − e ,
x

7. Let f(x) =  if x  0 If f(x) is continuous at x = 0, then the value of k is:
x


k, if x = 0

−e −e
(A) e (B) (C) (D) None
2 4

log e (1 + 3f(x))
8. y = f(x) is a continuous function such that its graph passes through (a,0). Then l im
x →a 2f(x)

is:
3 2
(A) 1 (B) 0 (C) (D)
2 3

1
9. For the function f(x) =  1 
, x  2 which of the following holds?
 
 x −2 
x+2
(A) f(2) = 1/2 and f is continuous at x = 2 (B) f(2)  0, 1/2 and f is continuous at x = 2
(C) f can not be continuous at x = 2 (D) f(2) = 0 and f is continuous at x = 2.

9 2  1 − cos 3x 
10. If f(x) is continuous and f   = , then the value of lim f   is-
2 9 x → 0
 x2 

2 9
(A) (B) (C) 0 (D) data insufficient
9 2

11. In [1, 3], the function [x2 + 1], [.] denoting the greatest integer function, is continuous:
(A) For all x (B) For all x except at nine points
(C) For all x except at seven points (D) For all x except at eight points

12. f is a continuous function on the real line. Given that x2 + (f(x) 2)x 3 ·f(x) + 2 3 3 = 0.

Then the value of f( 3 ):

(A) cannot be determined (B) is 2 (1 3)

2( 3 − 2)
(C) is zero (D) is
3

13. Let f : R → R be a continuous function  x  R and f(x) = 5  x  irrational. Then the value
of f (3) is:
(A) 1 (B) 2 (C) 5 (D) cannot determine

Continuity 39
| x+ 1 | if x  −2

2x + 3 if −2  x  0
14. Given f(x) =  2 . Then number of point(s) of discontinuity of f(x) is-
x + 3 if 0x3
x3 − 15 if x3

(A) 0 (B) 1 (C) 2 (D) 3

1 2
15. The function f: R/{0} → R given by f(x) = − can be made continuous at x = 0 by defining
x e2x − 1
f(0) as -
(A) 2 (B) 1 (C) 0 (D) 1

40 Continuity
Objective Exercise - II

Single Correct Type Questions

1. Let f : [0, 1] → R be a continuous function and assumes only rational values. If f(0) = 2 then the
  1   3  1 
value of tan 1  f    + tan 1  f    is:
  2   2  2 

  3
(A) (B) (C) (D) 
4 6 4

2x − 1
2. The function f(x) = [x]. cos , where [·] denotes the greatest integer function, is discontinuous
2
at:
(A) all x (B) all integer points
(C) no x (D) x which is not an integer

x2 + px + 1
3. Let f(x) = . If f(x) is discontinuous at exactly 2 values of x then number of integers
x2 − p

in the range of p is:


(A) 1 (B) 2 (C) 3 (D) 4

4. The function f(x) = [x]2 [x2] (where [y] is the greatest integer less than or equal to y), is
discontinuous at:
(A) all integers (B) all integers except 0 & 1
(C) all integers except 0 (D) all integers except 1

5. Number of points of discontinuity of f(x) = [2x3 5] in [1,2), is equal to:


(Where [x] denotes greatest integer less than or equal to x)
(A) 14 (B) 13 (C) 10 (D) 8

One or More than one Correct Type Questions

A sinx + sin2x
6. A function f(x) is defined as f(x) = , (x  0). If the function is continuous at
x3
x = 0, then:
(A) A = 2 (B) f(0) = 1 (C) A = 1 (D) f(0) = 1

Continuity 41
7. If f is defined on an interval [a, b]. Which of the following statement(s) is/are INCORRECT?
(A) If f(a) and f(b), have opposite sign, then there must be a point c  (a, b) such that f(c) = 0.
(B) If f is continuous on [a, b], f(a) < 0 and f(b) > 0, then there must be a point c  (a, b) such
that f(c) = 0.
(C) If f is continuous on [a, b] and there is a point c in (a, b) such that f(c) = 0, then f(a) and
f(b) have opposite sign.
(D) If f has no zeroes on [a, b], then f(a) and f(b) have the same sign.

8. In which of the following cases the given equations have atleast one root in the indicated interval?
(A) x cos x = 0 in (0, /2)
(B) x + sin x = 1 in (0, /6)
a b
(C) + = 0, a, b > 0 in (1, 3)
x−1 x−3
(D) f(x) g(x) = 0 in (a, b) where f and g are continuous on [a, b] and f(a) > g(a) and f(b) < g(b).

x
9. If f(x) = 1, then on the interval [0, ] ~ {2}, identify correct option(s) -
2
1
(A) tan (f(x)) and are both continuous
f(x)
1
(B) tan (f(x)) and are both discontinuous
f(x)
(C) tan (f(x)) and f 1(x) are both continuous
1
(D) tan (f(x)) is continuous but is discontinuous
f(x)

10. Which of the following function(s) is/are discontinuous at x = 0 ?


 
(A) f(x) = sin , x  0 and f(0) = 1 (B) g(x) = x sin   , x  0 and g(0) = 
2x x
|x| 1
(C) h(x) = , x  0 and h(0) = 1 (D) k(x) = , x   and k(0) = 0.
x 1 + ecot x

42 Continuity
Subjective Exercise - I

3x2 + ax + a + 3
1. If the function f(x) = is continuous at x = 2. Find f( 2):
x2 + x − 2

 tan 6x

  6  tan 5x 
  if 0  x 
 5 2


2. The function f(x) =  b+2 if x=
2
  a|tanx| 
( 1+ | cosx |) b  
if x
 2

Determine the values of 'a' & 'b', if f is continuous at x = /2.

 (x)
, x3
3. Suppose that f(x) = x 3
3x 2
4x + 12 and h(x) =  x − 3 then

 K , x=3
(a) find all zeroes of f(x).
(b) find the value of K that makes h continuous at x = 3.
(c) using the value of K found in (b) determine whether h is an even function.

 1 + x, 0  x  2
4. Let f(x) =  . Determine the form of g(x) = f[f(x)] & hence find the point of
3 − x, 2  x  3
discontinuity of g, if any.

 1 − sin3 x 
 if x 
 3cos x
2
2
  
5. Determine a & b so that f is continuous at x = where f(x) =  a if x =
2 2

 b(1 − sinx) 
if x 
 ( − 2 x)2 2

 sin(a + 1) x + sinx
 for x  0
 x
6. Determine the values of a, b & c for which the function f(x) =  c for x = 0 is

 (x+ bx ) − x
2 1/2 1/2
for x  0
 bx3/2
continuous at x = 0.

sin3x + A sin2x + Bsinx


7. If f(x) = (x  0) is Continuous at x = 0. Find A and B. Also find f(0).
x5

Continuity 43
 ax − b for x1

8. Find the locus of (a, b) for which the function f(x) =  3x for 1  x  2 is continuous at
bx2 − a for x2

x = 1 but discontinuous at x = 2.

ax2 + bx + c + enx
9. A function f : R → R is defined as f(x) = lim where f is continuous on R. Find
n→ 1 + c.enx
the values of a, b and c.

x3 7
10. Show that function f(x) = − sin x + 3 takes the value within the interval [ 2, 2].
4 3

44 Continuity
Subjective Exercise - II

1. Find all possible values of a and b so that f(x) is continuous for all x  R
 | ax+ 3 | if x  −1

 | 3 x+ a | if −1  x  0
f(x) =  b sin2x
 x − 2b if 0  x  

 cos x − 3 if x
2


 1 − sin x 1
, x
 1 + cos 2x 2
 1
2. Let f(x) =  p, x= .
 2
 2x − 1 1
 , x
 4 + 2x − 1 − 2 2

Determine the value of p, if possible, so that the function is continuous at x = 1/2.

3. Given the function g(x) = 6 − 2x and h(x) = 2x2 3x + a. Then


(a) evaluate h(g(2))
g(x), x  1
(b) If f(x) =  , find 'a' so that f is continuous.
h(x), x  1

4. If f(x) = x + { x} + [x], where [x] is the integral part & {x} is the fractional part of x. Discuss the
continuity of f in [ 2, 2].

 −
(sinx + cosx) ; x0
cos ecx

 2
5. Let f(x) =  a ; x=0 If f(x) is continuous at x = 0, find the value of (a2+ b2).
 1/x
 e +e +e
2/x 3/|x|

; 0x
 ae + be
2/x 3/|x|
2

Match the List Type


6. List-I List-II
x3 − 1
(P) lim is (1) 2
x→1 nx

(Q) lim
(
x cos x − cos2x ) is (2) 3
x →0 2sinx − sin2x
tanx tanx − sinx sinx 3
(R) lim is (3)
x→0
x 3
x 2
1 3
(S) If f(x) = cos (x cos ), x  0 and (4)
x 4
n(sec2 x)
g(x) = , x  0 are both continuous at x = 0
x sinx
then f(0) + g(0) equals

Continuity 45
JEE-Main (Previous Year Questions)


 sin(p+ 1) x + sinx
 , x0
 x
1. The values of p and q for which the function f(x) =  q , x=0

 x + x2 − x
, x0
 3
 x2
is continuous for all x in R are - [AIEEE 2011]
3 1 1 3
(1) p = ,q= (2) p = ,q=
2 2 2 2
1 3 5 1
(3) p = , q = − (4) p = , q =
2 2 2 2

2. Define F(x) as the product of two real functions f1(x) = x, x  R, and


 1
sin , if x  0 f (x).f2 (x) if x  0

f2(x) =  x as follows: F(x) =  1 [AIEEE 2011]
 0,
 if x = 0 0,
 if x = 0

Statement-1 : F(x) is continuous on .


Statement-2 : f1(x) and f2(x) are continuous on .
(1) Statemen-1 is false, statement-2 is true.
(2) Statemen-1 is true, statement-2 is true; Statement-2 is correct explanation for statement1.
(3) Statement-1 is true, statement-2 is true, statement-2 is not a correct explanation for statement1
(4) Statement-1 is true, statement-2 is false

 1 − cos 3x 
3. If f(x) is continuous and f(9/2) = 2/9, then lim f   is equal to: [JEE(Main)-2014]
x →0
 x2 
(1) 9/2 (2) 0 (3) 2/9 (4) 8/9

 2 + cos x − 1
 , x
4. If the function f(x) =  ( − x)2 is continuous at x = , then k equals:
 k, x=

[JEE(Main)-2014]
1 1
(1) (2) (3) 2 (4) 0
4 2

5. Let f : R → R be a function defined as :


5, if x  1

a + bx, if 1  x  3
= Then, f is: [JEE(Main)-2019]
b + 5x, if 3  x  5
30, if x  5

(1) continuous if a = 5 and b = 5 (2) continuous if a = 5 and b = 10
(3) continuous if a = 0 and b = 5 (4) not continuous for any values of a and b

46 Continuity
6. Let f : [ 1, 3] → R be defined as [JEE(Main)-2019]
| x | +[x], −1  x  1

f(x) = x+ | x |, 1  x  2
x + [x], 2  x  3,

where [t] denotes the greatest integer less than or equal to t. Then, f is discontinuous at:
(1) only two points (2) only three points
(3) only one point (4) four or more points

x
7. If f(x) = [x]   , x  R where [x] denotes the greatest integer function, then:
4
(1) lim− f(x) exists but lim+ f(x) does not exist [JEE(Main)-2019]
x→4 x→4

(2) lim+ f(x) exists but lim f(x) does not exist.
x→4 x→4

(3) f is continuous at x = 4.
(4) Both lim− f(x) and lim+ f(x) exist but are not equal.
x→4 x→4

a |  − x | +1, x  5

8. If the function f(x) = 
b | x−  | +3, x  5

Is continuous at x = 5, then the value of a b is: [JEE(Main)-2019]
2 2 −2 2
(1) (2) (3) (4)
−5 +5 +5 5−

 1 1
9. If the function f defined on  − ,  by
 3 3
1  1 + 3x 
 log e   , when x  0
f(x) =  x  1 − 2x 
 k, when x = 0

is continuous, then k is equal to ............... [JEE(Main)-2020]

4
10. Let [t] denote the greatest integer  t and limx   = A . Then the function, f(x) = [x2] sin(x) is
x →0
x
discontinuous, when x is equal to: [JEE(Main)-2020]
(1) A (2) A+5 (3) A + 21 (4) A+1

x
11. Let f(x) = x.   , for 10 < x < 10, where [t] denotes the greatest integer function. Then the
2
number of points of discontinuity of f is equal to ________. [JEE(Main)-2020]

Continuity 47
 2x − 1 
12. If f : R → R is a function defined by f(x) = [x − 1]cos   , where [.] denotes the greatest
 2 
integer function, then f is : [JEE(Main)-2021]
(1) discontinuous only at x = 1
(2) discontinuous at all integral values of x except at x = 1
(3) continuous only at x = 1
(4) continuous for every real x

  x 
 −  
  2 

13. Let f : R → R be defined as = 2
+ +   .If f (x) is continuous on R, then
 sin( x) if x  1



a + b equals : [JEE(Main)-2021]
(1) 3 (2) 1 (3) 3 (4) 1

 cos−1 (1 − {x} 2 ) sin−1 (1 − {x})


 x0
14. Let   R be such that the function f(x) =  {x} − {x} 3 is continuous at
, x=0

x = 0, where {x} = x [x], [x] is the greatest integer less than or equal to x. Then:
[JEE(Main)-2021]
 
(1)  = (2)  = 0 (3) no such  exists (4)  =
2 4

x + a,
 x0 x + 1, x0
15. Let f : R → R and g : R → R be defined as f(x) =  and g(x) = 
( x − 1) + b,
2
| x − 1 |,
 x0 x0

where a, b are non-negative real number. If (gof) (x) is continuous for all x  R , then a + b is
equal to _____. [JEE(Main)-2021]

cos(sinx) − cos x 1
16. if the function f(x) = is continuous at each point in its domain and f(0) = ,
x 4
k
then k is. [JEE(Main)-2021]

 sin(a + 1)x + sin2x


 ,if x  0
 2x
17. Let f : R → R be a function defined as f(x) =  b , if x = 0 . If f is continuous

 x + bx − x
3
, if x  0
 bx 5/2

at x = 0, then the value of a + b is equal to: [JEE(Main)-2021]


5 3
(1) − (2) 2 (3) 3 (4) −
2 2

48 Continuity
log e (1 − x + x2 ) + log e (1 + x + x2 )  −  
 ,x   ,  − {0}
18. If the function f(x) =  sec x − cos x  2 2 is continuous at x = 0, then
 k ,x = 0

k is equal to: [JEE(Main)-2022]
(1) 1 (2) 1 (3) e (4) 0

sin2 t
 12 1 1

19. lim  1sin t + 2sin t + .... + nsin t  is equal to: [JEE(Main)-2023]
2 2

t →0  
 
n ( n + 1)
(1) n2 (2) n2 + n (3) (4) n
2

  
(1+ | cos x |) , 0x
 cos x 2

 
20. If the function f(x) =   , x= [JEE(Main)-2023]
 2
 cot 6x
 
ecot 4x , x
 2


is continuous at x = , then 9 + 6loge + 6 e6 is equal to :
2
(1) 10 (2) 2e4 + 8 (3) 8 (4) 11

21. Let f(x) = [x2 x] + | x + [x]|, where x  R and [t] denotes the greatest integer less than or equal
to t. Then, f is [JEE(Main)-2023]
(1) continuous at x = 0, but not continuous at x = 1
(2) continuous at x = 0 and x = 1
(3) not continuous at x = 0 and x = 1
(4) continuous at x = 1, but not continuous at x = 0

22. Let [x] be the greatest integer  x. Then the number of points in the interval ( 2, 1), where the
function f(x) = | [x] | + x − [x] is discontinuous is________. [JEE(Main)-2023]

Continuity 49
JEE-Advanced (Previous Year Questions)

 e1/(x − 1) − 2
 , x1
1. Discuss the continuity of the function f(x) =  e1/(x − 1) + 2 at x = 1. [REE(Main)-2001]
 1, x=1

2. For every integer n, let an and bn be real numbers. Let function f : R → R be given by
an + sin x, for x  [2n,2n + 1]
f(x) =  , for all integers n.
bn + cos x, for x  (2n − 1,2n)
If f is continuous, then which of the following holds(s) for all n? [JEE(Advanced)-2012]
(A) an 1 bn 1 = 0 (B) an bn = 1 (C) an bn+1 = 1 (D) an 1 bn = 1

3. For every pair of continuous function f, g : [0, 1] → R such that


max{f(x) : x  [0, 1]} = max{g(x) : x  [0,1]},
the correct statement(s) is(are) :
(A) (f(c))2 + 3 f(c) = (g(c))2 + 3g(c) for some c  [0, 1]
(B) (f(c))2 + f(c) = (g(c))2 + 3g(c) for some c  [0, 1]
(C) (f(c))2 + 3f(c) = (g(c))2 + g(c) for some c  [0, 1]
(D) (f(c))2 = (g(c))2 for some c  [0,1] [JEE(Advanced)-2014]

4. Let [x] be the greatest integer less than or equals to x. Then, at which of the following point(s)
the function f(x) = x cos ((x + [x])) is discontinuous? [JEE(Advanced)-2017]
(A) x = 1 (B) x = 0 (C) x = 2 (D) x = 1

50 Continuity
3 Differentiability
Meaning of Derivative

The instantaneous rate of change of a function with respect to the dependent variable is called
derivative. Let 'f' be a given function of one variable and let x denotes a number (positive or negative)
to be added to the number x. Let f denote the corresponding change of 'f' then f = f(x + x) f(x)
f f(x + x) − f(x)
 =
x x
If f/x approaches a limit as x approaches zero, this limit is the derivative of 'f' at the point x. The
derivative of a function 'f' is a function; this function is denoted by symbols such as
df d df(x)
f'(x), , f(x) or
dx dx dx
df f f(x + x) − f(x)
 = lim = lim
dx  x → 0 x x → 0 x
df(x)
The derivative evaluated at a point a, is written, f'(a), , f'(x)x=a, etc.
dx x =a

Existence of Derivative At x = a

(a) Right Hand Derivative


The right hand derivative of f(x) at x = a denoted by f'(a+) is defined as :
f(a + h) − f(a)
f'(a+) = Lim , provided the limit exists and is finite. (h > 0)
h→ 0 h

Differentiability 51
(b) Left Hand Derivative
The left-hand derivative of f(x) at x = a denoted by f'(a ) is defined as:
f(a − h) − f(a)
f'(a ) = Lim , provided the limit exists and is finite. (h > 0)
h→ 0 −h
Hence f(x) is said to be derivable or differentiable at x = a. If f'(a+) = f'(a ) = finite quantity
and it is denoted by f'(a) ; where f'(a) = f '(a ) = f'(a+) and it is called derivative or
differential coefficient of f(x) at x = a.

Differentiability and Continuity

Theorem: If a function f(x) is derivable at x = a, then f(x) is continuous at x = a.


f(a + h) − f(a)
Proof: f'(a) = Lim exists.
h→ 0 h
f(a + h) − f(a)
Also f(a + h) f(a) = .h [h  0]
h
f(a + h) − f(a)
 Lim [f(a + h) f(a)] = Lim = f'(a).0 = 0
h→ 0 h→ 0 h
 Lim[f(a + h) f(a)] = 0  L im f(a + h) = f(a)  f(x) is continuous at x = a.
h→0 h→ 0

Note: (i) Differentiable  Continuous; Continuous  Differentiable; Not differentiable  Not


Continuous But Not Continuous  Not Differentiable
(ii) All polynomial, trigonometric, logarithmic and exponential function are continuous and
differentiable in their domains.
(iii) If f(x) and g(x) are differentiable at x = a then the function f(x) + g(x), f(x) g(x),
f(x).g(x) will also be differentiable at x = a and if g(a)  0 then the function f(x)/g(x)
will also be differentiable at x = a.

Example 1:

sgn(x) + x; −  x  0
  

Let f(x) =  −1 + sin x; 0  x  .Discuss the continuity and differentiability at x = 0 and .
 2 2
 
 cos x; 2
x

Solution:

 −1 + x; −  x  0
 

f(x) = −1 + sin x; 0x
 2
 
 cos x; 2
x

To check the differentiability at x = 0

52 Differentiability
− − −1 + 0 − h − (−1)
LHD = lim = lim =1
h→ 0 −h h→ 0 −h
+ − −1 + sinh+ 1
RHD = lim = lim =1
h→ 0 h h→0 h
 LHD = RHD
 Differentiable at x = 0.
 Continuous at x = 0.

To check the continuity at x =
2
LHL lim− f(x) = lim− ( 1 + sin x) = 0
 
x→ x→
2 2

RHL lim+ f(x) = lim+ cos x = 0


 
x→ x→
2 2


 LHL = RHL = f   = 0
2  

 Continuous at x = .
2

To check the differentiability at x =
2
  
 − −  
2  2  = lim −1 + cosh− 0 = 0
LHD = lim  
h→0 −h h→ 0 −h
  
 + −  
2  2  = lim − sinh− 0 = −1
RHD = lim  
h→0 h h→0 h
 LHD  RHD

 not differentiable at x = .
2

Example 2:
A + Bx2
 x1
If f(x) =  then find A and B so that f(x) become differentiable at x = 1.
3Ax − B + 2 x  1

Solution:
f(1 + h) − f(1) 3A(1 + h) − B + 2 − 3A + B − 2 3Ah
f'(1+) = lim = lim = lim = 3A
h→ 0 h h →0 h h→ 0 h
f(1 − h) − f(1) A + B(1 − h2 ) − 3 A + B− 2 (−2 A + 2B− 2) + Bh2 − 2Bh
f'(1 ) = lim = lim = lim
h→ 0 −h h→ 0 −h h→ 0 −h
hence for this limit to be defined
2A + 2B 2=0

Differentiability 53
B=A+1
f'(1 ) = lim (Bh 2B) = 2B
h→ 0

 f'(1 ) = f'(1+)
3A = 2B = 2(A + 1)
A = 2, B = 3

Example 3:

[cos  x] x  1

f(x) =  comment on the derivability at x = 1, where [ ] denotes greatest integer
 2{x} − 1 x  1

function and { } denotes fractional part function.
Solution:
− − [cos( −  h)] + 1 −1 + 1
f'(1 ) = lim = lim = lim =0
h→0 −h h→ 0 −h h → 0 −h
+ − 2{1 + h} − 1 + 1 2h
f'(1+) = lim = lim = lim =2
h→0 h h→0 h h → 0 h
Hence f(x) is not differentiable at x = 1.

Concept Builders - 1

(i) A function is defined as follows:



x ; x  1
3 2

f(x) =  discuss the continuity and differentiability at x = 1.


x ; x  1
2


 ax3 + b, for 0x1
(ii) If f(x) =  be the differentiable function in [0, 2], then find a and
2cos x + tan x, for 1x 2
−1

b. (where [ . ] denotes the greatest integer function)

Important Note

(a) Let f'(a+) = p and f'(a ) = q where p and q are finite then:
(i) p = q  f is differentiable at x = a  f is continuous at x = a
(ii) p  q  f is not differentiable at x = a, but f is continuous at x = a.

Example 4:
Determine the values of x for which the following functions fails to be continuous or
 (1 − x), x1

differentiable f(x) = (1 − x)(2 − x), 1  x  2 , Justify your answer.
(3 − x), x2

54 Differentiability
Solution:
By the given definition it is clear that the function f is continuous and differentiable at all
points except possibly at x = 1 and x = 2.
Check the differentiability at x = 1
− − 1 − (1 − h) − 0
q = LHD = lim = lim = −1
h→0 −h h→ 0 −h
+ − {1 − (1 + h)}{2 − (1 + h)} − 0
p = RHD = lim = lim = 1
h→0 h h →0 h
 q = p  Differentiable at x = 1.  Continuous at x = 1.
Check the differentiability at x = 2
− − (1 − 2 + h)(2 − 2 + h) − 0
q = LHD = lim = lim = 1 = finite
h→0 −h h→ 0 −h
+ − (3 − 2 − h) − 0
p = RHD = lim = lim → (not finite)
h→0 h h→ 0 h
 q  p  not differentiable at x = 2.
Now we have to check the continuity at x = 2
LHL = = lim− (1 x) (2 x) = lim (1 (2 h)) (2 (2 h)) = 0
x →2− x →2 h→ 0

RHL = lim+ f(x) = lim+ (3 x) = lim (3 (2 + h)) = 1


x →2 x →2 h→ 0

 LHL  RHL
 not continuous at x = 2.

Concept Builders - 2

(i) Let f(x) = (x 1) |x 1|. Discuss the continuity and differentiability of f(x) at x = 1.

(b) Vertical Tangent

If y = f(x) is continuous at x = a and lim f ' ( x ) approaches to , then y = f(x) has a vertical
x →a

tangent at x = a. If a function has vertical tangent at x = a then it is not differentiable at x = a.

y
f(x)= x1/3

o x

Differentiability 55
Example: (i) f(x) = x1/3 has vertical tangent at x = 0 since f'(0+) →  and f'(0 ) →  hence
f(x) is not differentiable at x = 0
g(x) = x2/3 doesn't have vertical tangent at x = 0
y

g(x) = x2/3

x
O
since g'(0+) →  and g'(0 ) →  hence g(x) is not differentiable at x = 0.

(c) Geometrical Interpretation of Differentiability


(i) If the function y = f(x) is differentiable at x = 0, then a unique tangent can be
drawn to the curve y = f(x) at the point P(a, f(a)) and f'(a) represent the slope
of the tangent at point P.
(ii) If the function f(x) does not have a unique tangent (p  q) but is continuous at
x = a. it geometrically implies a sharp corner at x = a. Note that p and q may
not be finite, where p = f'(a+) and q = f'(a )
Example: (1) f(x) = |x| and |x|1/3 is continuous but not differentiable at x = 0 and there
is sharp corner at x = 0.
y

f(x) = |x|1/3

(does not have unique tangent)

O x

y
f(x) = |x|
(does not have unique tangent)
x
O

(2) f(x) = x1/3 is continuous but not differentiable at x = 0 because f'(0+) → and f'(0 ) →.

y
f(x) = x1/3

x
O
(have a unique tangent but does not have sharp corner)

Note: sharp corner  non-differentiable non-differentiable 


/ sharp corner

56 Differentiability
Example 5:

 x−3
 x0
If f(x) =  2 . Draw the graph of the function and discuss the continuity and
x − 3x + 2 x  0

differentiability of f(|x|) and |f(x)|.
Solution:

y=f(|x|)

x
-2 -1 0 1 2


 | x | −3; | x |  0 → not possible
f(|x|) =  2
| x | −3 | x | +2;
 | x | 0


x + 3x + 2, x  0
2
f(|x|) =  2
x − 3x + 2, x  0

at x = 0
− − h2 − 3h + 2 − 2
q = LHD = lim = lim =3
h→ 0 −h h→ 0 −h
+ − h2 − 3h + 2 − 2
p = RHD = lim = lim = 3
h→ 0 h h→ 0 h

qp
 not differentiable at x = 0. but p and q both are finite
 continuous at x = 0
y
 3−x , x0 3 y=|f(x)|
 2
 (x − 3 x+ 2) , 0x1 2
Now, |f(x)| =  2
−(x − 3 x+ 2) , 1 x 2
x
 (x2 − 3 x+ 2) , x2 O 1 2

To check differentiability at x = 0
− − + − + 
q = LHD = lim = lim = lim → − 
h→ 0 −h h→ 0 −h h→0 −h 
2   not differentiableat x = 0.
+ − − + − 
p = RHD = lim = lim = −3
h →0 h h →0 h 

Now to check continuity at x = 0
= = = 
x →0− x → 0− 
2   not continuous at x = 0.
= = − + = 
x →0+ x →0+ 
To check differentiability at x = 1

Differentiability 57
f(1 − h) − f(1)
q = LHD = lim
h→ 0 −h
(1 − h2 ) − 3(1 − h) + 2 − 0 h2 + h
= lim = lim = 1
h →0 −h h→ 0 −h
f(1 + h) − f(1) −(h2 + 2h+ 1 − 3 + 3h+ 2) − 0 −(h2 − h)
p = RHD = lim = lim = lim =1
h→ 0 h h→0 h h→0 h
 not differentiable at x = 1.
but |f(x)| is continuous at x = 1, because p  q and both are finite.
To check differentiability at x = 2
f(2 − h) − f(2)
q = LHD = lim
h→0 −h
−(4 + h2 − 4h − 6 + 3h + 2) − 0 h2 − h
= lim = lim = 1
h→ 0 −h h→ 0 h
f(2 + h) − f(2) (h2 + 4h+ 4 − 6 − 3h+ 2) − 0 (h2 + h)
p = RHD = lim = lim = lim =1
h→0 h h→0 h h→ 0 h
 not differentiable at x = 2.
but |f(x)| is continuous at x = 2, because p  q and both are finite.

Concept Builders - 3

 −4 ;
 −4  x  0
(i) Let f(x) =  2
x − 4 ; 0  x  4

Discuss the continuity and differentiability of g(x) = |f(x)|.
(ii) Let f(x) = min {|x 1|, |x + 1|, 1}. Find the number of points where it is not differentiable.

Differentiability Over an Interval

(a) f(x) is said to be differentiable over an open interval (a, b) if it is differentiable at each
and every point of the open interval (a, b).
(b) f(x) is said to be differentiable over the closed interval [a, b] if:
(i) f(x) is differentiable in (a, b) and
(ii) for the points a and b, f'(a+) and f'(b ) exist.

Example 6:
 e−|x| , −5  x  0
 −|x − 1|
If f(x) = −e + e + 1, 0  x  2
−1

 e −|x −2|
, 2x4

Discuss the continuity and differentiability of f(x) in the interval ( 5, 4).

58 Differentiability
Solution:
 e+ x −5  x  0
 x−1
−e + e + 1 0  x  1
−1

f(x) =  − x + 1 −1
 −e e + 1 1x 2
 e − x +2
2x4

Check the differentiability at x = 0
− − e −h − 1
LHD = lim = lim =1
h→ 0 −h h→ 0 −h
+ − −eh− 1 + e−1 + 1 − 1
RHD = lim = lim = e 1
h→ 0 h h→0 h

 LHD  RHD

 Not differentiable at x = 0, but continuous at x = 0 since LHD and RHD both are finite.
Check the differentiability at x = 1
− − −e1−h− 1 + e−1 + 1 − e−1
LHD = lim = lim = 1
h→0 −h h→ 0 −h
+ − −e1−h− 1 + e−1 + 1 − e−1
RHD = lim = lim =1
h→0 h h→ 0 h

 LHD  RHD
 Not differentiable at x = 1, but continuous at x = 1 since LHD and RHD both are finite.
Check the differentiability at x = 2
− − −e−2+h+ 1 + e−1 + 1 − 1 −e−1 (eh − 1)
LHD = lim = lim = lim =e 1
h→0 −h h→0 −h h→ 0 −h
+ − e −h − 1
RHD = lim = lim = 1
h→0 h h→ 0 h

 LHD  RHD

 Not differentiable at x = 2, but continuous at x = 2 since LHD and RHD both are finite.

Note:
(i) If f(x) is differentiable at x = a and g(x) is not differentiable at x = a, then the product function
F(x) = f(x).g(x) can still be differentiable at x = a.
e.g. Consider f(x) = x and g(x) = |x|, f is differentiable at x = 0 and g is non-differentiable at
x = 0, but f(x).g(x) is still differentiable at x = 0.

(ii) If f(x) and g(x) both are not differentiable at x = a then the product function ; F(x) = f(x).g(x) can
still be differentiable at x = a.
e.g., Consider f(x) = |x| and g(x) = |x|. f & g are both non differentiable at x = 0, but f(x).g(x) still
differentiable at x = 0.

Differentiability 59
(iii) If f(x) and g(x) both are non-differentiable at x = a then the sum function F(x)=f(x)+g(x) may be
a differentiable function.
e.g. f(x) = |x| & g(x) = |x|. f & g are both non differentiable at x = 0, but (f + g)(x) still differentiable
at x = 0.

(iv) If f(x) is differentiable at x = a  f'(x) is continuous at x = a.


 2 1
x sin if x  0
e.g. f(x) =  x

 0 if x = 0

Concept Builders - 4

5
(i) Let f(x) = max {sinx, 1/2}, where 0  x  . Find the number of points where it is not
2
differentiable.

 [x]
 ; 0x2
(ii) Let f(x) =  , where [.] denotes greatest integer function.

2x − 2 ; 2x3
(a) Find that points at which continuity and differentiability should be checked.
(b) Discuss the continuity and differentiability of f(x) in the interval (0, 3).

Determination of Function Which Satisfying the Given Functional Rule

Example 7:
Let f(x + y) = f(x) + f(y) 2xy 1 for all x and y. If f'(0) exists and f'(0) = sin , then find f{f'(0)}.
Solution:
f(x+ h) − f(0)
f '(x) = lim
h→ 0 h
{f(x) + f(h) − 2 xy − 1} − f(x)
= lim (Using the given relation)
h→ 0 h
f(h) − 1 f(h) − f(0)
= lim 2x + lim = 2x + lim
h→ 0 h→ 0 h h → 0 h
[Putting x = 0 = y in the given relation we find f(0) = f(0) + f(0) + 0 1  f(0) = 1]
 f'(x) = 2x + f'(0) = 2x sin 
 f(x) = x2 (sin ). x + c
f(0) = 0 0 + c  c = 1
 f(0) = x2 (sin ). x + 1
So, f{f'(0)} = f( sin ) = sin2 + sin2 + 1
 f{f'(0)} = 1.

60 Differentiability
ANSWER KEY FOR CONCEPT BUILDER

1. (i) Continuous but not differentiable at x = 1

1  13
(ii) a= ,b= −
6 4 6

2. (i) Continuous and differentiable at x = 1

3. (i) Continuous everywhere but not differentiable at x = 2 only

(ii) 5

4. (i) 3

(ii) (a) 1 and 2

(b) Not continuous at x = 1 and 2 and not differentiable at x = 1 and 2.

Differentiability 61
Objective Exercise - I

1. Let f(x) = [tan2x], (where [.] denotes greatest integer function). Then:
(A) lim f(x) does not exist (B) f(x) is continuous at x = 0.
x→0

(C) f(x) is not differentiable at x = 0 (D) f'(0) = 1

3x2 − 4 x + 1 for x  1
2. Let g(x) =  . If g(x) is continuous and differentiable for all numbers in
ax + b for x  1
its domain then:
(A) a = b = 4 (B) a = b = 4
(C) a = 4 and b = 4 (D) a = 4 and b = 4

x + b, x  0
3. The function g(x) =  can be made differentiable at x = 0
cos x x  0
(A) if b is equal to zero (B) if b is not equal to zero
(C) if b takes any real value (D) for no value of b

4. Which one of the following functions is continuous everywhere in its domain but has atleast
one point where it is not differentiable?
|x|
(A) f(x) = x1/3 (B) f(x) = (C) f(x) = e x
(D) f(x) = tanx
x

5. If the right-hand derivative of f(x) = [x] tan x at x = 7 is k, then k is equal to


([y] denotes greatest integer  y)
(A) 6 (B) 7 (C) 7 (D) 49


 x + 2x
3 2
x Q
6. Let f(x) =  3 , then the integral value of 'a' so that f(x) is differentiable at
−x + 2x + ax x  Q
2

x = 1, is:
(A) 2 (B) 6 (C) 7 (D) not possible

7. The number of points where f(x) = [sin x + cos x] (where [·] denotes the greatest integer function),
x  (0,2) is not differentiable is:
(A) 3 (B) 4 (C) 5 (D) 6

f(h) − f(−2h)
8. Let f be differentiable at x = 0 and f'(0) = 1. Then lim =
h→0 h
(A) 3 (B) 2 (C) 1 (D) 1

g(1 + x) − g(1)
9. Let g(x) = min.(x, x2) where x  R, then lim equals:
x →0 x
(A) 0 (B) 1 (C) 2 (D) does not exist

62 Differentiability

2x + tan x + a, −   x  0
−1
10. Let g(x) =  3 .
x + x + bx, 0x
2

If g(x) is differentiable for all x  ( , ) then (a2 + b2) is equal to:
(A) 20 (B) 13 (C) 9 (D) 4

x + 2, x0

11. Let f(x) =  −(2 + x2 ), 0x1
x, x1

Then the number of points where |f(x)| is non-derivable is:
(A) 3 (B) 2 (C) 1 (D) 0

12. The function f(x) = (x2 1) | x2 3x + 2 | + cos (|x|) is NOT differentiable at:
(A) 1 (B) 0 (C) 1 (D) 2

 ax + b, −  x  2

13. If the function f(x) = x2 − 5x + 6, 2x3 is differentiable in ( , ), then:
px2 + qx + 1, 3x

−4 5 −5
(A) a = 1, p = (B) b = 2, q = (C) a = 1, b = 2 (D) a = 1, q =
9 3 3

14. Let R be the set of real numbers and f: R → R, be a differentiable function such that
| f(x) f(y)|  |x y|3x,y R. If f(10) = 100, then the value of f(20) is equal to:
(A) 0 (B) 10 (C) 20 (D) 100

15. If f(x) f(y) + 2 = f(x) + f(y) + f(xy) and f(1) = 2, f'(1) = 2 then sgn f(x) is equal to
(where sgn denotes signum function):
(A) 0 (B) 1 (C) 1 (D) 4

16. Number of points of non-differentiability of the function


g(x) = [x2]{cos24x} + {x2}[cos2 4x] + x2 sin2 4x + [x2][cos2 4x] + {x2}{cos2 4x} in ( 50, 50) where [x]
and {x} denotes the greatest integer function and fractional part function of x respectively, is
equal to:
(A) 98 (B) 99 (C) 100 (D) 0

17. Let f(x) = [n + p sin x], x  (0, ), n  I and p is a prime number. The number of points where
f(x) is not differentiable is:
(A) p 1 (B) p + 1 (C) 2p + 1 (D) 2p 1

Differentiability 63
Objective Exercise - II

Single Correct Type Questions

1. Let f : R →R be a continuous onto function satisfying f(x) + f( x) = 0,  x  R. If f( 3) = 2 and


f(5) = 4 in [ 5,5], then the equation f(x) = 0 has-
(A) exactly three real roots (B) exactly two real roots
(C) atleast five real roots (D) atleast three real roots

  x 
n

 ax(x− 1)  cot  + (px + 2)


2

  4 
 lim , x  (0, 1)  (1,2)
2. Let f(x) = n →   x 
n

  cot  +1
  4 
0 , x=1

If f(x) is differentiable for all x  (0,2) then (a2 + p2) equals -
(A) 18 (B) 20 (C) 22 (D) 24

3. If 2x + 3|y| = 4y, then y as a function of x i.e. y = f(x), is -


(A) discontinuous at one point
(B) non-differentiable at one point
(C) discontinuous& non-differentiable at same point
(D) continuous& differentiable everywhere

4. If f(x) = (x5 + 1) |x2 4x 5| + sin|x| + cos(|x 1|), then f(x) is not differentiable at -
(A) 2 points (B) 3 points (C) 4 points (D) zero points

5. For what triplets of real numbers (a, b, c) with a  0 the function:


 x x1
f(x) =  2 is differentiable for all real x ?
ax + bx + c otherwise
(A) {(a, 1 2a, a) | a  R, a  0} (B) {(a, 1 2a, c) | a, c  R, a  0}
(C) {(a, b, c) | a, b, c  R, a + b + c = 1} (D) {(a, 1 2a, 0) | a  R, a  0}

x , x  [0, 1)

x − 1, x  [1, 2)
6. Let f(x) = [x] and g(x) =  .
x − 2, x  [2, 3)
0, x=3

Then f(x) + g(x) is
[Note: [k] denotes the greatest integer function less than or equal to k.]
(A) discontinuous at x = 1 and x = 2.
(B) continuous in [0, 3] but non-derivable in [0,3].
(C) not twice differentiable in [0, 3].
(D) twice differentiable in [0, 3]

64 Differentiability
One or More than one Correct Type

| x − 3 |, x1

7. The function f(x) =  x2   3x   13  is:
  −   +  ,x  1
 4
   2   4 
(A) continuous at x = 1 (B) differentiable at x = 1
(C) continuous at x = 3 (D) differentiable at x = 3

8. If f(x) = sgn(x5), then which of the following is/are false (where sgn denotes signum function):
(A) f'(0+) = 1
(B) f'(0 ) = 1
(C) f is continuous but not differentiable at x = 0
(D) f is discontinuous at x = 0

9. Graph of f(x) is shown in adjacent figure, then in [0, 5]


y
2
1
x
0 1 2 3 4 5

(A) f(x) has non removable discontinuity at two points


(B) f (x) is non differentiable at four points
(C) lim f(f(x)) = 1
x→1

(D) Number of points of discontinuity = number of points of non-differentiability

10. Let S denotes the set of all points where 5


x2 | x3 | − 3 x2 | x | − 1 is not differentiable then S is a
subset of:
(A) {0, 1} (B) {0, 1, 1} (C) {0, 1} (D) {0}

11. Which of the following statements is/are correct ?


(A) There exist a function f : [0, 1] → R which is discontinuous at every point in [0,1] & | f(x)| is
continuous at every point in [0, 1]
(B) Let F(x) = f(x). g(x). If f(x) is differentiable at x = a, f(a) = 0 and g(x) is continuous at x = a
then F(x) is always differentiable at x = a.
(C) If f'(a+) = 2 & f'(a ) = 3, then f(x) is non-differentiable at x = a but will be always continuous
at x = a
(D) If f(a) and f(b) possess opposite signs then there must exist at least one solution of the
equation f(x) = 0 in (a, b) provided f is continuous on [a, b]

Differentiability 65
12. Let f : R → R be a function. Define g : R → R by g(x) = |f(x)| for all x. Then which of the following
is/are not always true-
(A) If f is continuous then g is also continuous
(B) If f is one-one then g is also one-one
(C) If f is onto then g is also onto
(D) If f is differentiable then g is also differentiable

 (x− m)
 , x0
 | x − m |
13. If function defined by f(x) = 2x2 + 3ax + b , 0  x  1 , is continuous & differentiable
 m2 x + b − 2 , x1


everywhere, then:
(A) b + m = 1 (B) b + m = 1 (C) b + m = 3 (D) m2 + a + b = 3

14. The function (x) = [| x | sin | x |] (where [.] denotes greater integer function) is -
(A) derivable at x = 0
(B) continuous at x = 0
(C) lim (x)does not exists
x →0

(D) continuous and derivable at x = 0

 2 1
x cos , x0
 x
15. Let f(x) =  0 , x = 0 , then which of the following is (are) correct ?
 1
 x2 sin , x0
 x
(A) f(x) is continuous but not differentiable at x = 0
(B) f(x) is continuous and differentiable at x = 0
(C) f' (x) is continuous but not differentiable at x = 0
(D) f' (x) is discontinuous at x = 0

16. Let [.] denotes the greatest integer function.


List I List II
(P) If P(x) = [2 cos x], x  [ , ], then P(x) (1) is discontinuous at exactly 7 points
(Q) If Q(x) = [2 sin x], x  [ , ], then Q(x) (2) is discontinuous at exactly 4 points
  
(R) If R(x) = [2 tan x/2], x   − , , then R(x) (3) is non differentiable at some points
 2 2 
 x  
(S) If S(x) = 3cosec  , x   , 2 , then S(x) (4) is continuous at infinitely many values
 3 2 

66 Differentiability
17. List I List II

(P) f(x) = |x3| is (1) continuous in ( 1, 1)

(Q) f(x) = | x | is (2) differentiable in ( 1, 1)

(R) f(x) = |sin 1 x| is (3) differentiable in (0, 1)

(S) f(x) = cos 1 |x| is (4) not differentiable atleast at one

point in ( 1, 1)

Differentiability 67
Subjective Exercise - I

Subjective Type Questions

1. Discuss the continuity & differentiability of the function f(x) = sin x + sin | x |, x  R. Draw a
rough sketch of the graph of f(x).

2. Examine the continuity and differentiability of f(x) = | x | + | x 1|+|x 2 | x  R. Also draw


the graph of f(x).

 x2
 − for x0
3. If the function f(x) defined as f(x) =  2 is continuous but not derivable at
xn sin 1 for x0
 x
x = 0 then find the range of n.

 1 for −  x  0

 
 1+ | sin x | for 0x
4. A function f is defined as follows: f(x) =  2
2
   
2 +  x −  for  x  +
  2 2

Discuss the continuity & differentiability at x = 0 & x = /2.

5. Examine the origin for continuity & derivability in the case of the function f defined by
f(x) = x tan 1 (1/x), x  0 and f(0) = 0.

6. Let f(0) = 0 and f' (0) = 1. For a positive integer k, show that
1 x  x  1 1 1
l im  f(x) + f   + ....... + f    = 1 + + + ......+
x →0 x 2  k  2 3 k

 1 1
− + 
 |x| x 
7. Let f(x) = xe ; x  0, f(0) = 0, test the continuity & differentiability at x = 0

ax2 − b if | x | 1
8. If f(x) =  1 is derivable at x = 1. Find the values of a & b.
− if | x | 1
 | x |

x+y f(x) + f(y) + f(0)


9. If f is a differentiable function such that f   = ,  x, y  R and
 3  3
f'(0) = 2, find f (x)

68 Differentiability
10. If f(x) = | x 1 | . ([x] [ x]), then find f' (1+) & f ' (1 ) where [x] denotes greatest integer function.

 −1, −2  x  0
11. Let f(x) be defined in the interval [ 2, 2] such that f(x) =  and g(x) = f(| x |) + | f (x)|.
x − 1, 0  x  2

Test the differentiability of g(x) in ( 2, 2).

Differentiability 69
Subjective Exercise - II

Integer Type Questions

 x
1 − cos  1 − cos 
 2
1. If lim is equal to the left-hand derivative of e |x|
at x = 0, then find the value
x →0 2 x
m n

of (n 10m)

2. Let f (x) be a differentiable function such that 2f(x + y) + f(x y) = 3f(x) + 3f(y) + 2xy  x,
y R & f'(0) = 0, then f(10) + f'(10) is equal to

a x + 2, 0  x  2
3. Let g(x) =  . If g(x) is derivable on (0, 5), then find (2a + b).
 bx + 2, 2  x  5

f(3 − sinx) − f(3 + x)


4. Let f(x) is a differentiable function and if lim = 8, then | '(3) | is
x→0 x

5. The number of points of non differentiability of the function f (x) = |sin x| + sin |x| in [ 4, 4]
is

 sin [x2 ] 
 + ax3 + b , 0  x  1
6. If f (x) =  x2 − 3x + 8 is differentiable in [0 , 2] , then the value of [a + b + 6]
 2cos  x + tan− 1 x , 1  x  2

is (Here [ . ] stands for the greatest integer function)

 x2 e
2(x − 1)
for 0  x  1
7. If f(x)=  is differentiable at x = 1 then
a sgn (x + 1) cos (2x − 2) + bx for 1  x  2
2

a3 + b3 =

8. Let [x] denote the greatest integer less than or equal to x. The number of integral points in
[ 1, 1] where f(x) = [x sin x] is differentiable are

9. Let f : R → R is a function satisfying f(10 x) = f(x) and f(2 x ) = f(2 +x), x  R. If f(0) = 101,
then the minimum possible number of values of x satisfying f(x) = 101 for x  [0,30] is

n
10. Find the natural number 'a' for which  f(a + k)
k=1
= 2048(2n

the relation f(x + y) = f(x) . f(y) for all natural numbers x & y and further f(1) = 2

70 Differentiability
Comprehension (Q. 11 to Q. 13)
 x g(x)
 , x0
Let f(x) =  , where g(t) = lim (1 + a tan x)t/x, a is positive constant, then
x + ax − x , x0
2 3 x →0

11. If a is even prime number, then g(2) =
(A) e2 (B) e3 (C) e4 (D) none of these

12. Set of all values of a for which function f(x) is continuous at x = 0


(A) ( 1, 10) (B) ( , ) (C) (0, ) (D) none of these

13. If f(x) is differentiable at x = 0, then a 


(A) ( 5, 1) (B) ( 10, 3) (C) (0, ) (D) none of these

Differentiability 71
JEE-Main (Previous Year Questions)

1. Let f(x) = x|x|, g(x) = sinx and h(x) = (gof)(x). Then [JEE(Mains)-2014]
(1) h' (x) is differentiable at x = 0
(2) h' (x) is continuous at x = 0 but is not differentiable at x = 0
(3) h(x) is differentiable at x = 0 but h' (x) is not continuous at x = 0
(4) h(x) is not differentiable at x = 0

2. Let f : R → R be a function such that | f(x)|  x2, for all x  R. Then, at x = 0 , f is:
(1) Neither continuous nor differentiable [JEE(Mains)-2014]
(2) Differentiable but not continuous
(3) Continuous as well as differentiable
(4) Continuous but not differentiable


max{| x |, x },
2
| x | 2
3. Let f(x) = 
 8 − 2 | x |,
 2  | x | 4

Let S be the set of points in the interval ( 4, 4) at which f is not differentiable. Then S:
[JEE(Mains)-2019]
(1) equals ( 2, 1, 1, 2) (2) equals { 2, 2}
(3) is an empty set (4) equals { 2, 1, 0, 1, 2}

4. Let f : ( 1, 1) → R be a function defined by f(x) = max {− | x |, − 1 − x2 }. If K be the set of all points


at which f is not differentiable, then K has exactly: [JEE(Mains)-2019]
(1) one element (2) two elements (3) five elements (4) three elements

5. Let K be the set of all real values of x where the function f(x) = sin|x| |x| + 2(x ) cos |x| is
not differentiable. Then the set K is equal to: [JEE(Mains)-2019]
(1) {} (2)  (an empty set) (3) {0} (4) {0, }

6. Let S be the set of all points in ( , ) at which the function, f(x) = min {sinx, cosx} is not
differentiable. Then S is a subset of which of the following? [JEE(Mains)-2019]
         3  3    3   3 
(1)  − , − , ,  (2)  − ,0,  (3) − ,− , ,  (4) − ,− , , 
 2 4 4 2  4 4   4 4 4 4   4 2 2 4

7. Let f : R → R be differentiable at c  R and f(3) = 0, If g(x) = | f (x)|, then at x = c, g is:


[JEE(Mains)-2019]
(1) differentiable if f'(3) = 0 (2) not differentiable
(3) not differentiable if f'(3) = 0 (4) differentiable if f'(3)  0

72 Differentiability
aex + be− x , −1  x  1

8. If a function f(x) defined by f(x) = cx2 , 1  x  3 be continuous for some a, b, cR and
ax2 + 2cx , 3x4

f'(0) + f'(2) = e, then the value of a is: [JEE(Mains)-2020]
e e 1 e
(1) (2) (3) (4)
e2 − 3e + 13 e2 + 3e + 13 e2 − 3e + 13 e2 − 3e − 13

9. Suppose a differentiable function f(x) satisfies the identity f(x + y) = f(x) + f(y) + xy2 + x2y, for
f(x)
all real x and y. If lim = 1 , then f'(3) is equal to ______. [JEE(Mains)-2020]
x →0 x

t2f2 (x) − x2 f2 (t)


10. Let f : (0, ) → (0, ) be a differentiable function such that f(1) = e and lim = 0.
t →x t−x
If f(x) = 1, then x is equal to: [JEE(Mains)-2020]
1 1
(1) 2e (2) (3) (4) e
2e e


k (x − ) − 1, x  
2
11. If the function f(x) =  1 is twice differentiable, then the ordered pair (k1, k2) is
 k2 cos x,
 x

equal to: [JEE(Mains)-2020]


1  1 
(1)  , −1  (2)  , 1  (3) (1, 0) (4) (1, 1)
 2  2 

12. Let f : R → R be a function defined by f(x) = max{x, x2}. Let S denote the set of all points in R,
where f is not differentiable. Then: [JEE(Mains)-2020]
(1) {0, 1} (2) {0} (3) (an empty set) (4) {1}

13. The number of points, at which the function f(x) = | 2x + 1 | −3 | x + 2 | + | x2 + x − 2 | , x  R is not

differentiable, is. [JEE(Main)-2021]

x + 2,
 x0
14. Let the function f : R → R and g : R → R be defined as : f(x) =  2 and
x ,
 x0


x
3
x1
g(x) = =  Then, the number of point in R where (f o g) (x) is NOT differentiable is
3x − 2,
 x1

equal to: [JEE(Main)-2021]


(1) 3 (2) 1 (3) 0 (4) 2

Differentiability 73
 1
 ,|x| 1
15. If f(x) = | x | is differentiable at every point of the domain, then the value of a
ax2 + b ,|x| 1

and b are respectively: [JEE(Main)-2021]
1 1 1 3 5 3 1 3
(1) , (2) ,− (3) ,− (4) − ,
2 2 2 2 2 2 2 2

 − 2
 
16. A function f is defined on [ 3, 3] as f(x) =  where [x] denotes the
 [| x |] , 2 | x | 3

greatest integer  x. The number of points, where f is not differentiable in ( 3,3) is.
[JEE(Main)-2021]

17. Let f : R → R satisfy the equation f(x + y) = f(x).f(y) for all x, y  R and f(x)  0 for any x  R .If
1
lim (f(h) − 1) is equal to .
h→ 0 h

[JEE(Main)-2022]

3x2 + k x + 1, 0  x  1
18. Let k and m be positive real numbers such that the function f(x) =  is
2 2
 mx + k , x1

8f '(8)
differentiable for all x > 0. Then is equal to _____ [JEE(Main)-2023]
 1
f' 
8

19. Let [x] denote the greatest integer function and f(x) = max {1 + x + [x], 2 + x, x + 2[x]}, 0  x  2.
Let m be the number of points in [0, 2], where f is not continuous and n be the number of
points in (0, 2), where f is not differentiable. Then (m + n)2 + 2 is equal to: [JEE(Main)-2023]
(1) 11 (2) 2 (3) 6 (4) 3

 x[x]
 , −2  x  0
20. Let f : ( 2, 2) → R be defined by f(x) =  Where [x] denotes the greatest
(x − 1)[x] ,0  x  2

integer function. If m and n respectively are the number of points in ( 2, 2) at which y = |f(x)| is
not continuous and not differentiable, then m + n is equal to__________. [JEE(Main)-2023]

21. Let f: R → R, be a differentiable function that satisfies the relation f(x + y) = f(x) + f(y) 1,
x, y  R . If f'(0) = 2, then |f( 2)| is equal to__________ [JEE(Main)-2023]

22. Let a  Z and [t] be the greatest integer  t. Then the number of points, where the function
f(x) = [a + 13 sin x], x  (0,  ) is not differentiable, is __________ [JEE(Main)-2023]

74 Differentiability
JEE-Advanced (Previous Year Questions)

1. Let f1 :R → R, f2 : [0, ) → R, f3 : R → R and f4 : R → [0, ) be defined by


| x | if x  0,

f1(x) =  x
e
 if x  0;
f2(x) = x2 ;
sinx if x  0,
  f (f (x)) if x  0,
f3(x) =  and f4(x) =  2 1

 x if x  0 f (f
2 1 (x)) − 1 if x  0
List-I List-II
P. f4 is 1. onto but not one-one
Q. f3 is 2. neither continuous nor one-one
R. f2 o f1 is 3. differentiable but not one-one
S. f2 is 4. continuous and one-one
[JEE(Advanced)-2014]
Codes:
P Q R S
(A) 3 1 4 2
(B) 1 3 4 2
(C) 3 1 2 4
(D) 1 3 2 4

2. Let f :R→R and g : R→R be respectively given by f(x) = |x| + 1 and g(x) = x2 + 1. Define
max {f(x), g(x)} if x  0,

h :R→R by h(x) = 
 min{f(x), g(x)} if x  0.

The number of points at which h(x) is not differentiable is [JEE(Advanced)-2014]

3. Let a, bR and f:R→R be defined by f(x) = a cos(|x3 x|) + b|x| sin(|x3 + x|). Then f is -
(A) Differentiable at x = 0 if a = 0 and b = 1
(B) Differentiable at x = 1 if a = 1 and b = 0
(C) NOT differentiable at x = 0 if a = 1 and b = 0
(D) NOT differentiable at x = 1 if a = 1 and b = 1 [JEE(Advanced)-2016]

 1   1 
4. Let f :  − ,2 →Rand g :  − ,2 →R be function defined by f(x) = [x2 3] and g(x) = |x|
 2   2 
f(x) + |4x 7| f(x), where [y] denotes the greatest integer less than or equal to y for y R . Then.
[JEE(Advanced)-2016]
 1 
(A) f is discontinuous exactly at three points in  − ,2
 2 
 1 
(B) f is discontinuous exactly at four points in  − ,2
 2 
 1 
(C) g is NOT differentiable exactly at four points in  − , 2 
 2 
 1 
(D) g is NOT differentiable exactly at five points in  − , 2 
 2 

Differentiability 75
5. Let f : R → R be a differentiable function with f(0) = 1 and satisfying the equation
f(x + y) = f(x) f(y) + f (x) f (y) for all x, y R. Then, the value of loge(f(4)) is _______.

[JEE(Advanced)-2018]

    

6. Let f1 : R → R, f2 :  − ,  → R, f3 :  −1,e 2 − 2  → R and f4 : R → R be functions defined by
 2 2  
 
[JEE(Advanced)-2018]

f1(x) = sin  1 − e− x  ,
2
(i)
 
 | sinx |
 if x  0
(ii) f2(x) =  tan−1 x , where the inverse trigonometric function tan 1x assumes values
 1 if x = 0

  
in  − ,  .
 2 2
(iii) f4(x) = [sin(loge(x + 2))], where, for t  R, [t] denotes the greatest integer less than or

equal to t,
 2  1
x sin   if x  0
(iv) f4 (x) =  x
 0 if x = 0

List-I List-II
(P) The function f1 is (1) NOT continuous at x = 0
(Q) The function f2 is (2) continuous at x = 0 and NOT

(R) The function f3 is differentiable at x = 0

(S) The function f4 is (3) differentiable at x = 0 and its

derivative is NOT continuous at x = 0


(4) differentiable at x = 0 and its derivative
is continuous at x = 0
The correct option is:
(A) (P)→(2); (Q)→(3); (R)→(1); (S)→(4) (B) (P)→(4); (Q)→(1); (R)→(2); (S)→(3)
(C) (P)→(4); (Q)→(2); (R)→(1); (S)→(3) (D) (P)→(2); (Q)→(1); (R)→(4); (S)→(3)

7. Let the function f : R → R be defined by f(x) = x3 x2 + (x 1) sin x and let g : R → R be an


arbitrary function. Let fg : R → R be the product function defined by (f g) (x) = f(x) g(x). Then
which of the following statements is/are TRUE? [JEE(Advanced)-2019]
(A) If g is continuous at x = 1, then fg is differentiable at x = 1
(B) If fg is differentiable at x = 1, then g is continuous at x = 1
(C) If g is differentiable at x = 1, then fg is differentiable at x = 1
(D) If fg is differentiable at x = 1, then g is differentiable at x = 1

76 Differentiability
8. Let f : → and g : → be functions satisfying f(x + y) = f(x) + f(y) + f(x) f(y) and
f(x) = xg(x) for all x, y  . If lim g(x) = 1, then which of the following statements is/are TRUE?
x →0

[JEE(Advanced)-2019]
(A) f is differentiable at every x 
(B) If g(0) = 1, then g is differentiable at every x 
(C) The derivative f '(1) is equal to 1
(D) The derivative f '(0) is equal to 1

9. Let S = (0,1)  (1,2)  (3,4) and T = {0, 1, 2, 3}. Then which of the following statements is(are)
true? [JEE(Advanced)-2023]
(A) There are infinitely many functions from S to T
(B) There are infinitely many strictly increasing functions from S to T
(C) The number of continuous functions from S to T is at most 120
(D) Every continuous function from S to T is differentiable

Differentiability 77
4 Methods of Differentiation
Definition

The Process of Calculating Derivative is Called Differentiation.

Derivative of F(x) From the First Principle

y f(x+  x) − f(x) dy
Obtaining the derivative using the definition L im = L im = f'(x) = is called
x →0 x x →0 x dx
calculating derivative using first principle or ab-initio or delta method.

dy dy
Note: can also be represented as y1 or y' or Dy or f'(x) represents instantaneous rate of change
dx dx
of y w.r.t. x.

Example 1:

Differentiate each of following functions by first principle:

(i) f(x) = tanx

(ii) f(x) = esinx

Solution:

tan(x+ h) − tanx tan(x+ h− x)[1 + tanx tan(x+ h)]


(i) f'(x) = lim = lim
h→ 0 h h → 0 h
tanh
= lim . (1 + tan2x) = sec2x. Ans.
h→ 0 h
sin x
esin(x +h) − e esin(x +h)−sinx − 1  sin(x+ h) − sinx 
(ii) f'(x) = lim = limesin x  
 
h→ 0 h h→0 sin(x+ h) − sinx  h 

sin(x+ h) − sinx
= esin x lim = esinxcosx Ans.
h→ 0 h

Concept Builders - 1

(i) Differentiate each of following functions by first principle:

(a) f(x) = nx

1
(b) f(x) =
x

Methods of Differentiation 79
Derivative of Standard Functions

f(x) f '(x) f(x) f '(x)

(i) xn nxn 1
(ii) ex ex

(iii) ax axna, a > 0 (iv) nx 1/x

(v) logax (1/x) logae, a > 0, a  1 (vi) sinx cosx

(vii) cosx sinx (viii) tanx sec2x

(ix) secx secxtanx (x) cosecx cosecx . cotx

(xi) cotx cosec2x (xii) constant 0

1 −1
(xiii) sin–1 x , –1 < x < 1 (xiv) cos–1 x ,–1 < x < 1
1− x 2
1 − x2

1 1
(xv) tan 1 x ,xR (xvi) sec 1 x ,| x | 1
1 + x2 | x | x2 − 1

−1 −1
(xvii) cosec 1 x , |x| > 1 (xviii) cot 1 x ,x  R
|x| x −1 2
1 + x2

Fundamental Theorems

If f and g are derivable functions of x, then,


d df dg
(a) (f ± g) = ±
dx dx dx
d df
(b) (cf) = c , where c is any constant
dx dx
d dg df
(c) (fg) = f + g known as "PRODUCT RULE"
dx dx dx
 df   dg 
g  −f 
d f  dx   dx 
(d)   = where g  0 known as "QUOTIENT RULE"
dx  g  g2
dy dy du
(e) If y = f(u) and u = g (x) then = . known as "CHAIN RULE"
dx du dx

dy du
Note: In general, if y = f(u) then = f '(u). .
dx dx

80 Methods of Differentiation
Example 2:
dy
If y = ex tan x + xlogex, find .
dx
Solution:
y = ex.tan x + x · logex
On differentiating we get,
dy 1
= ex · tan x + ex · sec2x + 1 · logex + x ·
dx x
dy
Hence, = ex(tanx + sec2 x) + (logex + 1) Ans.
dx

Example 3:
loge x dy
If y = + ex sin2x + log5x, find .
x dx

Solution:

On differentiating we get,

dy d  log e x  d x d
=   + (e sin2x) + (log5 x)
dx dx  x  dx dx

1
.x − log x.1
x 1
= + exsin2x + 2ex. cos2x +
x 2
xlog e 5

dy  1 − log e x  1
Hence, =   + ex(sin2x + 2cos2x) + Ans.
dx  x 2
 xlog e 5

Example 4:

dy
If y = loge(tan 1
1 + x2 ), find .
dx

Solution:

y = loge (tan 1
1 + x2 )

On differentiating we get,

1 1 1
= . . .2x
tan −1
1+ x
2
1+( 1+ x ) 2 2
2 1 + x2

x x
= = Ans.
( tan −1
1+ x 2
) ( 
1 + 1 + x

2
)
2

 1+ x

2
( tan
−1
1+ x 2
)( 2 + x2 ) 1 + x2

Methods of Differentiation 81
Concept Builders - 2

dy
(i) Find if -
dx
(a) y = (x + 1) (x + 2) (x + 3)
(b) y = e5x tan(x2 + 2)

Logarithmic Differentiation
If a function is non-zero and differentiable in some interval, then its derivative can be formed by first
taking logarithm and then differentiating.
To find the derivative of a function:
(a) which is the product or quotient of a number of functions or
(b) of the form [f(x)]g(x) where f and g are both derivable functions.
It is convenient to take the logarithm of the function first and then differentiate.

Example 5:
dy
If y = (sin x)n x, find
dx
Solution:
n y = n x. n (sin x)

On differentiating we get,
1 dy 1 cos x dy  n(sinx) 
= n (sinx) + n x.  = (sinx)n x  + cot x nx  Ans.
y dx x sin x dx  x 

Example 6:
  y − x2   dy
If x = exp  tan−1    , then equals -
 dx
 x 
2

(A) x [1 + tan (log x) + sec2 x] (B) 2x [1 + tan (log x)] + sec2 x
(C) 2x [1 + tan (log x)] + sec x (D) 2x + x[1 + tan(logx)]2
Solution:
Taking log on both sides, we get
 y − x2 
log x = tan 1    tan (log x) = (y x2) /x2  y = x2 + x2 tan (log x)
 x 
2

On differentiating, we get
dy
 = 2x + 2x tan (log x) + x sec2 (log x)
dx
dy
 2x [1 + tan (log x)] + x sec2 (log x) = 2x + x[1 + tan(logx)]2 Ans. (D)
dx

82 Methods of Differentiation
Example 7:
x 1/2 (1 − 2 x)2/3 dy
If y = find
(2 − 3x)3/4 (3 − 4 x)4/5 dx
Solution:
1 2 3 4
n y = n x + n (1 2x) n (2 3x) n (3 4x)
2 3 4 5
On differentiating we get,
1 dy 1 4 9 16
 = + +
y dx 2x 3(1 − 2 x) 4(2 − 3 x) 5(3 − 4 x)
dy  1 4 9 16 
= y − + +  Ans.
dx  2x 3(1 − 2 x) 4(2 − 3 x) 5(3 − 4 x) 

Concept Builders - 3

dy
(i) Find if y = xx
dx
dy 2 3 4
(ii) Find if y = ex. ex . ex . ex
dx

Parametric Differentiation

dy dy / d f ' ( )
If y = f() and x = g() where  is a parameter, then = =
dx dx / d g ' ( )

Example 8:
dy 
If y = a cos t and x = a(t sint) find the value of at t =
dx 2
Solution:
dy −a sin t dy
=  = 1 Ans.
dx a(1 − cos t) dx t = 
2

Example 9:
Prove that the function represented parametrically by the equations.
1+ t 3 2 dy
x= ;y= + satisfies the relationship: x(y')3 = 1 + y' (where y' = )
t 3
2t 2
t dx
Solution:
1+ t 1 1
Here x = = 3 + 2
t 3
t t
Differentiating w.r. to t
dx 3 2
=
dt t4 t3

Methods of Differentiation 83
3 2
y= +
2t 2
t
Differentiating w.r.to t
dy 3 2
=
dt t3 t2
dy dy / dt
= = t = y'
dx dx / dt
1+ t 1 + y'
Since x = x= or x(y')3 = 1 + y' Ans.
t3 ( y ')
3

Derivative of a Function w.r.t. Another Function


dy dy / dx f '(x)
Let y = f (x) ; z = g (x) then = =
dz dz / dx g '(x)
Example 10:
Differentiate loge (tan x) with respect to sin 1(ex).
Solution:
d
d(loge tanx) (log e tanx)
dx cot x. sec2 x e− x 1 − e2x
= = = Ans.
d(sin−1 (ex )) d ex .1 / 1 − e2x sinx cos x
sin−1 (ex )
dx

Concept Builders - 4

dy 
(i) Find at t = if y = cos4t and x = sin4t.
dx 4
(ii) Find the slope of the tangent at a point P(t) on the curve x = at 2, y = 2at.
(iii) Differentiate xnx with respect to nx.

Differentiation of Implicit Functions: (x, y) = 0


(a) To find dy /dx of implicit functions, we differentiate each term w.r.t. x regarding y as a function
of x and then collect terms with dy/dx together on one side.

dy x , where  = partial derivative of (x, y) w.r.t. x taking y as a constant and
(b) Also =
dx  x
y

= partial derivative of (x, y) w.r.t. y taking x as a constant.
y
(c) In the case of implicit functions, generally, both x and y are present in answers of dy/dx.

Example 11:
dy
If xy + yx = 2, then find .
dx

84 Methods of Differentiation
Solution:
Let u = xy and v = yx
du dv
u+v=2  + =0
dx dx
Now u = xy and v = yx
 n u = y nx and n v = x n y
1 du y dy 1 dv x dy
 = + nx and = n y +
u dx x dx v dx y dx
du y dy  dv  x dy 
 = xy  + nx  and = yx  ny + 
dx x dx  dx  y dx 
y dy   x dy 
 xy  + nx  + y  ny +
x
 =0
x dx   y dx 
 x y y
 y ny + x . 
dy  x
 = Ans.
dx  y x x
 x nx + y . 
 y
Aliter:
(x,y) = xy + yx 2 = 0
dy − /  x  yx y − 1 + y x ny 
= =  y x−1 
dx  / y  x nx + xy 

Example 12:
sinx dy (1 + y)cosx+ ysinx
If y = , prove that = .
cos x dx 1 + 2y + cos x − sinx
1+
sinx
1+
1 + cos x......
Solution:
sin x (1 + y) sinx
Given function is y = =
cos x 1 + y + cos x
1+
1+ y
or y + y2 + y cos x = (1 + y) sin x ......(i)

Differentiate both sides with respect to x,


dy dy dy dy
+ 2y + cos x y sin x = (1 + y) cos x + sinx
dx dx dx dx
dy
(1 + 2y + cosx sinx) = (1 + y) cosx + ysinx
dx
dy (1 + y)cos x+ ysinx
or = Ans.
dx 1 + 2y + cos x − sinx
Aliter:
From (i) (x,y) = (1 + y)sinx y y2 ycosx = 0
dy  / x (1 + y)cos x+ ysinx (1 + y)cosx+ ysinx
= = =
dx  / y sinx − 1 − 2y − cos x 1 + 2y + cos x − sinx

Methods of Differentiation 85
Concept Builders - 5

dy
(i) Find , if x + y = sin(x y)
dx
(ii) If x2 + xey + y = 0, find y', also find the value of y' at point (0, 0).

Differentiation by Trigonometric Transformation


Some Standard Substitutions:
Expression Substitution

a2 − x2 x = asin or acos

a2 + x2 x = atan or acot

x2 − a2 x = asec or acosec

a+x a−x
or x = acos or acos2
a−x a+x

2ax − x2 x = a(1 cos)

Example 13:
 2x 
If f(x) = sin 1  2 
then find
1+ x 
 1
(i) f '(2) (ii) f '   (iii) f '(1)
2
Solution:
 
x = tan, where <<  y = sin 1(sin2)
2 2
 
 − 2  2  
 2  − 2 tan−1 x x1
 −  
y = 2  2   f(x) = 2 tan x
−1
−1x  1
 2 2 −( + 2 tan−1 x)
   x  −1
−( + 2) −   2  − 2

 2
− x1
 1+ x
2

 2
 f '(x) =  −1 x  1
1 + x
2

 −2
 1 + x2 x  −1

2
(i) f '(2) =
5
 1 8
(ii) f '  =
2
  5
(iii) f '(1+) = 1 and f '(1 ) = +1  f '(1) does not exist Ans.

86 Methods of Differentiation
Example 14:
d  2  1 + x  
sin  cot
−1
 =
dx   1 − x  

1 1
(A) (B) 0 (C) (D) 1
2 2
Solution:
 1+ x   
Let y = sin2  cot −1  . Put x = cos 2    0, 
 1 − x   2

 1 + cos 2 
 y = sin2cot 1   = sin2cot 1(cot )
 1 − cos 2 
 
1 − cos 2 1− x 1 x
 y = sin2 = = =
2 2 2 2
dy 1
 = . Ans. (A)
dx 2

Example 15:

1 + x2 − 1 1 + 1 + x2
Obtain differential coefficient of tan 1
with respect to cos 1
x 2 1 + x2
Solution:

1 + x2 − 1 , 1 + 1 + x2
Assume u = tan 1
v = cos 1
x 2 1 + x2
  
The function needs simplification before differentiation Let x = tan; ,  − , 
 2 2
 sec  − 1  1 1 − cos 
    
 u = tan 1   = tan   = tan  tan  =
1

 tan    sin    2 2
1 + sec  1 + cos    
v = cos 1
= cos 1
= cos 1  cos  =  u = v
2 sec  2  2  2
du
 = 1. Ans.
dv

Concept Builders - 6

(i) If y = cos 1(4x3 3x), then find:


 3
(a) f '  − ,
 2 
 
(b) f '(0),
 3
(c) f '  
 2 
 

Methods of Differentiation 87
Derivative of a Function and its Inverse Function
If g is inverse of f, then
(a) g{f(x)} = x (b) f{g(x)} = x
g'{f(x)}f '(x)=1 f'{g(x)}g'(x) = 1

Example 16:
1
If g is inverse of f and f '(x) = , then g '(x) equals:
1 + xn
(A) 1 + xn (B) 1 + [f(x)]n (C) 1 + [g(x)]n (D) none of these
Solution:
Since g is the inverse of f. Therefore
f(g(x)) = x for all x
d
 f(g(x)) = 1for all x
dx
 f '(g(x)) g'(x) = 1
1
 g'(x) = = 1 + (g(x))n Ans. (C)
f '(g(x))

Concept Builders - 7

(i) If g is inverse of f and f (x) = 2x + sinx; then g'(x) equals:


3 1 1
(A) + (B) 2 + sin 1x (C) 2 + cos (g(x)) (D)
x2 1− x 2 2 + cos(g(x))

Higher Order Derivatives


Let a function y = f (x) be defined on an interval (a, b). If f (x) is differentiable function, then its
derivative f '(x) [or (dy/dx) or y'] is called the first derivative of y w.r.t. x. If f '(x) is again differentiable
function on (a, b), then its derivative f "(x) [or d2y/dx2 or y"] is called second derivative of y w.r.t. x.

d3 y d  d2 y 
Similarly, the 3rd order derivative of y w.r.to x, if it exists, is defined by =   and denoted
dx3 dx  dx2 

by f '''(x) or y''' and so on.

dy dy / d d2 y d  dy  dx
Note: If x = f() and y = g() where '' is a parameter then = and =  
dx dx / d dx2 d  dx  d

dn y d  dn− 1 y  dx
In general, =   .
dxn d  dxn− 1  d

88 Methods of Differentiation
Analysis and Graphs of Some Inverse Trigonometric Functions
2 tan−1 x ; |x|  1
 2x  
(a) y = f(x) = sin 
1
2 
=   − 2 tan x ;
−1
x1
1+ x   −( + 2 tan−1x); x  −1

Important Points
  
(i) Domain is x  R and range is  − , 
 2 2
(ii) f is continuous for all x but not
differentiable at x = 1, 1
 2
 for | x | 1
 1 + x 2
dy
(iii) = non existent for| x |= 1
dx  −2
 for | x | 1
 1 + x2
(iv) Increasing in ( 1, 1) and Decreasing in ( , 1)  (1, )

 1 − x2  2 tan−1 x if x0
(b) Consider y = f(x) = cos 1   = 
1+ x 
2
 −2 tan x if x0
−1

Important Points
(i) Domain is x  R and range is [0, )
(ii) Continuous for all x but not differentiable
at x = 0
 2
 for x  0
1 + x
2
dy
(iii) = non existent for x = 0
dx  2
− for x  0
 1 + x2
(iv) Increasing in (0,) and Decreasing in ( ,0)
 2 tan−1 x | x | 1
2x   + 2 tan−1 x x  −1
(c) y = f(x) = tan 1 = 
1 − x2
 −( − 2 tan−1x) x  1

Important Points
  
(i) Domain is R {1, 1} and range is  − , 
 2 2
(ii) It is neither continuous nor differentiable
at x = 1, 1

Methods of Differentiation 89
dy
 2
(iii) =  | x | 1
dx 1 + x2
nonexistent
 | x |= 1

(iv) Increasing  x in its domain

(v) It is bounded for all x

 1
 −( + 3sin x) if −1  x  −
−1

 2
1 1
(d) y = f(x) = sin 1(3x 4x3) =  3sin−1 x if −  x 
 2 2
  − 3sin−1 x if 1
x1
 2

Important Points
  
(i) Domain is x  [ 1, 1] and range is  − , 
 2 2
(ii) Continuous everywhere in its domain
1 1
(iii) Not derivable at x = − ,
2 2
 3 1 1
if x  (− , )
dy  1 − x2 2 2
(iv) =
dx  − 3 1 1
if x  (−1, − )  ( , 1)
 2 2
 1 − x2
 1 1  1  1 
(v) Increasing in  − ,  and Decreasing in  −1, −    , 1
 2 2  2 2 

 1
3cos x − 2 if −1  x  −
−1

 2
1 1
(e) y = f(x) = cos 1 (4x3 3x) = 2 − 3cos x if −  x 
 −1

 2 2
 3cos−1 x 1
if x1
 2

Important Points

(i) Domain is x  [ 1, 1] and range is [0,]

(ii) Continuous everywhere in its domain

1 1
(iii) Not derivable at x = ,
2 2

 3  1 1
 if x  − , 
dy  1 − x 2
 2 2
(iv) =
dx  − 3 if
 1  1 
x   −1, −    , 1 
 2
 1− x 2
  2 
 1 1  1  1 
(v) Increasing in  − ,  and Decreasing in  −1, −    , 1
 2 2   2  2 

90 Methods of Differentiation
General Note
Concavity is decided by the sign of 2nd derivative as:
d2 y d2 y
0  Concave upwards; <0  Concave downwards
dx2 dx2

Example 17:
Find the interval for which f(x) = x3 + x + 1 is
(i) concave upwards (ii) concave downwards.
Solution:
f(x) = x3 + x + 1
f '(x) = 3x2 + 1
f ''(x) = 6x
(i) f ''(x) = 6x > 0  Concave upwards  x  (0, )
(ii) f ''(x) = 6x < 0  Concave downwards  x  ( , 0) Ans.

Example 18:
d2 y
If x = a (t + sin t) and y = a(1 cos t), find .
dx2
Solution:
Here x = a (t + sin t) and y = a (1 cos t)
Differentiating both sides w.r.t. t, we get:
dx dy
= a(1 + cos t) and = a(sin t)
dt dt
t t
2sin .cos

dy
=
a sint
= 2 2 = tan  t 
 
dx a(1 + cos t)
2cos2
t 2
2
Again, differentiating both sides, we get,
t
sec2  
d2 y  t 
= sec2   . .
1 dt 1
= sec2 (t/2).
1
=
1
. 2
dx  2  2 dx 2 a(1 + cos t) 2a t
2

2  cos2 
 2 
dy
2
1 t
Hence, = .sec4   Ans.
dx 4a 2
2

Example 19:

y = f(x) and x = g(y) are inverse functions of each other than express g'(y) and g''(y) in terms of

derivative of f(x).

Methods of Differentiation 91
Solution:
dy dx
= f '(x) and = g '(y)
dx dy

1
 g '(y) = ...........(i)
f '(x)
Again differentiating w.r.t. to y
d  1  d  1  dx f "(x)  1 
g "(y) =   =  . = . 
dy  f '(x)  dx  f '(x)  dy (f '(x))2  f '(x) 

f "(x)
 g "(y) = ...........(ii)
(f'(x))3

d2 y
dx
2
dx2
Which can also be remembered as = Ans.
dy 2  dy 
3

 
 dx 

Concept Builders - 8
2
(i) If y = x ex then find y''.

(ii) Find y" at x = /4, if y = x tan x.

(iii) Prove that the function y= ex sin x satisfies the relationship y'' 2y' + 2y = 0.

Differentiation of Determinants

f(x) g(x) h(x)


If F(x) = (x) m(x) n(x) , where f, g, h , m, n, u, v, w are differentiable functions of x then
u(x) v(x) w(x)

f '(x) g '(x) h'(x) f(x) g(x) h(x) f(x) g(x) h(x)


F'(x) = (x) m(x) n(x) + '(x) m'(x) n'(x) + (x) m(x) n(x)
u(x) v(x) w(x) u(x) v(x) w(x) u'(x) v '(x) w '(x)

Note: Sometimes it is better to expand the determinant first and then differentiate.

Example 20:
x x2 x 3
If f(x) = 1 2x 3x2 , find f '(x).
0 2 6x
Solution:
x x2 x 3
Here, f(x) = 1 2x 3x2
0 2 6x
on differentiating, we get,

92 Methods of Differentiation
d d 2 d 3
(x) (x ) (x ) x x2 x3
dx dx dx 2 x x2 x3
d d d
 f'(x) = 1 2x 3x + (1) (2x) (3x2 ) + 1 2x 3x2
0 2 6x dx dx dx d d d
0 2 6x (0) (2) (6x)
dx dx dx
1 2x 3x2 x x2 x3 x x2 x 3
or f '(x) = 1 2x 3x + 0 2
2
6x + 1 2x 3x2
0 2 6x 0 2 6x 0 0 6

As we know if any two rows or columns are equal, then value of determinant is zero.
x x2 x 3
= 0 + 0 + 1 2x 3x2  f '(x) = 6(2x2 x2)
0 0 6

Therefore, f '(x) = 6x2 Ans.

Concept Builders - 9

2x x2 x3
ex x2
(i) If f (x) = , then find f '(1). (ii) If f (x) = x + 2x
2
1 3x + 1 , then find f '(1).
nx sin x
2x 1 − 3x2 5x

L 'Hopital's Rule
0 
(a) This rule is applicable for the indeterminate forms of the type , . If the function f(x)
0 
and g(x) are differentiable in certain neighborhood of the point 'a', except, may be, at
the point 'a' itself and g'(x)  0, and if
f(x) f '(x)
lim f(x) = lim g(x) = 0 or lim f(x) = lim g(x) = ,then lim = lim
x →a x →a x →a x →a x →a g(x) x → a g '(x)
f '(x)
provided the limit lim exists (L' Hopital's rule). The point 'a' may be either finite or
x →a g '(x)
improper (+ or ).
0 
(b) Indeterminate forms of the type 0. or   are reduced to forms of the type or
0 
by algebraic transformations.
(c) Indeterminate forms of the type 1, 0 or 00 are reduced to forms of the type 0 ×  by
taking logarithms or by the transformation [f(x)] (x) = e(x).nf(x).

Example 21:
Evaluate lim |x|sinx
x →0

Solution:
loge |x| 1/x
lim lim
lim |x|sin x = lim esin x loge |x| = ex →0 cos ecx = ex → 0 − cos ecx cot x (applying L'Hopital's rule)
x →0 x →0
2
2
sin x  sin x   x 
lim − lim −  .  2

= e ( ) ( ) = e0 = 1
− 1 . 0
ex →0 x cos x = e x →0  x   cos x 
Ans.

Methods of Differentiation 93
Example 22:
Solve lim+ logsin x sin 2x.
x →0

Solution:
Here lim+ logsin x sin2x
x →0

log sin2x  − 
= lim+  form 
x →0 log sin x  − 
1
.2cos2x
= lim+ sin2x {applying L'Hopital's rule}
x →0 1
.cos x
sinx
 (2 x) 
  cos 2 x
 sin(2 x)  cos2x
= lim+ = lim+ =1 Ans.
x →0  x  x →0 cos x
  cos x
 sin x 

Example 23:
1/n
 en 
Evaluate lim   .
 
n →

Solution:
1/n
 en 
Here, A = lim   (° form)
 
n →

1  en  nlog e − log   
 log A = lim log   = lim  form 
n → n 
 
n → n   
log e − 0
= lim {applying L'Hospital's rule}
n → 1
log A = 1
1/n
 en 
 A = e1 or lim   =e
 
n →

Concept Builders - 10

(i) Using L'Hopital's rule find

tanx − x ex − x − 1
(a) lim (b) lim
x →0
x3 x →0
x2

(ii) Using L'Hopital's rule verify that:

sinx − tanx 1 n(1 + x)


(a) lim = − (b) lim =1
x →0
x3 2 x →0 x

94 Methods of Differentiation
Miscellaneous Example

Example 24:

Find second order derivative of y = sinx with respect to z = ex.

Solution:

dy dy / dx cos x
= =
dx dz / dx ex

d2 y d  cos x  dx −ex sinx − cos xex 1


 =   . = . x
dz2 dx  ex  dz
( ) e
2
ex

d2 y (sinx+ cosx)
 =
dz2 e2x

Example 25:

x+ y f(x) + f(y)


Let a function f satisfies f   =  x, y  R and f '(0) = a, f(0) = b, then find f(x)
 2  2

hence find f"(x).

Solution:

x+ y f(x) + f(y)


f  =
 2  2

Diff. w.r.t. 'x'

x+ y 1 1  dy 
f '  . = f '(x)  x and y are independent to each other,  = 0
 2  2 2  dx 

x+ y
f '  = f '(x)
 2 

x
Let x = 0 and y = x f '   = f '(0) = a
2

 f '(x) = a

On integrating, we get f(x) = ax + b (f(0) = b)

 f "(x) = 0

Example 26:

1 x 1 x 1 x 1
Prove that sec2 + 4 sec2 2 + 6 sec2 3 + .... = cosec2x
22
2 2 2 2 2 x2

Methods of Differentiation 95
Solution:

x x x
Let cos . cos 2 . cos 3 ....... 
2 2 2

x x x x
= lim cos .cos 2 .cos 3 .........cos n
n → 2 2 2 2

sin x sin x x x x sin x


 lim = cos .cos 2 cos 3 ........... =
n → x x 2 2 2 x
2n sin n
2

 x  x   x 
 n  cos  + n  cos 2  + n  cos 3  +...... = n sin x nx
 2  2   2 

Differentiating w.r.t. x

1 x 1 x  1
 tan + 2 tan 2 + ......  = cot x
 2 2 2 2  x

Differentiating w.r.t. x again

1 x 1 x 1 x 1
sec2 + 4 sec2 2 + 6 sec2 3 + ........ = cosec2x Hence proved
22
2 2 2 2 2 x2

96 Methods of Differentiation
ANSWER KEY FOR CONCEPT BUILDER

1 1
1. (i) (a) (b)
x x2

2. (i) (a) 3x2 + 12x + 11 (b) 5e5x tan (x2 + 2) + 2xe5x sec2(x2 + 2)

3. (i) xx (lnx + 1) (ii) y(1 + 2x + 3x2 + 4x3)

1
4. (i) 1 (ii) (iii) 2(xlnx)(lnx)
t

cos(x− y) − 1  2x + ey 
5. (i) (ii) y'=  y , 1
cos(x− y) + 1  xe + 1 

6. (i) (a) 6 (b) 3 (c) 6

7. (i) D

8. (i) y '' = 4y + 2xy' (ii) +4

9. (i) e( sin 1 + cos 1) 1 (ii) 9

1 1
10. (i) (a) (b)
3 2

Methods of Differentiation 97
Objective Exercise - I

1 1 1 dy
1. If y = + + , then is equal to-
1+ x −
+x  −
1+ x −
+x −
1+ x −
+x −
dx
(A) 0 (B) 1
(C) ( +  + )x +  +  1
(D) 

 3 
2. If f(x) = |cosx|, then f '   is equal to -
 4 
1 1
(A) (B) (C) 1 (D) 1
2 2

d x
3. (e sin 3 x) equals-
dx
(A) ex sin( 3 x + /3) (B) 2ex sin( 3 x + /3)
1 x 1 x
(C) e sin( 3 x + /3) (D) e sin( 3 x /3)
2 2

d
4. (n sin x ) is equal to-
dx
tan x cot x cot x cot x
(A) (B) (C) (D)
2 x 3 x 2x 2 x

1− x dy
5. If y = , then equals
1+ x dx
y y y y
(A) (B) (C) (D)
1 − x2 x −1
2
1 + x2 y −1
2

 x + (a2 + x2 )  dy
6. If y = n   , then the value of is-
 a  dx

1
(A) a2 − x2 (B) a a2 + x2 (C) (D) x a2 + x2
a +x
2 2

dy
7. If x = a(t sint), y = a(1 + cost), then equals-
dx
t t t t
(A) tan (B) cot (C) cot (D) tan
2 2 2 2

 1 
8. The differential coefficient of sec 1  2  w.r.t. 1 − x2 is-
 2x − 1 
(A) 1/x2 (B) 2/x3 (C) x/2 (D) 2/x

98 Methods of Differentiation
dy
9. If x3 y3 + 3xy2 3x2y + 1 = 0, then at (0, 1) equals-
dx
(A) 1 (B) 1 (C) 2 (D) 0

d  −1  1 − cos  

10.  tan    equals, if < <  -
d   sin   
(A) 1/2 (B) 1 (C) sec (D) cosec

d 1+ x
11. cot 1   is equal to, if x > 1
dx 1− x
1 1 1 −1
(A) (B) (C) − (D)
1 + x2 1 − x2 1 + x2 1 − x2

dy
12. If y = tan 1(cot x) + cot 1(tan x), then is equal to-
dx
(A) 1 (B) 0 (C) 1 (D) 2

d   x − x 
13.  tan−1    equals- (x  0)
dx   1 + x3/2  
  
1 1 1 1
(A) − (B) +
2 x(1 + x) 1 + x2 2 x(1 + x) 1 + x2
1 1 1 1
(C) − (D) +
1 + x 1 + x2 1 + x 1 + x2

1
14. If g is the inverse of f and f '(x) = then g'(x) is equal to-
1 + x3
−1 1 1
(A) 1 + [g(x)]3 (B) (C) (D)
2(1 + x2 ) 2(1 + x2 ) 1 +  g(x)
3

15. If x2 + y2 = 1, then-
(A) yy" 2(y')2 + 1 = 0 (B) yy" + (y')2 + 1 = 0
(C) yy" + (y')2 1=0 (D) yy" + 2(y')2 + 1 = 0

16. Let f be a function defined for all x  R. If f is differentiable and f(x3) = x5 for all xR
(x  0), then the value of f '(27) is-
(A) 15 (B) 45 (C) 0 (D) 35

dy dx
17. Let y = x3 8x + 7 and x = f(t). If = 2 and x = 3 at t = 0, then at t = 0 is given by:
dt dt
19 2
(A) 1 (B) (C) (D) None of these
2 19

Methods of Differentiation 99
dy 5
18. If y = cos 1(cos x), then at x = is equal to:
dx 4
1 1
(A) 1 (B) 1 (C) (D) −
2 2

19. If y = x x2, then the derivative of y2 w.r.t. x2 is:


(A) 2x2 + 3x 1 (B) 2x2 3x + 1 (C) 2x2 + 3x + 1 (D) none of these

cos x x 1
f '(x)
20. Let f(x) = 2 sin x x2 2x . Then lim =
tan x x 1
x →0 x

(A) 2 (B) 2 (C) 1 (D) 1

21. Let ef(x) = lnx. If g(x) is the inverse function of f(x) then g'(x) equal to:
x
(A) ex (B) ex + x (C) ex + e (D) ex + ln x

 1 dy
22. If 8f(x) + 6f   = x + 5 and y = x2 f(x), then at x = 1 is equal to:
x dx
1 1
(A) 0 (B) (C) − (D) none of these
14 14

23. Let f(x) be a polynomial in x. Then the second derivative of f(ex) w.r.t. x is:
(A) f "(ex)ex + f '(ex) (B) f "(ex)e2x + f '(ex)e2x
(C) f "(ex)e2x (D) f "(ex)e2x + f '(ex)ex

x
 1
24. Let y = xx + 1 +  1 +  then y ' (1) equals
 x
(A) (ln 2) + 1 (B) (2 ln 2) + 1 (C) (ln 2) 1 (D) (2 ln 2) 1

d2 y
25. If x = t3 + t + 5 and y = sin t, then =
dx2
(3 t2 + 1) sint + 6 tcost (3 t2 + 1) sint + 6 tcost
(A) − (B)
(3 t2 + 1)3 (3 t2 + 1)2
(3 t2 + 1) sint + 6 tcost cos t
(C) − (D)
(3 t2 + 1)2 3t2 + 1

26. Suppose that f (0) = 0 and f '(0) = 2, and let g (x) = f ( x + f (f (x))). The value of g ' (0) is equal
to:
(A) 0 (B) 1 (C) 6 (D) 8

100 Methods of Differentiation


f(4) − f(x2 )
27. If f is differentiable in (0, 6) and f '(4) = 5, then L imit =
x →2 2−x
(A) 5 (B) 5/4 (C) 10 (D) 20

d2 x
28. If y = x + ex then is -
dy 2
ex ex −1
(A) ex (B) − (C) (D)
(1 + ex )3 (1 + ex )2 (1 + ex )3

d2 y dy
29. If y = (A + Bx) emx + (m - 1) 2
ex then 2m + m2y is equal to -
dx2 dx
(A) ex (B) emx (C) e mx
(D) e(1 m) x

(x+ h) f(x) − 2hf(h)


30. Let f(x) be differentiable at x = h, then L im is equal to -
x →h x −h
(A) f(h) + 2hf '(h) (B) 2f(h) + hf '(h)
(C) hf(h) + 2f '(h) (D) hf(h) 2f '(h)

dy
31. If xm.yn = (x + y)m+n, then is
dx
x+y x y
(A) (B) xy (C) (D)
xy y x

32. Let y be an implicit function of x defined by x2x 2xx cot y 1 = 0. then y'(1) equals:
(A) log 2 (B) log2 (C) 1 (D) 1

dy
33. If y = sec(tan 1x), then at x = 1 is equal to:
dx
1 1
(A) (B) (C) 1 (D) 2
2 2

Methods of Differentiation 101


Objective Exercise - II

Single Correct Type Questions

dy
1. If 2x + 2y = 2x+y, then is equal to -
dx
2x + 2y 2x + 2y  2y − 1  2x + y − 2x
(A) (B) (C) 2x y  x 
(D)
2x − 2y 1 + 2x + y 1−2  2y

x b b
x b
2. If 1 = a x b and 2 = are given, then
a x
a a x

d d
(A) 1 = 3(2)2 (B) 1 = 32 (C) 1= 3(2)2 (D) 1 = 3(2)3/2
dx dx

3. Consider f(x) be a polynomial function of second degree. If f(1) = f( 1) and a, b, c are in A.P.,
then f '(a), f '(b) and f '(c) are in -
(A) G.P. (B) H.P. (C) A.G.P. (D) A.P.

4. Suppose the function f (x) f (2x) has the derivative 5 at x = 1 and derivative 7 at x = 2. The
derivative of the function f (x) f (4x) at x = 1, has the value equal to -
(A) 19 (B) 9 (C) 17 (D) 14

x tan−1 x + sec−1 (1 / x), x  (−1, 1) − {0}


5. If f(x) =  , then f '(0) is -
  / 2, if x = 0

(A) equal to 1 (B) equal to 0 (C) equal to 1 (D) non-existent

6. If f is twice differentiable such that


f "(x) = f(x), f '(x) = g(x)
h'(x) = [f(x)]2 + [g(x)]2 and
h (0) = 2, h(1) = 4
then the equation y = h(x) represents:
(A) a curve of degree 2 (B) a curve passing through the origin
(C) a straight line with slope 2 (D) a straight line with y intercept equal to 2

3
d2 x  dy  d2 y
7. If 2   + = K then the value of K is equal to -
dy  dx  dx2
(A) 1 (B) 1
(C) 2 (D) 0

102 Methods of Differentiation


 1 
8. Let f (x) = x + sin x. Suppose g denotes the inverse function of f. The value of g'  +  has
4 2
value equal to -
2+1
(A) 2 1 (B) (C) 2 2 (D) 2+1
2

d2 x
9. equals:
dy 2
−2 −3
 d2 y   dy   d2 y   dy 
(A)  2    (B) −  2   
 dx   dx   dx   dx 
−1 −1 −3
 d2 y   d2 y   dy 
(C)  2  (D) −  2   
 dx   dx   dx 

One or More than One Correct Type Questions

4
10. If f(x) = (2x 3)5 + x + cos x and g is the inverse function of f, then -
3
7 3 7
(A) g'(2) = (B) g'(2) = (C) g''(2) = (D) g''(2) = 0
3 7 3

11. If f(x) = x.|x|, then its derivative is -


(A) 2x (B) 2x (C) 2|x| (D) 2xsgnx

dy
12. If y = tan x tan 2x tan 3x, (sin12x  0) then has the value equal to -
dx
(A) 3 sec2 3x tan x tan 2x + sec2 x tan 2x tan 3x + 2 sec2 2x tan 3x tan x
(B) 2y (cosec 2x + 2 cosec 4x + 3 cosec 6x)
(C) 3 sec2 3x 2 sec2 2x sec2 x
(D) sec2 x + 2 sec2 2x + 3 sec2 3x

x  − x
13. L im is equal to
x →
x x − 
e 
(A) log e   (B) log e  
 e
(C) tan(cot 1(n) cot 1(1)) (D) tan(tan 1(1) tan 1(n))

14. Let P(x) be the polynomial x3 + ax2 + bx + c, where a, b, c  R. If P( 3) = P(2) = 0 and


P'( 3) < 0, which of the following is a possible value of 'c' ?
(A) 27 (B) 18 (C) 6 (D) 3

Methods of Differentiation 103


15. Two functions f and g have first and second derivatives at x = 0 and satisfy the relations,
2
f (0) = , f '(0) = 2g'(0) = 4g(0), g"(0) = 5 f"(0) = 6f(0) = 3 then
g(0)
f(x) 15
(A) if h(x) = then h' (0) = (B) if k(x) = f(x). g(x) sin x then k'(0) = 2
g(x) 4
g '(x) 1
(C) L imit = (D) none
x →0 f '(x) 2

nx) n( nx ) dy
16. If y = x( , then is equal to
dx
y y
(A) (nxn (x 1)
+ 2nx n (nx)) (B) (nx)n (n x) (2 n (nx) + 1)
x x
y y ny
(C) ((nx)2 + 2 n (nx)) (D) (2 n (nx) + 1)
x nx x nx

Match the List Type Question


17. List-I contains function defined on R and List-II contains their properties. Match them
List-I List II
n
  
 1 + tan 
(A) L im  2n  equal (P) e
n→
 1 + sin  
 
 3n 
1
(B) Lim+ 1
equals (Q) e2
x →0
(1 + cosecx) n(sinx)

1/x
2  2/
(C) L im  cos−1 x  equals (R) e
x →0
 
(S) e/6

104 Methods of Differentiation


Subjective Exercise - I

1. Let f, g and h are differentiable functions. If f(0) = 1 ; g(0) = 2 ; h(0) = 3 and the derivatives of
their pair wise products at x = 0 are (f g)' (0) = 6 ; (g h)' (0) = 4 and (h f)' (0) = 5 then compute
the value of (fgh)'(0).

x2 1
2. If y = + x x2 + 1 + n x + x2 + 1 prove that 2y = xy'+ny'. Where y' denotes the derivative
2 2
of y.

2
 dy 
3. If x = cosec  sin  ; y = cosecn sinn, then show that (x2 + 4)   n2(y2 + 4) = 0.
 dx 

4. If a curve is represented parametrically by the equations


 7     3 
x = sin  t +  + sin  t −  + sin  t + ,
 12   12   12 
 7     3 
y = cos  t +  + cos  t −  + cos  t + 
 12   12   12 
d x y 
then find the value of  −  at t = .
dt  y x  8


1 + nt 3 + 2 nt dy  dy 
5. If x = and y = . Show that y = 2x   + 1.
t2 t dx  dx 

 g(x), x0
6. 
Let g(x) be a polynomial, of degree one and f(x) be defined by f(x) =  1 + x 
1/x

, x0
 2 + x 
Find the continuous function f(x) satisfying f ' (1) = f ( 1).

1 dy 1
7. If y = x + , prove that =
1 dx x
x+ 2−
1 1
x+ x+
x + ......... 1
x+
x + .........

1
8. Let f(x) = x + Compute the value of f(100). f ' (100).
1
2x +
1
2x + .....
2x

9. Suppose f (x) = tan(sin 1 (2x))


(a) Find the domain and range of f.
(b) Express f(x) as an algebraic function of x.
(c) Find f ' (1/4)

Methods of Differentiation 105


10. (a) Let f(x) = x2 4x 3, x > 2 and let g be the inverse of f. Find the value of g' where f(x) = 2.
(b) Let f: R→R be defined as f(x) = x3 + 3x2 + 6x 5 + 4e2x and g(x) = f 1(x), then find g'( 1).
1
(c) Suppose f 1
is the inverse function of a differentiable function f and let G(x) = .
f (x)
−1

1
If f(3) = 2 and f'(3) = , find G'(2).
9

1 1 1 1
11. If y = tan 1
+ tan 1 2 + tan 1 2 + tan 1 2 +...... to n terms.
x2 + x + 1 x + 3x + 3 x + 5x + 7 x + 7x + 13
Find dy/dx, expressing your answer in 2 terms.

u 1  1   1  dy
12. If y = tan 1
and x = sec 1
, u   0,  , 1  prove that 2 + 1 = 0.
1−u 2 2u − 1
2
 2  2  dx

1 + sinx + 1 − sinx dy    
13. If y = cot 1
, find if x   0,    ,   .
1 + sinx − 1 − sinx dx  2 2 

(x− a)4 (x − a)3 1 (x− a)4 (x − a)2 1


14. If f(x) = (x− b)4 (x − b)3 1 , then f'(x) =  (x− b)4 (x − b)2 1 . Find the value of .
(x− c)4 (x − c)3 1 (x− c)4 (x − c)2 1

15. (a) If y = y(x) and it follows the relation exy + y cos x = 2, then find (i) y'(0) and (ii) y"(0).
(b) A twice differentiable function f(x) is defined for all real numbers and satisfies the
following conditions
f(0) = 2; f '(0) = 5 and f "(0) = 3.
The function g(x) is defined by g(x) = eax + f (x)  x  R, where 'a' is any constant.
If g'(0) + g"(0) = 0. Find the value(s) of 'a'.

16. If x = 2cost cos2t and y = 2sint sin2t, find the value of (d2y/dx2) when t = (/2).

d2 y
17. Find the value of the expression y3 on the ellipse 3x2 + 4y2 = 12.
dx2

cos(x+ x2 ) sin(x + x2 ) − cos(x + x2 )


18. If f(x) = sin(x− x2 ) cos(x− x2 ) sin(x − x2 ) then find f ' (x).
sin2x 0 sin2x2

19. Let P(x) be a polynomial of degree 4 such that P(1) = P(3) = P(5) = P'(7) = 0. If the real number
x  1, 3, 5 is such that P(x) = 0 can be expressed as x = p/q where 'p' and 'q' are relatively prime,
then (p + q) equals.

106 Methods of Differentiation


 1 1 − x2 
20. Lim  − 
 x sin x x2 
x →0 −1

 1 1 
21. L im  2 − 
x →0
 x sin 2
x

1 + sinx − cos x + n(1 − x)


22. L im
x →0
x. tan2 x

x6000 − (sinx)6000
23. L im
x →0 x2 .(sinx)6000

1 1
− d2 y dy
24. If 2x = y 5 + y 5
then (x2 1) +x = ky, then find the value of 'k'.
dx 2
dx

25. Let y = x sin kx. Find the number of possible value(s) of k for which the differential equation
d2 y
+ y = 2k coskx holds true for all x  R.
dx2

Methods of Differentiation 107


Subjective Exercise - II

1. Given a real valued function f (x) as follows:


x2 + 2cos x − 2 1 sinx − n(excosx)
f(x) = for x < 0 ; f(0) = and f(x) = for x > 0. Test the
x 4
12 6x2
continuity and differentiability of f (x) at x = 0.

2. If x1, x1, x2, x3, x4 be 5 zero's of the polynomial P(x) = x5 + x + ,


where xi  xj i and j = 1, 2, 3, 4.
If the value of Q(x) = (x1 x2)(x1 x3)(x1 x4) is equal to (x1)3.

3. The function f: R → R satisfies f(x2) · f " (x) = f ' (x) · f ' (x2) for all real x. Given that f(1) = 1 and
f"'(1) = 8, compute the value of f'(1) + f"(1).

4. A polynomial function f(x) such that f(2x) = f '(x) f "(x). Find f(3)

sin x
5. Let f(x) = if x  0 and f(0) = 1. Define the function f ' (x) for all x and find f "(0) if it exist.
x

6. Suppose f and g are two functions such that f, g: R →R,


f(x) = ln(1+ 1 + x2 ) and g(x) = ln (x + 1 + x2 )
  1 '
then find the value of x eg(x)  f    + g ' (x) at x = 1.
  x 

f(x+ y) − f(x) f(y) − a


7. Let = + xy for all real x and y. If f(x) is differentiable and f ' (0) exists for all
2 2
real permissible values of 'a' and is equal to 5a − 1 − a2 . Prove that f(x) is positive for all real x.

x  1− x  dy
8. If y = tan 1
+ sin  2 tan−1  , then find for x  ( 1, 1).
1 + 1 − x2  
1+ x  dx

9. If f: R →R is a function such that f(x) = x3 + x2f ' (1) + xf "(2) + f "'(3) for all x  R, then prove that
f(2) = f(1) f(0).

x
10. If the function f(x) = x3 + e 2 and g(x) = f 1 (x), then the value of g'(1) is

  sin      d
11. Let f() = sin  tan−1    , where    . then the value of (f()) is
 4 4 d(tan )
  cos2  

108 Methods of Differentiation


JEE-Main (Previous Year Questions)

1. Let f: ( 1, 1) → R be a differentiable function with f(0) = 1 and f'(0) = 1.


Let g(x) = [f(2f(x) + 2)] . Then g'(0):
2
[AIEEE-2010]
(1) 4 (2) 4 (3) 0 (4) 2

1
2. If g is the inverse of a function f and f'(x) = , then g'(x) is equal to: [JEE(Main)-2014]
1 + x5
1
(1) 1 + x5 (2) 5x4 (3) (4) 1+{g(x)}5
1 + {g(x)}5

3. If for x  R, f(x) = |log2 sinx| and g(x) = f(f(x)), then: [JEE(Main)-2016]


(1) g is differentiable at x = 0 and g'(0) = sin(log2)
(2) g is not differentiable at x = 0
(3) g'(0) = cos(log2)
(4) g'(0) = cos(log2)

 1  6x x 
4. If for x   0,  , the derivative of tan 1   is x .g(x) then g(x) equals [JEE(Main)-2017]
 4  1 − 9x 3 
 

3 9 3x x 3x
(1) (2) (3) (4)
1 + 9x3 1 + 9x3 1 − 9x 3 1 − 9x3

d2 y 
5. If x = 3 tant and y = 3 sect, then the value of at t = , is: [JEE(Main)-2019]
dx 2
4
3 1 1 1
(1) (2) (3) (4)
2 2 6 2 3 2 6

6. Let f: R → R be a function such that f(x) = x3 + x2f '(1) + xf "(2) + f "'(3), xR. Then f(2) equals:
[JEE(Main)-2019]
(1) 8 (2) 30 (3) 4 (4) 2

dy
7. If xloge (loge x) x2 + y2 = 4 (y > 0), then at x = e is equal to: [JEE(Main)-2019]
dx

(1)
e
(2)
(2e − 1)
4 + e2 2 4 + e2

(3)
( 1 + 2e) (4)
( 1 + 2e)
2 4+e 2
4 + e2

Methods of Differentiation 109


dx
8. For x > 1, if (2x)2y = 4e2x 2y
, then (1 + loge 2x)2 is equal to: [JEE(Main)-2019]
dy

xloge 2x − loge 2
(1) (2) x loge 2x
x
xloge 2x + loge 2
(3) loge 2x (4)
x

2
  3 cos x + sin x     dy
9. If 2y =  cot    , x   0,  , then is equal to: [JEE(Main)-2019]
    2  dx
  
   
(1) (2) (3) (4)
6 6 3 3

1
dy  y 3
10. Let xk + yk = ak (a, k > 0) and +   = 0 , then k is: [JEE(Main)-2020]
dx x
2 1 4 3
(1) (2) (3) (4)
3 3 3 2

 tan  + cot   1  3  dy 5
11. If y() = 2 + ,    ,   then at  = is: [JEE(Main)-2020]
 1 + tan   sin   4 d 6
2 2

4 1
(1) 4 (2) 4 (3) (4)
3 4

12. fog is the identity function. If for some


a, b : [JEE(Main)-2020]
1 2
(1) 1 (2) (3) 5 (4)
5 5

  
13. If y2 + loge(cos2x) = y, x   − ,  , then: [JEE(Main)-2020]
 2 2
(1) |y'(0)| + |y"(0)| = 1 (2) |y'(0)| + |y"(0)| = 3
(3) y"(0) = 0 (4) |y"(0)| = 2

 1 + x2 − 1   2x 1 − x2  1
14. The derivative of tan 1   with respect to tan 1   at x = is:
 x   1 − 2x2  2
   
[JEE(Main)-2020]

3 3 2 3 2 3
(1) (2) (3) (4)
10 12 3 5

110 Methods of Differentiation


xf(a) − af(x)
15. lim
x →a x−a
equals: [JEE(Main)-2021]
(1) 2a+ 4 (2) 2a 4 (3) 4 2a (4) a + 4

16. Let f : S → S where S = (0, ) be a twice differentiable function such that f(x+1) = xf(x). If
g : S → R be defined as g (x)= loge
[JEE(Main)-2021]
205 197 187
(1) (2) (3) (4) 1
144 144 144

  1 − 22x  b
17. If f(x) = sin  cos−1    and its first derivative with respect to x is − loge 2 when x = 1,
  1 + 22x a
  
where a and b are integers, then the minimum value of | a 2 − b2 | is: [JEE(Main)-2021]

 3
18. If y = tan−1 (sec x3 − tan x3 ),  x3  , then: [JEE(Main)-2021]
2 2
3
(1) xy "+ 2y ' = 0 (2) x2 y "− 6y + =0
2
(3) x2 y"− 6y + 3 = 0 (4) xy "− 4y ' = 0

19. Let f: R → R be defined as f(x) = x3 + x 5. If g(x) is a function such that f(g(x)) = x, x  R , then
[JEE(Main)-2022]
1 3 43 91
(1) (2) (3) (4)
49 49 49 49

f '(4)
20. Let f : R → R satisfy f(x+y) = 2xf(y) + 4y f(x), x , y  R . If f (2) = 3, then 14. is equal to____.
f '(2)
[JEE(Main)-2022]
5
y x
21. If cos−1   = loge   ,| y | 2 : [JEE(Main)-2022]
2 5
(1) x2 y''+ xy'− 25y = 0 (2) x2 y''− xy'− 25y = 0

(3) x2 y''− xy'+ 25y = 0 (4) x2 y''+ xy'+ 25y = 0

a −1 0
22. Let f(x) = ax a −1 ,a  R . then the sum of which the squares of all the values of a for
ax2 ax a
2f'(10) − f'(5) + 100 = 0 is: [JEE(Main)-2022]
(1) 117 (2) 106 (3) 125 (4) 136

Methods of Differentiation 111


x d2 x
23. If y(x) = (xx ), x  0 then + 20 at x = 1 is equal to. [JEE(Main)-2022]
dy 2

d 
24 The value of loge2 (logcos x cosecx) at x = is: [JEE(Main)-2022]
dx 4
(1) −2 2 (2) 2 2 (3) 4 (4) 4

25. For the curve C : (x2 + y2 − 3) + (x2 − y2 − 1)5 = 0 y3  ),  > 0,


on C, is equal to _______. [JEE(Main)-2022]

2
 dy 
1+  
26.
 
Let x(t) = 2 2 cos t sin2t and y(t) = 2 2 sin t sin2t, t   0,  . Then  dx  at t =  is
 2 d2 y 4
dx 2

equal to: [JEE(Main)-2022]


−2 2 2 1 −2
(1) (2) (3) (4)
3 3 3 3

27. If y(x) = xx , x  0, then y ''(2) − 2y '(2) is equal to: [JEE(Main)-2023]


(1) 8 loge2 2 (2) 4 (loge2) + 2
2
(3) 4 (loge2) 2
2 (4) 4 loge2 + 2

    3 
28.

let y = f ( x ) = sin3   cos 
 3  3 2 (
−4x3 + 5x2 + 1 ) 2    .Then, at x = 1
  
[JEE(Main)-2023]
   
(1) 2y '+ 32 y = 0 (2) 2y '+ 32 y = 0 (3) 2y '− 32 y = 0 (4) y '+ 32 y = 0

29. Let y(x) = (1 + x)(1 + x2)(1 + x4)(1 + x8)(1 + x16), Then y' y" at x = −1 is equal to: [JEE(Main)-2023]
(1) 976 (2) 464 (3) 496 (4) 944

sin x + cos x − 2   7   7 


30. Let f(x) = , x  [0, ] −   , then f   f    is equal to:
sin x − cos x 4  12   12 
[JEE(Main)-2023]
2 −2 −1 2
(1) (2) (3) (4)
9 3 3 3 3 3

112 Methods of Differentiation


JEE-Advanced (Previous Year Questions)

1. For x > 0, L im ((sin x)1/x+ (1/ x)sin x) is: [JEE(Advanced)-2006]


x →0

(A) 0 (B) 1
(C) 1 (D) 2

d2 x
2. equals: [JEE(Advanced)-2007]
dy 2
−1 −1 −3
 d2 y   d2 y   dy 
(A)  2  (B)  2  
 dx   dx   dx 
−2 −3
 d2 y   dy   d2 y   dy 
(C)  2    (D)  2  
 dx   dx   dx   dx 

3. Let f: R→R, g: R→R and h: R→R be differentiable functions such that f(x) = x3 + 3x + 2,
g(f(x)) = x and h(g(g(x))) = x for all x R. Then- [JEE(Advanced)-2016]
1
(A) g'(2) = (B) h'(1) = 666
15
(C) h(0) = 16 (D) h(g(3)) = 36

Methods of Differentiation 113


5 Rate Measure, Tangent and Normal

Rate Measurement
dy
Whenever one quantity y varies with another quantity x, satisfying some rule y = f(x), then
dx
dy 
(or f'(x)) represents the rate of change of y with respect to x and  (or f '(a)) represents the rate
dx  x =a

of change of y with respect to x at x = a.

Example 1:
The volume of a cube is increasing at a rate of 9cm3/s. How fast is the surface area increasing
when the length of an edge is 10cm?
Solution:
Let x be the length of side, V be the volume and S be the surface area of the cube.
Then V = x3and S = 6x2, where x is a function of time t.
dV d 3 dx dx 3
= 9cm3 /s = (x ) = 3x2  = 2
dt dt dt dt x
ds d  3 36
= (6x2) = 12x  2  =
dt dt x
  x

dS 
 = 3.6 cm2/s.
dt  x = 10cm

Example 2:
x and y are the sides of two squares such that y = x x2. Find the rate of change of the area
of the second square with respect to the first square.
Solution:
Given x and y are sides of two squares. Thus, the area of two squares are x2 and y2 We have to
dy
2y
d(y 2 ) dx = y . dy
obtain = ........ (i)
d(x2 ) 2x x dx

dy
where the given curve is, y = x x2  =1 2x ........ (ii)
dx
d(y 2 ) y
Thus, = (1 2x) [from (i) and (ii)]
d(x )
2
x

d(y 2 ) (x − x2 )(1 − 2 x) d(y 2 )


or =  = (2x2 3x + 1)
d(x )
2
x d(x2 )

The rate of change of the area of second square with respect to first square is (2x2 3x + 1)

Rate Measure, Tangent and Normal 115


Concept Builders - 1

(i) What is the rate of change of the area of a circle with respect to its radius r at r = 6cm.
(ii) A stone is dropped into a quiet lake and waves move in circles at the speed of 5cm/s. At the
instant when the radius of the circular wave is 8cm, how fast is the enclosed area increasing?
(iii) If the area of circle measures at a uniform rate, show that the rate of increase of the perimeter
varies inversely at its radius.
(iv) If the side of an equilateral triangle increases at the rate of 3 cm/sec and area increase at the
rate 12 cm /sec then the side of the equilateral triangle is ______.
2

2
(v) An aeroplane is flying horizontally at a height of km with a velocity of 15 km/hr. Find the rate
3
at which it is receding from a fixed point on the ground which it passed over 2 minutes ago.

Approximation Using Differentials


In order to calculate the approximate value of a function, differentials may be used where the
differential of a function is equal to its derivative multiplied by the differential of the independent
variable. In general, dy = f '(x)dx or df(x) = f '(x)dx

Note: (i)  x and differential dx are equal but this is


y  dy.
 Approximate value of y when increment  x is given to independent variable x in
y = f(x) is
dy
y + y = f(x + x) = f(x) + .x
dx
dy
(ii) The relation dy = f '(x) dx can be written as = f '(x) ; thus the quotient of the
dx
differentials of 'y' and 'x' is equal to the derivative of 'y' w.r.t. 'x'.

Example 3:
Find the approximate value of square root of 25.2.
Solution:
Let f(x) = x
x
Now, f(x + x) f(x) = f '(x) . x =
2 x
we may write, 25.2 = 25 + 0.2
Taking x = 25 and x = 0.2, we have
0.2
f(25.2) f(25) =
2 25
0.2
or f(25.2) 25 = = 0.02  f(25.2) = 5.02
10
or (25.2) = 5.02

116 Rate Measure, Tangent and Normal


Concept Builders - 2

(i) Find the approximate value of (0.009)1/3


(ii) Compute the square root of 101
(iii) Find the approximate change in volume V of a cube of side 5m caused by increasing its side
length by 2%.
(iv) The approximate value of tan 46° is (take  = 22/7)
(A) 3 (B) 1.035 (C) 1.033 (D) 1.135

Tangent to The Curve at a Point


dy
Geometrical Meaning of
dx
As Q → P, h → 0 and slope of chord PQ tends to slope of tangent at P (see figure below).
f(x + h) − f(x)
Slope of chord PQ =
h
f(x + h) − f(x)
lim slope of chord PQ = lim
Q →P h→0 h
dy
 slope of tangent at P = f(x) =
dx

Tangent
Let y = f(x) be function with graph as shown in figure. Consider secant PQ. If Q tends to P along
the curve passing through the points Q1, Q2, .....I.e. Q → P, secant PQ will become tangent at P.

tangent
P

Normal
Let y = f(x) be function with graph as shown in figure. A line through P perpendicular to tangent
is called normal at P.

Rate Measure, Tangent and Normal 117


Equation of tangent
dy 
 = f(x1) denotes the slope of tangent at point (x1, y1) on the curve y = f(x). Hence the
dx (x , y )
1 1

equation of tangent at (x1, y1) is given by


(y y1) = f(x1) (x x1) ; when, f(x1) is real.

Equation of normal
Also, since normal is a line perpendicular to tangent at (x1 , y1) so its equation is given by
1
(y y1) = (x x1), when f(x1) is non zero real.
f (x1 )
If f(x1) = 0, then tangent is the line y = y1 and normal is the line x = x1.
f(x1 + h) − f(x1 )
If lim =  or , then x = x1 is tangent (VERTICAL TANGENT) and y = y1 is normal.
h→0 h

Example 4:
Find equation of tangent to y = ex at x = 0. Hence draw graph
Solution:
At x = 0  y = e0 = 1
dy dy
= ex  =1
dx dx x =0

Hence equation of tangent is


1 (x 0) = (y 1)
 y=x+1

Example 5:
If y = 4x 5 is a tangent to the curve y2 = px3 + q at (2, 3) then
(A) p = 2, q = 7 (B) p = 2, q = 7 (C) p = 2, q = 7 (D) p = 2, q = 7
Solution:
dy
= 4 and 9 = 8p + q
dx
dy
2y = 3px2
dx
dy dy
6 = 3p(4)  = 2p = 4  p = 2 and q = 7
dx dx

118 Rate Measure, Tangent and Normal


Example 6:
1
Find the equation of all straight lines which are tangent to curve y = and which are parallel
x−1
to the line x + y = 0.
Solution:
Suppose the tangent is at (x1, y1) and it has slope 1.

dy
 = 1.
dx (x 1 , y 1 )

1
 = 1.
(x1 − 1)2
 x1 = 0 or 2

y1 = 1 or 1
Hence tangent at (0, 1) and (2, 1) are the required lines (see figure) with equations
1(x 0) = (y + 1) and 1 (x 2) = (y 1)
 x+y+1=0 and y+x=3

Example 7:
Find equation of normal to the curve y = |x2 | x | | at x = 2.
Solution:
In the neighborhood of x = 2, y = x2 + x.
Hence the point of contact is ( 2, 2)
dy dy
= 2x + 1  = 3.
dx dx x =−2

1
So the slope of normal at ( 2, 2) is .
3
Hence equation of normal is
1
(x + 2) = y 2  3y = x + 8
3

Example 8:
Find the equation of the tangent to the curve y = (x3 1)(x 2)at the points where the curve
cuts the x-axis.
Solution:
The equation of the curve is y = (x3 1)(x 2) .......... (i)
It cuts x-axis at y = 0. So, putting y = 0 in (i), we get (x 3
1)(x 2) = 0
 (x 1) (x 2) (x2 + x + 1) = 0  x 1 = 0, x 2=0 [ x2 + x + 1  0]
 x = 1, 2.

Rate Measure, Tangent and Normal 119


Thus, the points of intersection of curve (i) with x-axis are (1, 0) and (2, 0). Now,
dy  dy   dy 
y = (x3 1) (x 2)  = 3x2 (x 2) + (x3 1)    = 3 and   =7
dx  dx (1,0)  dx (2,0)

The equations of the tangents at (1, 0) and (2, 0) are respectively


y 0 = 3(x 1) and y 0 = 7 (x 2)  y + 3x 3 = 0 and 7x y 14 = 0 Ans.

Example 9:

The equation of the normal to the curve y = x + sin x cos x at x = is -
2
(A) x = 2 (B) x =  (C) x +  = 0 (D) 2x = 
Solution:
    
x= y= + 0 = , so the given point =  , 
2 2 2 2 2

dy  dy 
Now from the given equation = 1 + cos2 x sin2x    =1+0 1=0
dx  dx   ,  
2 2

 
 The curve has vertical normal at  ,  .
2 2

the equation to this normal is x =
2

x = 0  2x =  Ans. (D)
2

Note:
(i) The point P (x1, y1) will satisfy the equation of the curve & the equation of tangent and
normal line.
(ii) If the tangent at any point P on the curve is parallel to the axis of x then
dy
= 0 at the point P.
dx
dy
(iii) If the tangent at any point on the curve is parallel to the axis of y, then not defined
dx
dx
or = 0.
dy
(iv) If the tangent at any point on the curve is equally inclined to both the axes then
dy
= 1 .
dx

120 Rate Measure, Tangent and Normal


Myths About Tangent
(a) Myth
A line meeting the curve only at one point is a tangent to the curve.
Explanation
A line meeting the curve in one point is not necessarily tangent to it.

Here L is not tangent to C


(b) Myth
A line meeting the curve at more than one point is not a tangent to the curve.
Explanation
A line may meet the curve at several points and may still be tangent to it at some
point

Here L is tangent to C at P, and cutting it again at Q.


(c) Myth
Tangent at a point to the curve can not cross it at the same point.
Explanation
A line may be tangent to the curve and also cross it.

Here X-axis is tangent to y = x3 at origin.

Concept Builders - 3

(i) Find the distance between the point (1, 1) and the tangent to the curve y = e2x + x2 drawn at the
point where the curve cuts y-axis
(ii) Find the equation of a line passing through ( 2, 3) and parallel to tangent at origin for the circle
x2 + y2 + x y = 0.

(iii) Find the slope of the normal to the curve x = 1 a sin , y = b cos2  at  = .
2

Rate Measure, Tangent and Normal 121


(iv) Find the equation of the tangent and normal to the given curves at the given points.
(a) y = x4 6x3 + 13x2 10x + 5 at (1, 3)
3
x
(b) y2 = at (2, 2).
4−x
(v) Prove that area of the triangle formed by any tangent to the curve xy = c 2 and coordinate axes
is constant.

Angle of Intersection Between Two Curves


Angle of intersection between two curves is defined as the angle between the two tangents drawn to
the two curves at their point of intersection.

Orthogonal Curves
If the angle between two curves at each point of intersection is 90° then they are called orthogonal
curves.
For example, the curves x2 + y2 = r2 & y = mx are orthogonal curves.

Example 10:
The angle of intersection between the curve x2 = 32y and y2 = 4x at point (16, 8) is-
3 4
(A) 60° (B) 90° (C) tan 1   (D) tan 1  
5 3
Solution:
dy x dy 2
x2 = 32y  =  y2 = 4x  =
dx 16 dx y
 dy   dy  1
at (16, 8),   = 1,   =
 dx  1  dx 2 4

 
 1− 1 
So, required angle = tan 1  4  = tan 1  3  Ans. (C)
  
 1  5
 1 + 1   
 4
 

Example 11:
Check the orthogonality of the curves y2 = x & x2 = y.

122 Rate Measure, Tangent and Normal


Solution:
Solving the curves simultaneously we get points of intersection as (1, 1) and (0, 0).
 dy  1
At (1,1) for first curve 2y   =1  m1 =
 dx  1 2
 dy 
and for second curve 2x =    m2 = 2
 dx 2

m1m2  1 at (1, 1).


But at (0, 0) clearly x-axis and y-axis are their respective tangents hence they are orthogonal
at (0,0) but not at (1, 1). Hence these curves are not said to be orthogonal.

Example 12:
If curve y = 1 ax2 and y = x2 intersect orthogonally then the value of a is -
1 1
(A) (B) (C) 2 (D) 3
2 3
Solution:
y=1 ax2
dy
 = 2ax
dx
y = x2
dy
 = 2x
dx
 dy   dy 
Two curves intersect orthogonally if     = 1
 dx  1  dx 2
 ( 2ax) (2x) = 1
 4ax2 = 1 ..... (i)
Now eliminating y from the given equations we have 1 ax2 = x2
 (1 + a)x2 = 1 ..... (ii)
4a 1
Eliminating x2 from (i) and (ii) we get =1 a= Ans. (B)
1+a 3

Concept Builders - 4

(i) If two curves y = ax and y = bx intersect at an angle , then find the value of tan 
(ii) Find the angle of intersection of curves y = 4 x2 and y = x2.
(iii) Find the cosine of angle of intersection of curves y = 2x nx and y = x2x 1 at (1, 0).

(iv) Find the angle between the curves y = lnx and y = (lnx) 2 at their point of intersections.
(v) Find the shortest distance between line y = x 2 and y = x2 + 3x + 2

Rate Measure, Tangent and Normal 123


ANSWER KEY FOR CONCEPT BUILDER

1. (i) 12 cm (ii) 80 cm2/s (iv) 8 (v) 9 km/hr

2. (i) 0.208 (ii) 10.05 (iii) 7.5 m3 (iv) (B)

2 a
3. (i) units (ii) x y+5=0 (iii)
5 2b

(iv) (a) Tangent : y = 2x + 1, Normal :x + 2y = 7

(b) Tangent : 2x + y = 2, Normal :x 2y = 6

na − nb 4 2
4. (i) (ii) tan 1   (iii) 1
1 + na nb  7 
 

 e  3
(iv) 45° at (1, 0) and tan 1
 2  at (e, 1) (v)
e +2 2

124 Rate Measure, Tangent and Normal


Objective Exercise - I

1. A spherical iron ball 10 cm in radius is coated with a layer of ice of uniform thickness that melts
at a rate of 50 cm3/min. When the thickness of ice is 5 cm, then the rate at which the thickness
of ice decreases, is-
5 1 1 1
(A) cm/min (B) cm/min (C) cm/min (D) cm/min
6 54 18 36

2. A Spherical balloon is being inflated at the rate of 35cc/min. The rate of increase in the surface
area (in cm2/min.) of the balloon when its diameter is 14 cm, is:
(A) 10 (B) 10 10 (C) 100 (D) 10

3. If the surface area of a sphere of radius r is increasing uniformly at the rate 8cm 2/s, then the
rate of change of its volume is:
(A) proportional to r2 (B) constant
(C) proportional to r (D) proportional to r

4. The approximate value of tan 46° is (take  = 22/7):


(A) 3 (B) 1.035 (C) 1.033 (D) 1.135

5. The equation of tangent at the point (at 2, at3) on the curve ay2 = x3is-
(A) 3tx 2y = at3 (B) tx 3y = at3 (C) 3tx + 2y = at3 (D) None of these

6. The equation of tangent to the curve x + y = a at the point (x1, y1) is-
x y 1 x y
(A) + = (B) + = a
x1 y1 a x1 y1

(C) x x 1 + y y 1 = a (D) None of these

7. The equation of normal to the curve y = x3 2x2 + 4 at the point x = 2 is-


(A) x + 4y = 0 (B) 4x y=0 (C) x + 4y = 18 (D) 4x y = 18

8. The slope of the normal to the curve x = a( sin), y = a(1 cos) at point  = /2 is-
(A) 0 (B) 1 (C) 1 (D) 1/ 2

9. The coordinates of the points on the curve x = a( + sin ), y = a(1 cos ), where tangent is
inclined at an angle /4 to the x-axis are-
          
(A) (a, a) (B)  a  − 1  ,a  (C)  a  + 1  ,a  (D)  a,a  + 1  
 2    2    2 

Rate Measure, Tangent and Normal 125


x
x y −
10. The line + = 1 touches the curve y = be a at the point-
a b
(A) (0, a) (B) (0, 0) (C) (0, b) (D) (b, 0)

11. If the tangent to the curve 2y3 = ax2 + x3 at a point (a, a) cuts off intercepts p and q on the
coordinate's axes, where p2 + q2 = 61, then a equal-
(A) 30 (B) 30 (C) 0 (D) ±30

12. The sum of the intercepts made by a tangent to the curve x + y = 4 at point (4, 4) on
coordinate axes is-
(A) 4 2 (B) 6 3 (C) 8 2 (D) 256

13. Consider the curve represented parametrically by the equation x = t3 4t2 3t and y = 2t2 + 3t
5 where t  R. If H denotes the number of points on the curve where the tangent is horizontal
and V the number of point where the tangent is vertical then
(A) H = 2 and V = 1 (B) H = 1 and V = 2 (C) H = 2 and V = 2 (D) H = 1 and V = 1

14. The slope of the curve y = sinx + cos2x is zero at the point, where-
 
(A) x = (B) x = (C) x =  (D) No where
4 2

15. The angle of intersection of curves 2y = x3 and y2 = 32x at the origin is-
  
(A) (B) (C) (D) None of these
6 4 2

16. The lines tangent to the curve y3 x2y + 5y 2x = 0 and x4 x3y2 + 5x + 2y = 0 at the origin
intersect at an angle  equal to-
   
(A) (B) (C) (D)
6 4 3 2

17. The angle of intersection between the curves y2 = 8x and x2 = 4y 12 at (2, 4) is-
(A) 90° (B) 60° (C) 45° (D) 0°

18. The length of subtangent to the curve x2 + xy + y2 =7 at the point (1, 3) is-
(A) 3 (B) 5 (C) 15 (D) 3/5

19. The normal to the curve x = a(1 + cos), y = asin at '' always passes through the fixed point-
(A) (a, 0) (B) (0, a) (C) (0, 0) (D) (a, a)

20. The normal to the curve x = a(cos + sin), y = a(sin cos) at any point  is such that-
 
(A) it passes through the origin (B) it makes angle  −   with the x-axis
 2 
  
(C) it passes through  a , −a  (D) it is a constant distance from the origin
 2 

126 Rate Measure, Tangent and Normal


Objective Exercise - II

Single Correct Type Questions

1. If an equation of a tangent to the curve, y = cos (x + y), 1  x  1 + , is x + 2y = k then k is


equal to:
 
(A) (B) 2 (C) (D) 1
4 2

x2 y 2
2. The curve x2 y2 = 5 and + = 1 cut each other at any common point at an angle-
18 8
(A) /4 (B) /3 (C) /2 (D) None of these

3 The value of n for which the area of the triangle included between the co-ordinate axes and
any tangent to the curve xyn = an+1 is constant, is
(A) 1 (B) 0 (C) 1 (D) a

One or More Than One Correct

4. If tangent drawn to the curve f(x) = x3 9x 1 at P(x0, f(x0)) meets the curve again at Q, mA
denotes the slope of the tangent at A and mOB denotes the slope of the line joining 'O' origin
and a point B on the curve, then
(A) mQ 4mP = 27 (B) mQ 4mP = 9
mOP mOP 1
(C) = 2, where x0 = 1 (D) = , where x0 = 1
mOQ mOQ 2

5. The abscissa of a point on the curve xy = (a + x) 2, the normal at which cuts off numerically
equal intercepts from the coordinate axes, is
a
(A) − (B) 2a
2
2a
(C) (D) 2a
2

nx
6. For function f(x) = , which of the following statements are true.
x
(A) f(x) has horizontally tangent at x = e (B) f(x) cuts the x-axis only at one point
(C) f(x) is many-one function (D) f(x) has one vertical tangent

7. The equation of tangent drawn to the curve y = (x + 1) 3, from origin is


(A) y = 3x (B) y = 3x
(C) 4y = 27x (D) y = 0

Rate Measure, Tangent and Normal 127


8. For the curve represented parametrically by the equations, x = 2ln cot t + 1 and y = tant + cott
where t is parameter:

(A) tangent at t = is parallel to x axis
4

(B) normal at t = is parallel to y axis
4

(C) tangent at t = is parallel to the line y = x
4
(D) tangent and normal intersect at the point (2,1)

9. The equation of the tangents to the curve y = x4 from the point (2, 0) not on the curve, are given
by
(A) y = 0 (B) y 1 = 5 (x 1)
4096 2048  8 32 64  2
(C) y = x −  (D) y = x − 
81 27  3 243 81  3

10. A curve f(x) is given by.


 x2 − 2x + 3, x  0

f(x) =  2 11
 −x − 4x − , x  0
 2
If the line y = mx + c touches the f(x) at two points then.
3 13
(A) m = 1 (B) m = 7 (C) c = (D) c =
4 4

11. If y = mx + c is tangent at one point (x1, y1) and normal at other point (x2, y2) to the curve
y = 8x3 2x, then which of the following is true

5 5
(A) x2 = 2x1 (B) x21 =
96
3
(C) sum of slopes of all such lines is 3 (D) sum of slopes of all such lines is
2

x y K
12. If + = 1 is a tangent to the curve x = Kt, y = , K > 0 then:
a b t
(A) a > 0, b > 0 (B) a > 0, b < 0 (C) a < 0, b > 0 (D) a < 0, b < 0

x3 5x2
13. The co-ordinates of the point(s) on the graph of the function, f(x) = − + 7x 4
3 2
where the tangent drawn cut off intercepts from the co-ordinate axes which are equal in
magnitude but opposite in sign, is
(A) (2, 8/3) (B) (3, 7/2) (C) (1, 5/6) (D) none

128 Rate Measure, Tangent and Normal


Match the List
14. List I List II
x

(P) If curves y2 = 4ax and y = e 2a
(1) 3
can take value
(Q) If  is angle between the curves y = [| sin x | + | cos x|], (2) 1
([  ] denote GIF) and x2 + y2 = 5 then cosec2 is
(R) If curves y2 = 4a (x + a) and y2 = 4b (x + b) intersects
a
each other orthogonally then can be equal to____ (3) 5/4
b

(S) If y = x2 + 3x + c and x =y2 + 3y + c touches each other


(4) 2
(A) (P)→(1,2,3,4); (Q)→(3); (R)→(1,2,3,4); (S)→(2)
(B) (P)→(4); (Q)→(1,2); (R)→(3); (S)→(2,3,4)
(C) (P)→(1); (Q)→(2); (R)→(3,4); (S)→(1,3)
(D) (P)→(2,3); (Q)→(1,4); (R)→(2); (S)→(4)

Rate Measure, Tangent and Normal 129


Subjective Exercise - I

1. Water is being poured on to a cylindrical vessel at the rate of 1 m 3/min. If the vessel has a
circular base of radius 3 m, find the rate at which the level of water is rising in the vessel.

2. A man 1.5 m tall walks away from a lamp post 4.5 m high at the rate of 4 km/hr.
(i) how fast is the farther end of the shadow moving on the pavement?
(ii) how fast is his shadow lengthening?

3. A particle moves along the curve 6y = x3 + 2. Find the points on the curve at which the y
coordinate is changing 8 times as fast as the x coordinate.

4. An inverted cone has a depth of 10 cm & a base of radius 5 cm. Water is poured into it at the
rate of 1.5 cm3/min. Find the rate at which level of water in the cone is rising, when the depth
of water is 4 cm.

5. (i) Use differentials to approximate the values of ; (a) 36.6 and (b) 3
26 .
(ii) If the radius of a sphere is measured as 9 cm with an error of 0.03 cm, then find the
approximate error in calculating its volume.

6. Sand is pouring from a pipe at the rate of 12 cc/sec. The falling sand forms a cone on the ground
in such a way that the height of the cone is always 1/6th of the radius of the base. How fast is
the height of the sand cone increasing when the height is 4 cm.

7. Find the equation of the normal to the curve y = (1 + x) y + sin 1(sin2x) at x = 0.

8. Find all the lines that pass through the point (1, 1) and are tangent to the curve represented
parametrically as x = 2t t2and y = t + t2.

7
9. The tangent to y = ax2+ bx + at (1, 2) is parallel to the normal at the point ( 2, 2) on the curve
2
y = x2 + 6x + 10. Find the value of a and b.

41x3
10. A line is tangent to the curve f(x) = at the point P in the first quadrant, and has a slope of
3
2009. This line intersects the y-axis at (0, b). Find the value of 'b'.

11. The curve y = ax3 + bx2 + cx + 5, touches the x - axis at P ( 2, 0) & cuts the y-axis at a point Q
where its gradient is 3. Find a, b, c.

12. Find the gradient of the line passing through the point (2,8) and touching the curve y = x 3.

13. Find the point of intersection of the tangents drawn to the curve x2y = 1 y at the points where
it is intersected by the curve xy = 1 y.

130 Rate Measure, Tangent and Normal


Subjective Exercise - II

Integer type Questions

1
1. If the two curves C1: x = y2 and C2: xy = k cut at right angles find the value of .
k2

2. Water is dripping out from a conical funnel of semi vertical angle /4, at the uniform rate of
2 cm3/ sec through a tiny hole at the vertex at the bottom. When the slant height of the water
2
is 4 cm, and the rate of decrease of the slant height of the water is cm/s, find the value of
k
k.

3. A water tank has the shape of a right circular cone with its vertex down. Its altitude is 10 cm
and the radius of the base is 15 cm. Water leaks out of the bottom at a constant rate of 1cu.
cm/sec. Water is poured into the tank at a constant rate of m + k  cu. cm/sec cu. cm/sec.
Compute k + m so that the water level will be rising at the rate of 4 cm/sec at the instant when
the water is 2 cm deep.

4. A variable ABC in the xy plane has its orthocentre at vertex 'B', a fixed vertex 'A' at the origin
7x2
and the third vertex 'C' restricted to lie on the parabola y = 1 + . The point B starts at the
36
point (0, 1) at time t = 0 and moves upward along the y axis at a constant velocity of 2cm/sec.
m
If the area of the triangle increasing at the rate of cm2/sec (where m and n are coprime
n
7
numbers) when t = sec, find m + n.
2

5. The number of distinct line(s) which is/are tangent at a point on curve 4x3 = 27 y2 and normal
at other point, is:

6. The sum of the ordinates of point of contacts of the common tangent to the parabolas
y = x2 + 4x + 8 and y = x2 + 8x + 4, is

Comprehension (Numerical Type)


Let P (h, k) be any point on curve y = f(x). Let tangent drawn at point P meets x-axis at T &
dy 
normal at point P meets x-axis at N (as shown in figure) and m =  = slope of tangent.
dx (h, k)

Rate Measure, Tangent and Normal 131


1
(i) Length of Tangent = PT = | k | 1+
m2
(ii) Length of Normal = PN = | k | 1 + m2

k
(iii) Length of subtangent = Projection of segment PT on x-axis = TM =
m
(iv) Length of subnormal = projection of line segment PN on x axis =MN = |km|

7. Find the product of length of tangent and length of normal for the curve y = x3 + 3x2 + 4x 1 at
point x = 0, is a the find 4a.

8. Find number of values of 'p' such that the length of the subtangent and subnormal is equal
for the curve y = epx + px at the point (0, 1).

−
9. If the length of subnormal to x = 2 cos t, y = 3sin t at t = is l then find 2l
4


10. The length of the normal to the curve x = a ( + sin), y = a (1 cos ) at  = is p a then find
2
the value of p (p is not a perfect square)

132 Rate Measure, Tangent and Normal


JEE-Main (Previous Year Questions)

1. The normal to the curve, x2 + 2xy 3y2= 0, at (1, 1): [JEE(Main)-2015]


(1) meets the curve again in the third quadrant
(2) meets the curve again in the fourth quadrant
(3) does not meet the curve again
(4) meets the curve again in the second quadrant

 1 + sin x    
2. Consider f(x) = tan 1   , x   0,  , A normal to y = f(x) at x = also passes through
 1 − sin x   2  6
 
the point: [JEE(Main)-2016]
   2   
(1)  , 0  (2) (0, 0) (3)  0,  (4)  , 0 
4   3  6 

3. The normal to the curve y(x 2) (x 3) = x + 6 at the point where the curve intersects the
y-axis passes through the point: [JEE(Main)-2017]
 1 1  1 1  1 1 1 1
(1)  ,  (2)  − , −  (3)  ,  (4)  , − 
2 3  2 2 2 2 2 3

2
4. The tangent to the curve, y = xex passing through the point (1, e) also passes through the point:
[JEE(Main)-2019]
4  5 
(1) (2, 3e) (2) (3, 6e) (3)  ,2e  (4)  , 2e 
3  3 

5. The tangent to the curve y = x2 5x + 5, parallel to the line 2y = 4x + 1, also passes through the
point: [JEE(Main)-2019]
1  7 1  1   1 7
(1)  , −7  (2)  ,  (3)  − , 7  (4)  , 
8  2 4  8  4 2

6. Let S be the set of all values of x for which the tangent to the curve y = f(x) = x 3 x2 2x at
(x, y) is parallel to the line segment joining the points (1, f(1)) and ( 1, f( 1)), then S is equal to:
[JEE(Main)-2019]
 1  1   1  1 
(1) − , −1 (2)  , −1 (3) − , 1 (4)  , 1
 3  3   3  3 

7. If the tangent to the curve, y = x3 + ax b at the point (1, 5) is perpendicular to the line,
x + y + 4 = 0, then which one of the following points lies on the curve? [JEE(Main)-2019]
(1) (2, 1) (2) (2, 2) (3) ( 2, 1) (4) ( 2, 2)

Rate Measure, Tangent and Normal 133


8. If the tangent to the curve y =
x
x −3
2 ( )
, x  R, x   3 , at a point (, )  (0, 0) on it is parallel

to the line 2x + 6y 11 = 0, then: [JEE(Main)-2019]


(1) |6 + 2| = 9 (2) |6 + 2| = 19 (3) |2 + 6| = 11 (4) |2 + 6| = 19

9. A spherical iron ball of radius 10 cm is coated with a layer of ice of uniform thickness that melts
at a rate of 50 cm3 /min. When the thickness of the ice is 5 cm, then the rate at which the
thickness (in cm/min) of the ice decreases, is: [JEE(Main)-2019]
1 1 1 5
(1) (2) (3) (4)
18 36 9 6

 3
10. Let the normal at a point P on the curve y2 3x2 + y + 10 = 0 intersect the y-axis at  0,  . If m
 2
is the slope of the tangent at P to the curve, then |m| is equal to ______. [JEE(Main)-2020]

11. The equation of the normal to the curve y = (1 + x)2y + cos2(sin 1x) at x = 0 is:
[JEE(Main)-2020]
(1) y + 4x = 2 (2) x + 4y = 8 (3) 2y + x = 4 (4) y = 4x + 2

12. If the surface area of a cube is increasing at a rate of 3.6 cm2/sec, retaining its shape; then the
rate of change of its volume (in cm3/sec.), when the length of a side of the cube is 10 cm is:
[JEE(Main)-2020]
(1) 9 (2) 20 (3) 18 (4) 10

13. If the tangent to the curve, y = ex at a point (c, ec) and the normal to the parabola, y 2 = 4x at
the point (1, 2) intersect at the same point on the x-axis, then the value of c is_______.
[JEE(Main)-2020]

14. Which of the following points lies on the tangent to the curve x 4ey + 2 y + 1 = 3 at the point
(1, 0)? [JEE(Main)-2020]
(1) (2, 2) (2 (2, 6) (3) ( 2, 4) (4) ( 2, 6)

15. If the lines x + y = a and x y = b touches the curve y = x2 3x + 2 at the points where the
a
curve intersects the x-axis, then is equal to ______. [JEE(Main)-2020]
b

16. If the tangent to the curve, y = f(x) = xlogex, (x > 0) at a point (c, f(c)) is parallel to the line-
segment joining the points (1,0) and (e, e), then c is equal to: [JEE(Main)-2020]
 1   1 
1     e−1
(1) (2) e  1−e 
(3) e  e− 1 
(4)
e−1 e

134 Rate Measure, Tangent and Normal


17. If the tangent to the curve y = x3 at the point P(t, t3) meets the curve again at Q, then the
ordinate of the point which divides PQ internally in the ratio 1 : 2 is : [JEE(Main)-2021]
(1) 2t 3
(2) t
3
(3) 0 (4) 2t
3

18. If the curve y = ax2 + bx + c, x  R, passes through the point (1, 2) and the tangent line to this
curve at origin is y = x, then the possible values of a, b, c are: [JEE(Main)-2021]
(1) a = 1, b = 1, c = 0 (2) a = 1, b = 1, c = 1
1 1
(3) a = 1, b = 0, c = 1 (4) a = ,b= ,c=1
2 2

19. Let M and N be the number of points on the curve y 5 9xy + 2x = 0, where the tangents to the
curve are parallel to x-axis and y-axis, respectively. Then the value of M + N equal.
[JEE(Main)-2021]

20. If the tangent to the curve y = x3 − x2 + x at the point (a, b) is also tangent to the curve
y = 5x2 + 2x 25 at the point (2, 1), then |2a + 9b| is equal to ________.
[JEE(Main)-2021]
x y
21. Let S be the set of all the natural numbers, for which the line + = 2 is a tangent to the
a b
n n
x y
curve   +   = 2 at the point (a, b), ab  0, Then: [JEE(Main)-2022]
a b
(1) S =  (2) n(S) = 1 (3) S = {2k : k  N} (4) S = N

x−a
22. If the equation of the normal to the curve y = at the point (1, -3) is x 4y = 13,
(x + b)(x − 2)
then the value of a + b is equal to ________. [JEE(Main)-2023]

23. the number of points on the curve y = 54x5 135x4 70x3 + 180x2 + 210x at which the normal
lines are parallel to x + 90y + 2 = 0 is: [JEE(Main)-2023]

24. Let then tangent to the curve x2 + 2x 4y + 9 = 0 at the point P(1, 3) on it meet the y-axis at
A. Let the line passing through P and parallel to the line x 3y = 6 meet the parabola y2 = 4x at
B. If B lies on the line 2x 3y = 8, then (AB)2 is equal to ________. [JEE(Main)-2023]

 3  1
25. Let a curve y = f(x), x  (0 ) pass through the point P  1,  and Q  a,  . If the tangent at any
 2  2
point R(b, f(b)) to the given curve cuts the y-axis at the point S(0, c) such that bc = 3, then (PQ)2
is equal to ________. [JEE(Main)-2023]

26. Let the quadratic curve passing through the point (-1, 0) and touching the line y = x at (1, 1) be
y = f(x). Then the x-intercept of the normal to the curve at the point (,  + 1) in the first
quadrant is: [JEE(Main)-2023]

Rate Measure, Tangent and Normal 135


JEE-Advanced (Previous Year Questions)

1. Find the equation of the straight line which is tangent at one point and normal at another point
of the curve, x = 3t2, y = 2t3. [REE 2000 (Mains)]

3
2. If the normal to the curve, y = f(x) at the point (3, 4) makes an angle with the positive
4
x-axis. Then f '(3) = [JEE 2000 (Scr.)]
3 4
(A) 1 (B) (C) (D) 1
4 3

3. The point(s) on the curve y3 + 3x2 = 12y where the tangent is vertical, is(are)
[JEE 2002 (Scr.)]
 4   11   4 
(A)   , −2  (B)   , 1 (C) (0, 0) (D)   ,2 
 3   3   3 

4. Tangent to the curve y = x2 + 6 at a point P (1, 7) touches the circle x2 + y2 + 16x + 12y + c = 0 at
a point Q. Then the coordinates of Q are [JEE 2005 (Scr.)]
(A) ( 6, 11) (B) ( 9, 13) (C) ( 10, 15) (D) ( 6, 7)

5. The tangent to the curve y = ex drawn at the point (c, ec) intersects the line joining the points
(c 1, ec 1) and (c + 1, ec+1) [JEE 2007]
(A) on the left of x = c (B) on the right of x = c
(C) at no point (D) at all points

6. The slope of the tangent to the curve (y x5)2 = x(1 + x2)2 at the point (1, 3) is
[JEE (Advanced) 2014]

136 Rate Measure, Tangent and Normal


6 Monotonicity
Monotonicity at a Point

(a) A function f(x) is called an increasing function at point x = a, if in a sufficiently small


neighbourhood of x = a ; f(a h) < f(a) < f(a + h)

(b) A function f(x) is called a decreasing function at point x = a, if in a sufficiently small


neighbourhood of x = a ; f(a h) > f(a) > f(a + h)

Note: If x = a is a boundary point, then use the appropriate one sides inequality to test monotonicity
of f(x).

(c) Testing of Monotonicity of Differentiable Function at A Point.


(i) If f '(a) > 0, then f(x) is increasing at x = a.
(ii) If f '(a) < 0, then f(x) is decreasing at x = a.
(iii) If f '(a) = 0, then examine the sign of f '(a+) and f '(a ).
(1) If f '(a+) > 0 and f '(a ) > 0, then increasing
(2) If f '(a+) < 0 and f '(a ) < 0, then decreasing
(3) otherwise neither increasing nor decreasing.

Monotonocity 137
Example 1:
Let f(x) = x3 3x + 2. Examine the nature of function at points x = 0, 1 & 2.
Solution:

f(x) = x3 3x + 2

f'(x) = 3(x2 1) = 0
 x = ±1
(i) f '(0) = 3  decreasing at x = 0
(ii) f '(1) = 0
also, f '(1+) = positive and f'(1 ) = negative
 neither increasing nor decreasing at x = 0.
(iii) f '(2) = 9  increasing at x = 2

Concept Builders - 1

(i) If function f(x) = x3 + x2 x + 1 is increasing at x = 0 & decreasing at x = 1, then find the greatest
integral value of .

Monotonicity Over an Interval

(a) A function f(x) is said to be monotonically increasing (MI) in (a, b) if f '(x)  0 where equality
holds only for discrete values of x i.e. f '(x) does not identically become zero for x  (a, b) or
any sub interval.
(b) f(x) is said to be monotonically decreasing (MD) in (a, b) if f'(x) 0 where equality holds only for
discrete values of x i.e. f'(x) does not identically become zero for x  (a, b) or any sub interval.

Note: (i) A function is said to be monotonic if it's either increasing or decreasing.


(ii) If a function is invertible it has to be either increasing or decreasing.

Example 2:

( )
Prove that the function f(x) = log x3 + x6 + 1 is entirely increasing.

Solution:

(
Now, f(x) = log x3 + x6 + 1 )
1  2 x5  3x2
f '(x) =  3x +  = >0
x3 + x6 + 1  2 x6 + 1  x6 + 1
 f(x) is increasing.
Example 3:
Find the intervals of monotonicity of the function y = x2 loge|x|, (x  0).

Solution:

Let y = f(x) = x2 loee|x|

138 Monotonocity
1
f'(x) = 2x ; for all x (x  0)
x

f'(x) =
2x2 − 1
 f '(x) =
( )(
2x − 1 )
2x + 1
x x
 1   1 
So f' (x) > 0 when x   − ,0    ,   and f '(x) < 0
 2   2 
 1   1 
when x   −, −    0, 
 2  2
 1   1 
 f(x) is increasing when x   − ,0    ,
 2   2 
 1   1 
and decreasing when x   −, −    0,  Ans.
 2  2

Concept Builders - 2

  
(i) If f(x) = sinx + n |secx + tanx| 2x for x   − ,  then check the monotonocity of f(x)
 2 2
(ii) Prove that y = ex + sinx is increasing in x  R+

Greatest and Least Value of a Function

(a) Extreme Value Theorem: If f is continuous on [a, b] then f takes on, a least value m and
a greatest value M on this interval.

NOTE:
(a) Continuity through the interval [a,b] is essential for the validity of this theorem. There is
a discontinuity at x = c, c  [a,b]. The function has a minimum value at the x = a and has
no maximum value.

Monotonocity 139
(b) If a continuous function y = f(x) is increasing in the closed interval [a, b], then f(a) is the
least value and f(b) is the greatest value of f(x) in [a, b] (figure-1)

(c) If a continuous function y = f(x) is decreasing in [a, b], then f(b) is the least and f(a) is
the greatest value of f(x) in [a, b]. (figure-2)

(d) If a continuous function y = f(x) is increasing/decreasing in the (a, b), then no greatest
and least value exist.

Example 4:
x  1 
Show that f(x) = sin 1
nx is decreasing in x   , 3  . Also find its range.
1 + x2  3 
Solution:

f(x) = sin 1 x
nx = tan 1x nx  f '(x) =
1 1
=
(
− 1 + x2 − x )
1 + x2 1 + x2 x x 1 + x2( )
 1 
 f '(x) < 0  x   , 3
 3 
 f(x) is decreasing.
 1 
f(x)|max = f 
 3
 =

6
1
+ n3 and f(x)|min = f
2
( 3 ) = 3 1
2
n3

 1  1 
 Range of f(x) =  − n3, + n3 Ans.
 3 2 6 2 

Example 5:
Find the greatest and least value of f(x) = x3 + 5x + ex in [1, 3]
Solution:
f '(x) = 3x2 + 5 + ex f(x) is always increasing.
Least value = f(1) = 6 + e
greatest value = f(3) = (42 + e3)

140 Monotonocity
Concept Builders - 3

x3 x2
(i) Let f(x) = − + 2 in [ 2, 2]. Find the greatest and least value of f(x) in [ 2, 2]
3 2

Proving Inequalities Using Monotonicity


Comparison of two functions f(x) and g(x) can be done by analyzing their monotonic behavior.

Significance of the Sign of IInd Order Derivative


The sign of the 2nd order derivative determines the concavity of the curve.
If f ''(x) > 0  x  (a, b) then graph of f(x) is concave upward in (a, b).
Similarly, if f ''(x) < 0  x  (a, b) then graph of f(x) is concave downward in (a, b).

Rolle's Theorem
Let f be a function that satisfies the following three conditions:
(a) f is continuous on the closed interval [a, b].
(b) f is differentiable on the open interval (a, b)
(c) f(a) = f(b)
Then there exist at least one number c in (a, b) such that f '(c) = 0.

Note: If f is differentiable function then between any two consecutive roots of f(x) = 0, there is at
least one root of the equation f '(x) = 0.
(d) Geometrical Interpretation:
Geometrically, the Rolle's theorem says that somewhere between A and B the curve has at
least one tangent parallel to x-axis.
Example 6:
Verify Rolle's theorem for the function f(x) = x3 3x2 + 2x in the interval [0, 2].

Monotonocity 141
Solution:
Here we observe that
(a) f(x) is polynomial and since polynomial are always continuous, as well as differentiable.
Hence f(x) is continuous in the [0,2] and differentiable in the (0, 2).
and
(b) f(0) = 0, f(2) = 23 3. (2)2 + 2(2) = 0
 f(0) = f(2)
Thus, all the condition of Rolle's theorem are satisfied.
So, there must exists some c  (0, 2) such that f'(c) = 0
1
 f '(c) = 3c2 6c + 2 = 0  c = 1 ± 1
3
1
where both c = 1 ±  (0, 2) thus Rolle's theorem is verified.
3

Example 7:
4
Let Rolle's theorem holds for f(x) = x3 + bx2 + ax, when 1  x  2 at the point c = , then find
3
a + b.
Solution:
f(1) = f(2) 1 + b + a = 8 + 4b + 2a
a + 3b + 7 = 0 .......(1)
f '(c) = 3x2 + 2bx + a = 0
16 8b
+ + a = 0  3a + 8b + 16 = 0 .........(2)
3 3
By solving a = 8, b = 5

Concept Builders - 4

(i) Verify Rolle's theorem for y = 1 x4/3 on the interval [ 1,1]


(ii) (a) Let f(x) = 1 x2/3. Show that f( 1) = f(1) but there is no number c in ( 1, 1) such that
f '(c) = 0.
Why does this not contradict Rolle's Theorem?
(b) Let f(x) = (x 1) 2. Show that f(0) = f(2) but there is no number c in (0, 2) such that
f '(c) = 0.
Why does this not contradict Rolle's Theorem?

142 Monotonocity
Lagrange's Mean Value Theorem (LMVT)

Let f be a function that satisfies the following conditions:


(i) f is continuous in [a, b]
(ii) f is differentiable in (a, b).
f(b) − f(a)
Then there is a number c in (a, b) such that f '(c) =
b−a
(a) Geometrical Interpretation:
Geometrically, the Mean Value Theorem says that somewhere between A and B the curve
has at least one tangent parallel to chord AB.
(b) Physical Interpretations:
If we think of the number (f(b) f(a))/(b a) as the average change in f over [a, b] and
f'(c) as an instantaneous change, then the Mean Value Theorem says that at some
interior point the instantaneous change must equal the average change over the entire
interval.

Example 8:
Find c of the Lagrange's mean value theorem for the function f(x) = 3x2 + 5x + 7 in the interval
[1, 3].
Solution:
Given f(x) = 3x2 + 5x + 7 ...... (i)
 f(1) = 3 + 5 + 7 = 15 and f(3) = 27 + 15 + 7 = 49
Again f '(x) = 6x + 5
Here a = 1, b = 3
Now from Lagrange's mean value theorem
f(b) − f(a) f(3) − f(1) 49 − 15
f '(c) =  6c + 5 = = = 17 or c = 2.
b−a 3−1 2

Example 9:
If f(x) is continuous and differentiable over [ 2, 5] and 4  f '(x)  3 for all x in
( 2, 5), then the greatest possible value of f(5) f( 2) is -
(A) 7 (B) 9 (C) 15 (D) 21

Monotonocity 143
Solution:
Apply LMVT
f(5) − f(−2)
f '(x) = for some x in ( 2, 5)
5 − (−2)
f(5) − f( −2)
Now, 4  3
7
28  f(5) f( 2)  
 Greatest possible value of f(5) f( 2) is 21.

Concept Builders - 5

(i) If f(x) = x2 in [a, b], then show that there exist atleast one c in (a, b) such that a, c, b are in A.P.
(ii) Find C of LMVT for f(x) = |x|3 in [2, 5].

Special Note
Use of Monotonicity in identifying the number of roots of the equation in a given interval. Suppose a
and b are two real numbers such that,
(a) Let f(x) is differentiable & either MI or MD for 0  x  b.
and
(b) f(a) and f(b) have opposite signs.
Then there is one & only one root of the equation f(x) = 0 in (a, b).

Miscellaneous Examples

Example 10:
If g(x) = f(x) + f(1 x) and f ''(x) < 0; 0  x  1, show that g(x) increasing in x  (0, 1/2) and
decreasing in x  (1/2, 1).
Solution:
 f ''(x) < 0  f '(x) is decreasing function.
Now, g(x) = f(x) + f(1 x)
 g'(x) = f '(x) f '(1 x) ......... (i)
Case I:
If x > (1 x)
 x > 1/2
 f '(x) < f '(1 x)
 f '(x) f '(1 x) < 0
 g'(x) < 0
1 
 g(x) decreases in x   , 1 
2 

144 Monotonocity
Case II:
If x < (1 x) x < 1/2
 f '(x) > f '(1 x)
 f '(x) f '(1 x) > 0
 g'(x) > 0
 g(x) increases in x  (0, 1/2)

Example 11:
 
Which of the following functions are decreasing on  0, 
 2
(A) cos x (B) cos2x (C) cos3x (D) tan x
Solution:

f(x) = cosx

f(x) = cos2x

f(x) = cos3x Non-monotonic

 
f(x) = tanx is increasing in  0,  Option A and B are correct.
 2

Example 12:
Prove that the equation e(x 1)
+ x = 2 has one solution
Solution:
Let f(x) = e(x 1)
+x
f '(x) = e (x 1)
+1
f(x) is always an increasing function
lim f(x) =  and lim f(x) = 0
x → x →

f(x) = 2 has exactly one solution.

Monotonocity 145
ANSWER KEY FOR CONCEPT BUILDER

1. (i) 4

2. (i) Increasing

8 8
3. (i) Greatest is and least value is .
3 3

4. (i) Rolle's theorem is valid

(ii) (a) f(x) is non-differentiable at x = 0 in ( 1,1)

(b) f(x) is discontinuous at x = 1 in (0,2)

117
5. (ii) C=
9

146 Monotonocity
Objective Exercise - I

1. If the function f (x) = 2 x2 kx + 5 is increasing in [1, 2], then 'k' lies in the interval
(A) ( , 4) (B) (4, ) (C) ( , 8] (D) (8, )

2. f(x) = x + 1/x, x  0 is monotonic increasing when-


(A) |x| < 1 (B) |x| > 1 (C) |x| < 2 (D) |x| > 2

3. The function xx decreases on the interval-


 1
(A) (0, e) (B) (0, 1) (C)  0,  (D) None of these
 e
4. Function f(x) = x2(x 2)2 is-
(A) increasing in (0, 1)  (2, ) (B) decreasing in (0, 1)  (2, )
(C) decreasing function (D) increasing function

5. If f and g are two decreasing function such that fog is defined, then fog will be-
(A) increasing function (B) decreasing function
(C) neither increasing nor decreasing (D) None of these

6. If function f(x) = 2x2 + 3x m log x is monotonic decreasing in the interval (0, 1), then the least
value of the parameter m is-
15 31
(A) 7 (B) (C) (D) 8
2 4
x 2
7. If f(x) = + for 7  x  7, then f(x) is monotonic increasing function of x in the interval-
2 x
(A) [7, 0] (B) [2, 7] (C) [ 2, 2] (D) [0, 7]

8. If f(x) = x3 10x2 + 200x 10, then f(x) is-


(A) decreasing in ( , 10] and increasing in (10,)
(B) increasing in ( , 10] and decreasing in (10,)
(C) increasing for every value of x
(D) decreasing for every value of x

9. The value of K in order that f(x) = sinx cosx Kx + b decreases for all real values is given by-
(A) K < 1 (B) K 1 (C) K  2 (D) K < 2

10. When 0  x  1, f(x) = |x| + |x 1| is-


(A) increasing (B) decreasing (C) constant (D) None of these

11. If 2a + 3b + 6c = 0, then at least one root of the equation ax2+ bx + c = 0 lies in the interval-
(A) (0, 1) (B) (1, 2) (C) (2, 3) (D) none

Monotonocity 147
12. The greatest value of x3 18x2+ 96x in the interval (0, 9) is-
(A) 128 (B) 60
(C) 160 (D) 120

13. Difference between the greatest and the least values of the function f(x) = x(n x 2) on [1, e2]
is
(A) 2 (B) e
(C) e 2
(D) 1

14. A maximum point of 3x4 2x3 6x2+ 6x + 1 in [0, 2] is-


(A) x = 0 (B) x = 1
(C) x = 1/2 (D) does not exist

nx
15. Range of the function f(x) = is
x
(A) ( , e) (B) ( , e2)
 2  1
(C)  −,  (D)  −, 
 e  e 


16. f(x) = 1 + [cosx]x, in 0  x 
2
(where [.] denotes greatest integer function)
(A) has a minimum value 0 (B) has a maximum value 2
  
(C) is continuous in 0,  (D) is not differentiable at x =
 2 2

17. A value of C for which the conclusion of Mean values theorem holds for the function
f(x) = logex on the interval [1, 3] is-
1
(A) 2log3e (B) loge3
2
(C) log3e (D) loge3

  1
x cos   , x  0
18. The value of c in Lagrange's theorem for the function f(x) =  x in the interval
 0, x = 0

[ 1, 1] is:
1
(A) 0 (B)
2
1
(C) (D) Non-existent in the interval
2

148 Monotonocity
19. If the function f(x) = x3 6x2+ ax + b defined on [1, 3], satisfies the rolle's theorem
2 3+1
for c = then-
3
(A) a = 11, b = 6 (B) a = 11, b = 6
(C) a = 11, b  R (D) None of these

20. The function f: [a, ) →R where R denotes the range corresponding to the given domain, with
rule f(x) = 2x3 3x2+ 6, will have an inverse provided
(A) a 1 (B) a 0
(C) a 0 (D) a 1

21. If the function f (x) = 2x2+ 3x + 5 satisfies LMVT at x = 2 on the closed interval [1, a], then the
value of 'a' is equal to:
(A) 3 (B) 4
(C) 6 (D) 1

Monotonocity 149
Objective Exercise - II

Single Correct Type Questions

1. Let f (x) and g (x) be two continuous functions defined from R → R, such that f(x1) > f(x2) and g
(x1) < g (x2),  x1 > x2, then solution set of f(g(2 2)) > f(g(3 4)) is:
(A) R (B)  (C) (1, 4) (D) R [1, 4]

2. If the equation anxn + an 1xn 1 + .... + a1x = 0 has a positive root x = , then the equation
nanxn 1 + (n 1) an 1xn 2 + .... + a1= 0 has a positive root, which is:
(A) Smaller than  (B) Greater than 
(C) Equal to  (D) Greater than or equal to 

 1 − x2 
3. The function f (x) = tan 1  2 
is -
1+ x 
(A) increasing in its domain
(B) decreasing in its domain
(C) decreasing in ( , 0) and increasing in (0,)
(D) increasing in ( , 0) and decreasing in (0,)

d
4. Given f '(1) = 1 and (f (2x)) = f '(x)  x > 0. If f '(x) is differentiable then there exists a number
dx
c  (2, 4) such that f '' (c) equals
(A) 1/4 (B) 1/8 (C) 1/4 (D) 1/8

5. Let f (x) and g (x) are two function which are defined and differentiable for all x x0.
If f(x0) = g (x0) and f ' (x) > g ' (x) for all x > x0 then
(A) f (x) < g (x) for some x > x0 (B) f (x) = g (x) for some x > x0
(C) f (x) > g (x) only for some x > x0 (D) f (x) > g (x) for all x > x0

One or More Than One Correct Type Questions

6. Let y = f(x) be a bijective function and differentiable  x  R then which of the following is/are
correct?
(A) y = f 1(x) is differentiable if y = f(x) is always concave up
(B) y = f 1(x) is differentiable if y = f(x) is always concave down
(C) y = f 1(x) is differentiable if f '(x) = 0 has no real roots
(D) None of these

7. Let f(x) is a derivable function, which is increasing for all x R (having no critical point), then:
(A) f(3 4x) is an increasing function for all x.
(B) f(3 4x) is a decreasing function for all x.
1
(C) f(x2 x) increasing for x >
2
(D) (f(x))3is an increasing function for all x.

150 Monotonocity
8. Let g'(x) > 0 and f '(x) < 0,  x  R, then
(A) g (f(x +1)) > g (f(x 1)) (B) f(g(x 1)) > f(g (x + 1))
(C) g(f (x +1)) < g(f(x 1)) (D) g(g(x + 1)) < g(g(x 1))

9. If f(x) = x3 x2 + 100x + 1001, then


 1   1 
(A) f(2000) > f(2001) (B) f  > f 
 1999   2000 
(C) f(x + 1) > f(x 1) (D) f (3x 5) > f(3x)

10. If the derivative of an odd cubic polynomial vanishes at two different value of 'x' then
(A) coefficient of x3and x in the polynomial must be same in sign
(B) coefficient of x3and x in the polynomial must be different in sign
(C) the values of 'x' where derivative vanishes are closer to origin as compared to the respective
roots on either side of origin.
(D) the values of 'x' where derivative vanishes are far from origin as compared to the respective
roots on either side of origin.

11. Let h(x) = f(x) (f(x))2 + (f(x))3for every real number 'x' and f(x) is a differentiable function,
then
(A) 'h' is increasing whenever 'f' is increasing
(B) 'h' is increasing whenever 'f' is decreasing
(C) 'h' is decreasing whenever 'f' is decreasing
(D) Nothing can be said in general

12. The set of all x for which the function h(x) = log2( 2x 3 + x2) is defined and monotonic, is
(A) (1, 3) (B) ( , 1) (C) ( 1, 1) (D) (3,)

3x2 + 12x − 1, −1  x  2
13. If f (x) =  then:
 37 − x, 2x3

(A) f(x) is increasing on [ 1, 2] (B) f(x) is continuous on [ 1, 3]


(C) f (D) f(x) has the maximum value at x = 2

14. Which of the following is/are correct?


(A) Between any two roots of excosx = 1, there exists at least one root of tanx = 1.
(B) Between any two roots of exsinx = 1, there exists at least one root of tanx = 1.
(C) Between any two roots of excosx = 1, there exists at least one root of exsinx = 1.
(D) Between any two roots of exsinx = 1, there exists at least one root of excosx = 1.

Monotonocity 151
1 
2/3  tan[x]

15. Given: f (x) = 4  − x g(x) =  x , x  0 h(x) = {x} k(x) = 5log2 (x + 3) then in [0, 1], Lagrange's
2   1, x=0
Mean Value Theorem is NOT applicable to
(A) f, g, h (B) h, k (C) f, g (D) g, h, k
where [x] and {x} denotes the greatest integer and fractional part function.

152 Monotonocity
Subjective Exercise - I

1. Find the intervals of monotonocity for the following functions & represent your solution set on

the number line.


2
(A) f(x) = 2. ex − 4x
(B) f(x) = ex/x (C) f(x) = x2e x
(D) f(x) = 2x2 n | x |

2. Find the intervals of monotonocity of the functions in [0, 2]

(A) f (x) = sin x cos x in x [0, 2] (B) g (x) = 2 sinx + cos 2x in (0  x  2).

3. Find the greatest & the least values of the following functions in the given interval if they exist.

x  1 
(A) f(x) = sin 1
n x in  , 3
x2 + 1  3 

(B) f(x) = 12x4/3 6x1/3, x  [ 1, 1]

(C) y = x5 5x4 + 5x3 + 1 in [ 1, 2]

4. Find the set of values of x for which the inequality n (1 + x) > x/(1 + x) is valid.

5. Verify Rolle's theorem for f(x) = (x a)m(x b)n on [a, b] ; m, n being positive integer.

6. Let f(x) = 4x3 3x2 2x + 1, use Rolle's theorem to prove that there exist a "c", 0 < c <1 such

that f(c) = 0.

7. f(x) and g(x) are differentiable functions for 0  x  2 such that f(0) = 5, g(0) = 0, f(2) = 8, g(2) = 1.

Show that there exists a number c satisfying 0 < c < 2 and f ' (c) = 3 g' (c).

 3 x=0
8. For what value of a, m and b does the function f (x) =  − x 2
+ 3x + a 0 x1

mx + b 1x 2

satisfy the hypothesis of the mean value theorem for the interval [0, 2].

9. Let f (x) be a increasing function defined on (0, ). If f(2a2+ a + 1) > f(3a2 4a + 1). Find the range

of a.

Monotonocity 153
Subjective Exercise - II

1. Let f(x) = 1 x x3. Find all real values of x satisfying the inequality, 1 f(x) f3(x) > f(1 5x)

max f(t) : 0  t  x , 0  x  1
2. Let f (x) = x3 x2 + x + 1 and g(x) = 
 3 − x, 1 x 2
Discuss the continuity & differentiability of g(x) in the interval (0, 2).

3. If f (x) = 2ex ae x
+ (2a + 1)x 3 monotonically increases for every x R then find the range

4. Assume that f is continuous on [a, b], a > 0 and differentiable on an open interval
f(a) f(b)
(a, b). Show that if = , then there exist x0  (a, b) such that x0f '(x0) = f(x0).
a b

5. Find all the values of the parameter 'a' for which the function ;
f(x) = 8ax a sin 6x 7x sin 5x increasing and has no critical points for all x R.

1
6. Find the values of 'a' for which the function f(x) = sin x a sin2x sin3x + 2ax increases
3
throughout the number line.

7. Find the set of values of 'a' for which the function,


 21 − 4a − a2 
f(x) =  1 −  x3 + 5x + 7 is increasing at every point of its doMain.
 a+1 
 

8. Prove that, x2 1 > 2x n x > 4(x 1) 2 n x for x > 1.

9. Prove that if f is differentiable on [a, b] and if f (a) = f (b) = 0 then for any real  there is an
x (a, b) such that f (x) + f ' (x) = 0.

10. Let a > 0 and f be continuous in [ a, a]. Suppose that f '(x) exists and f '(x)  1 for all
x ( a, a). If f(a) = a and f( a) = a, show that f(0) = 0.

Comprehension # 1
 x + sin x
Consider a function f defined by f(x) = sin 1 sin   ,  x  [0, ], which satisfies
 2 
f(x) + f(2 x) = ,  x  [, 2] and f(x) = f(4 x) for all x  [2, 4], then

11. If  is the length of the largest interval on which f(x) is increasing, then  =

(A) (B)  (C) 2 (D) 4
2

154 Monotonocity
12. If f(x) is symmetric about x = , then  =
 
(A) (B)  (C) (D) 2
2 4

13. Maximum value of f(x) on [0, 4] is:


 
(A) (B)  (C) (D) 2
2 4

Monotonocity 155
JEE(Main)-(Previous Year Questions)

1. If f and g are differentiable functions in [0, 1] satisfying f(0) = 2 = g(1), g(0) = 0 and f(1) = 6, then
for some c  ] 0, 1 [: [JEE(Main)-2014]
(1) 2f '(c) = g'(c) (2) 2f '(c) = 3g'(c)
(3) f '(c) = g'(c) (4) f '(c) = 2g '(c)

x d−x
2. Let f(x) = − , x  R , where a, b and d are non-zero real constants. Then:
a +x
2 2
b + (d− x)2
2

(1) f is an increasing function of x [JEE(Main)-2019]


(2) f is a decreasing function of x
(3) f is neither increasing nor decreasing function of x
(4) f' is not a continuous function of x

3. Let f: [0, 2] → R be a twice differentiable function such that f "(x) > 0, for all x  (0, 2).
If (x) = f(x) + f(2 x), then  is [JEE(Main)-2019]
(1) decreasing on (0, 2)
(2) increasing on (0, 2)
(3) increasing on (0, 1) and decreasing on (1, 2)
(4) decreasing on (0, 1) and increasing on (1, 2)

4. Let the function, f: [ 7, 0] → R be continuous on [ 7, 0] and differentiable on ( 7, 0). If


f( 7) = 3 and f'(x)  2, for all x  ( 7, 0), then for all such functions f, f( 1) + f(0) lies in the
interval: [JEE(Main)-2020]
(1) [ 3, 11] (2) ( , 20] (3) ( , 11] (4) [ 6, 20]

5. The value of c in the Lagrange's mean value theorem for the function f(x) = x3 4x2 + 8x + 11,
when x  [0,1] is: [JEE(Main)-2020]
2 7 −2 4− 5 4− 7
(1) (2) (3) (4)
3 3 3 3

  
6. Let f(x) = x cos 1 ( sin|x|), x   − ,  then which of the following is true?
 2 2

(1) =−
2
    
(2) f' is decreasing in  − , 0  and increasing in  0, 
 2   2
(3) f is not differentiable at x = 0
    
(4) f' is increasing in  − , 0  and decreasing in  0,  [JEE(Main)-2020]
 2   2

156 Monotonocity
 x2 +  
7. If c is a point at which Rolle's theorem holds for the function, = e   in the interval
 7x 
[3, 4], where a  R, then f"(c) is equal to: [JEE(Main)-2020]
1 1 3 1
(1) (2) − (3) (4) −
12 12 7 24

8. Let f be a twice differentiable function on (1, 6). If f(2) = 8, f'(2) = 5, f'(x)  1 and f"(x)  4, for all
x  (l, 6), then: [JEE(Main)-2020]
(1) f'(5) + f"(5)  20 (2) f(5)  10
(3) f(5) + f'(5)  28 (4) f(5) + f'(5)  26

4
9. 3
ax2 + bx 4, x    = 0, then
3
ordered pair (a, b) is equal to : [JEE(Main)-2021]
(1) ( 5, 8) (2) (5, 8) (3) (5, 8) (4) ( 5, 8)

10. Let a be an integer such that all the real roots of the polynomial 2x5 + 5x4 + 10x3 + 10x2 + 10x + 10
lie in the interval (a, a + 1) Then, |a| is equal to
[JEE(Main)-2021]

11. The number of real solutions of x7 + 5x3 + 3x + 1 = 0 is equal to ______.


[JEE(Main)-2021]
(1) 0 (2) 1 (3) 3 (4) 5

12. The number of distinct real roots of the equation x7 7x 2=0


[JEE(Main)-2022]
(1) 5 (2) 7 (3) 1 (4) 3

13. f(x)= 4loge(x 1) 2x2 + 4x + 5, x > 1, which one of the following is NOT correct?
[JEE(Main)-2022]
(1) f is increasing in (1, 2) and decreasing in (2, )
(2) f(x) = 1 has exactly two solutions

(4) f(x) = 0 has a root in the interval (e, e + 1)

1
14. Let f : (0, 1) → R be a function defined by f(x) = , and g(x) = (f( x) f(x)). Consider two
1 − e− x
statements [JEE(Main)-2023]
(I) g is increasing function in (0, 1)
(II) g is one-one in (0, 1)
Then,
(1) only (II) is true (2) Both (I) and (II) are true
(3) Neither (I) nor (II) is true (4) Only (I) is true

Monotonocity 157
15. Let f:[2, 4] →R be a differentiable function such that (x logex) f'(x) + (logex) + f(x)  1, x[2, 4]
1 1
with f ( 2 ) = and f ( 4 ) = . Consider the following two statements: [JEE(Main)-2023]
2 4
(A) : f(x)  1, for all x [2, 4]
1
(B) : f ( x )  , for all x[2, 4] Then,
8
(1) Only statement (B) is true
(2) Neither statement (A) nor statement (B) is true
(3) Both the statement (A) and (B) are true
(4) Only statement (A) is true

158 Monotonocity
JEE(Advanced)-(Previous Year Questions)

1. Let f, g: [ 1, 2] → R be continuous function which are twice differentiable on the interval


( 1, 2). Let the values of f and g at the points 1, 0 and 2 be as given in the following table:
X= 1 X=0 X=2

f(x) 3 6 0

g(x) 0 1 1

In each of the intervals ( 1, 0) and (0, 2) the function (f 3g)'' never vanishes. Then the correct
statement(s) is(are) [JEE(Advanced)-2015]
(A) f '(x) 3g'(x) = 0 has exactly three solutions in ( 1, 0) (0, 2)
(B) f '(x) 3g'(x) = 0 has exactly one solution in ( 1, 0)
(C) f '(x) 3g'(x) = 0 has exactly one solutions in (0, 2)
(D) f '(x) 3g'(x) = 0 has exactly two solutions in ( 1, 0) and exactly two solutions in (0, 2)

Answer Q. 11, Q.12 and Q.13 by appropriately matching the information given in the three
columns of the following table.
Let f(x) = x + logex x logex, x (0,)
• Column 1 contains information about zeros of f(x), f'(x) and f''(x).
• Column 2 contains information about the limiting behavior of f (x), f'(x) and f''(x) at infinity.
• Column 3 contains information about increasing/decreasing nature of f (x), and f '(x).

Column 1 Column 2 Column 3

(I) f(x) = 0 for some x (1,e2 ) (i) lim f(x) = 0 (P) f is increasing in (0,1)
x→

(II) f ' (x) = 0 for some x  (1, e) (ii) lim f(x) =  (Q) f is decreasing in (e,e2 )
x→

(III) f ' (x) = 0 for some x  (0, 1) (iii) lim f '(x) =  (R) f ' is increasing in (0, 1)
x→

(IV) f " (x) = 0 for some x  (1, e) (iv) lim f "(x) = 0 (S) f ' is decreasing in (e, e2)
x→

2. If f: R → R is a twice differentiable function such that f ''(x) > 0 for all x R, and
 1 1
f   = , f(1) = 1,then [JEE(Advanced)-2017]
2 2
(A) f '(1) > 1 (B) f '(1) 0
1 1
(C) < f '(1)  1 (D) 0 < f ' (1) 
2 2

Monotonocity 159
3. Which of the following options is the only CORRECT combination? [JEE(Advanced)-2017]
(A) (IV) (i) (S) (B) (I) (ii) (R) (C) (III) (iv) (P) (D) (II) (iii) (S)

4. Which of the following options is the only CORRECT combination? [JEE(Advanced)-2017]


(A) (III) (iii) (R) (B) (I) (i) (P) (C) (IV) (iv) (S) (D) (II) (ii) (Q)

5. Which of the following options is the only INCORRECT combination? [JEE(Advanced)-2017]


(A) (II) (iii) (P) (B) (II) (iv) (Q) (C) (I) (iii) (P) (D) (III) (i) (R)

6. For every twice differentiable function f: R → [ 2, 2] with (f(0))2 + (f '(0))2= 85, which of the
following statement(s) is (are) TRUE? [JEE(Advanced)-2018]
(A) There exist r, s  R where r < s, such that f is one-one on the open interval (r, s)
(B) There exists x0  ( 4, 0) such that |f'(x0)|  1
(C) =
x→

(D) There exists  ( 4,4) such that f () + f"() = 0 and f'()  0

7. Let f : → be defined by [JEE(Advanced)-2021]


x − 3x − 6
2
f(x) =
x2 + 2x + 4
Then which of the following statements is (are) TRUE ?
(A) f is decreasing in the interval ( 2, 1)
(B) f is increasing in the interval (1,2)
(C) f is onto
 3 
(D) Range of f is  − , 2 
 2 

8. Let [JEE(Advanced)-2022]


=  sin
k=1
2k
 
6
Let g ∶ [0, 1] → be the function defined by
g(x) = 2x + 2(1 x)

Then, which of the following statements is/are TRUE?


7
(A) The minimum value of g(x) is 2 6
1
(B) The maximum value of g(x) is 1 + 23
(C) The function g(x) attains its maximum at more than one point
(D) The function g(x) attains its minimum at more than one point

160 Monotonocity
d2f
9. Let S be the set of all twice differentiable functions f from to such that (x)  0 ,for all
dx2

x  (-1, 1). For f  S, let Xf be the number of points x  (-1, 1) for which f(x) = x. Then which of

the following statements is (are) true? [JEE(Advanced)-2023]

(A) There exists a function f  S such that Xf = 0

(B) For every function f  S, we have Xf  2

(C) There exists a function f  S such that Xf = 2

(D) There does NOT exist any function f in S such that Xf = 1

Monotonocity 161
7 Maxima-Minima
Introduction

Some of the most important applications of differential calculus are optimization problems, in which
we are required to find the optimal (best) way of doing something. Here are examples of such problems
that we will solve in this chapter
What is the shape of a vessel that can with-stand maximum pressure?
What is the maximum acceleration of a space shuttle? (This is an important question to the astronauts
who have to withstand the effects of acceleration)
What is the radius of a contracted windpipe that expels air most rapidly during a cough?
These problems can be reduced to finding the maximum or minimum values of a function. Let's first
explain exactly what we mean by maxima and minima.

(a) Maxima (Local/Relative maxima):

A function f(x) is said to have a maxima at x=a if there exist a neighbourhood (a h, a + h) {a} such
that f(a) > f(x) x  (a h, a + h) {a}

(b) Minima (Local/Relative minima):

A function f(x) is said to have a minima at x = a if there exist a neighbourhood


(a h, a + h) {a} such that f(a) < f(x) x  (a h, a + h) {a}

(c) Absolute maxima (Global maxima):

A function f has an absolute maxima (or global maxima) at c if f(c)  f(x) for all x in D,
where D is the domain of f. The number f(c) is called the maximum value of f on D.

(d) Absolute minima (Global minima):

A function f has an absolute minima at c if f(c) f(x) for all x in D and the number f(c)
is called the minimum value of f on D. The maximum and minimum values of f are called
the extreme values of f.

Note: (i) The maximum & minima values of a function are also known as local/relative maxima
or local/relative minima as these are the greatest and least values of the function
relative to some neighbourhood of the point in question.
(ii) The term 'extrema' is used both for maxima or minima.

Maxima-Minima 163
(iii) A maximum (minimum) value of a function may not be the greatest (least) value in a
finite interval.
(iv) A function can have several extreme values & a local minimum value may even be greater
than a local maximum value.
(v) Local maximum & local minimum values of a continuous function occur alternately and
between two consecutive local maximum values there is a local minimum value & vice
versa.
Explanation: Consider graph of y = f(x), x  [a, b]

x = c2, x = c4 are points of local maxima, with maximum values f(c2), f(c4) respectively.

x = c1, x = c3 are points of local minima, with minimum values f(c 1), f(c3) respectively

x = c2 is a point of global maximum

x = c3 is a point of global minimum

Consider the graph of y = h(x), x  [a, b)

h(c4)

h(c1)
h(c3)
h(c2)
h(a)

a c1 c2 c3 c4 b

x = c1, x = c4 are points of local maxima, with maximum values h(c 1), h(c4) respectively.
x = c2 are points of local minima, with minimum values h(c2) respectively.
x = c3 is neither a point of maxima nor a minima.
Global maximum is h(c4)
Global minimum is h(a)

164 Maxima-Minima
Derivative Test for Ascertaining Maxima/Minima
(a) First Derivative Test
If f '(x) = 0 at a point (say x = a) and
(i) If f '(x) changes sign from positive to negative in the neighbourhood of x = a then
x = a is said to be a point local maxima.
(ii) If f '(x) changes sign from negative to positive in the neighbourhood of x = a then
x = a is said to be a point local minima.

Note: If f '(x) does not change sign i.e. has the same sign in a certain complete neighbourhood of a,
then f(x) is either increasing or decreasing throughout this neighbourhood implying that x=a is
not a point of extremum of f.

Example 1:
1
Let f(x) = x+ ; x  0. Discuss the local maximum and local minimum values of f(x).
x
Solution:
1 x2 − 1 (x− 1)(x+ 1)
Here, f '(x) = 1 = =
x 2
x 2
x2
Using number line rule, f(x) will have local maximum at x = 1 and local minimum at x = 1
 local maximum value of f(x) = 2 at x = 1
and local minimum value of f(x) = 2 at x = 1 Ans.

Example 2:
3x2 + 12x − 1,
 −1 x 2
If f(x) =  , then
 37 − x,
 2  x  3,

(A) f(x) is increasing on [ 1, 2) (B) f(x) is continuous on [ 1, 3]


(C) f'(x) does not exist at x = 2 (D) f(x) has the maximum value at x = 2
Solution:

3x + 12x − 1,
2
−1 x 2
Given, f(x) = 
 37 − x,
 2  x  3,

6x + 12,
 −1 x 2
 f '(x) = 
 − 1,
 2  x  3,

Maxima-Minima 165
(A) Which shows f '(x) > 0 for x  [ 1, 2) So, f(x) is increasing on [ 1, 2)
Hence, (A) is correct.
(B) for continuity of f(x). (check at x = 2) RHL = 35, LHL = 35 and f(2) = 35
So, (B) is correct
(C) Right hand derivative of f at 2 = 1 and Left hand derivative of f at 2 = 24 so, not
differentiable at x = 2.
Hence, (C) is correct.
(D) We know f(x) is increasing on [ 1, 2) and decreasing on (2, 3],
Thus maximum at x = 2,
Hence, (D) is correct.
(A), (B), (C), (D) all are correct. Ans.

Concept Builders - 1

(i) Find local maxima and local minima for the function f(x) = x3 3x.
(ii) If function f(x) = x3 62x2 + ax + 9 has local maxima at x = 1, then find the value of a

(b) Second Derivative Test


If f(x) is continuous and differentiable at x = a where f'(a) = 0 and f''(a) also exists then
for ascertaining maxima/minima at x = a, 2nd derivative test can be used -
(i) If f''(a) > 0  x = a is a point of local minima
(ii) If f''(a) < 0  x = a is a point of local maxima
(iii) If f''(a) = 0  second derivative test fails. To identify maxima/minima at this
point either first derivative test or higher derivative test can be used.

Example 3:
If f (x) = 2x3 3x2 36x + 6 has local maximum and minimum at x = a and x = b respectively,
then ordered pair (a, b) is -
(A) (3, 2) (B) (2, 3) (C) ( 2, 3) (D) ( 3, 2)
Solution:
f(x) = 2x3 3x2 36x + 6
f'(x) = 6x 2
6x 36 and f''(x) = 12x 6
Now f'(x) = 0  6(x 2
x 6) = 0  (x 3) (x + 2) = 0  x = 2, 3
f ''( 2) = 30
x = 2 is a point of local maximum
f''(3) = 30  x = 3 is a point of local minimum
Hence, ( 2, 3) is the required ordered pair. Ans. (C)

Example 4:
Find the point of local maxima of f(x) = sinx (1+cosx) in x  (0, /2).

166 Maxima-Minima
Solution:
1
Let f(x) = sinx (1+ cosx) = sinx + sin 2x
2
 f'(x) = cos x + cos 2x
f''(x) = sin x 2sin 2x
Now f'(x) = 0  cos x + cos2x = 0  cos 2x = cos ( x) x = /3
 − 3
Also, f''   = − 3< 0  f(x) has a maxima at x = /3 Ans.
3 2

Example 5:
ex + e − x
Find the global maximum and global minimum of f(x) = in [ loge2, loge7].
2
Solution:
ex + e − x
f(x) = is differentiable at all x in its domain.
2
ex − e − x ex + e − x
Then f '(x) = , f "(x) =
2 2
ex − e − x
f '(x) = 0  = 0  e2x = 1 x = 0
2
f ''(0) = 1  x = 0 is a point of local minimum
Points x = loge2 and x = loge7 are extreme points.
Now, check the value of f(x) at all these three points x = loge2, 0, loge7
− loge 2 + loge 2
e +e 5
 f( loge2) = =
2 4
e0 + e−0
f(0) = =1
2
loge 7 − loge 7
e +e 25
f(loge7) = =
2 7
 x = 0 is absolute minima & x = loge7 is absolute maxima
Hence, absolute/global minimum value of f(x) is 1 at x = 0
25
and absolute/global maximum value of f(x) is at x = loge7 Ans.
7

Concept Builders - 2

nx
(i) Find local maximum value of function f(x) =
x
(ii) If f(x) = x2e 2x
(x > 0), then find the local maximum value of f(x).

Example 6:
Identify a point of local maxima/minima in f(x) = (x + 1)4.

Maxima-Minima 167
Solution:
f(x) = (x + 1)4
f '(x) = 4(x +1)3
-ve +ve dy
dx
-1
at x = 1 f(x) is having local minima  at x = 1 f(x) has point of minima.

Example 7:
Find point of local maxima and minima of f(x) = x5 5x4 + 5x3 1
Solution:
f(x) = x5 5x4 + 5x3 1
f '(x) = 5x 4
20x + 15x2 = 5x2 (x2
3
4x + 3) = 5x2 (x 1)(x 3)
f '(x) = 0  x = 0, 1, 3
f ''(x) = 10x(2x2 6x + 3)
But at x = 0, derivative sign is positive in its neighbourhood.
Now f ''(1) < 0  Maxima at x = 1
f ''(3) > 0  Minima at x = 3  Neither maxima nor minima at x = 0.

Concept Builders - 3

(i) Identify the point of local maxima/minima in f(x) = (x 3)10.

Useful Formulae of Mensuration to Remember


(a) Volume of a cuboid = bh.
(b) Surface area of a cuboid = 2 (b + bh + h).
(c) Volume of a prism = area of the base × height.
(d) Lateral surface area of prism = perimeter of the base × height.
(e) Total surface area of a prism = lateral surface area + 2 area of the base
(Note that lateral surfaces of a prism are all rectangles).
1
(f) Volume of a pyramid = (area of the base) x height.
3
1
(g) Curved surface area of a pyramid = (perimeter of the base) x slant height.
2
(Note that slant surfaces of a pyramid are triangles).
1
(h) Volume of a cone =  r2h.
3
(i) Curved surface area of a cylinder = 2  rh.
(j) Total surface area of a cylinder = 2 rh + 2 r2.
4 3
(k) Volume of a sphere = r .
3
(l) Surface area of a sphere = 4  r2.
1 2
(m) Area of a circular sector = r , when  is in radians.
2

168 Maxima-Minima
Summary of Working Rules for Solving Real Life Optimization
Problem
First
When possible, draw a figure to illustrate the problem and label those parts that are important
in the problem. Constants and variables should be clearly distinguished.
Second
Write an equation for the quantity that is to be maximized or minimized. If this quantity is

require some algebraic manipulations.


Third
If y = f (x) is a quantity to be maximum or minimum, find those values of x for which
dy
= f'(x) = 0.
dx
Fourth
Using derivative test, test each value of x for which f '(x) = 0 to determine whether it provides
a maximum or minimum or neither.
Fifth
If the derivative fails to exist at some point, examine this point as possible maximum or
minimum.
Sixth
If the function y = f(x) is defined only for x  [a, b] then examine x = a & x = b for possible
extreme values.

Example 8:
Determine the largest area of the rectangle whose base is on the x-axis and two of its vertices
2
lie on the curve y = e− x .
Solution:
2
Area of the rectangle will be A = 2a. e−a
dA d 2 2
For max. area, = (2a e−a ) = e−a [2 4a2]
da da
dA 1
=0a=
da 2
dA 1
and sign of changes from positive to negative at a = +
da 2
2
 1 
1 2 −   2
x= are points of maxima  Amax = .e  2
= sq units. Ans.
2 2 e 1/2
Example 9:
A box of maximum volume with top open is to be made by cutting out four equal squares from
four corners of a square tin sheet of side length a ft, and then folding up the flaps. Find the
side of the square base cut off.

Maxima-Minima 169
Solution:
Volume of the box is, V = x(a 2x)2 i.e., squares of side x are cut out then we will get a box
with a square base of side (a 2x) and height x.

dV
 = (a 2x)2 + x·2(a 2x)( 2)
dx
dV
= (a 2x) (a 6x)
dx
dV a a
For V to be extremum =0x= ;
dx 2 6
a
But when x = ; V = 0 (minimum) and we know minimum and maximum occurs alternately in
2
a continuous function.
a
Hence, V is maximum when x = . Ans.
6

Example 10:
If a right circular cylinder is inscribed in a given cone. Find the dimension of the cylinder such
that its volume is maximum.
Solution:
Let x be the radius of cylinder and y be its height
V = x2y
x, y can be related by using similar triangles
y h h
=  y = (r x)
r−x r r
h h
 V(x) = x2 (r x)  V(x) = (rx2 x3)
r r
h
V'(x) = (2rx 3x2)
r
2r
V'(x) = 0  x = 0,
3
h
V"(x) = (2r 6x)
r
V''(0) = 2h  x = 0 is point of minima
 2r  2r
V ''   = 2h  x = is point of maxima
3 3
 2r  h
Thus volume is maximum at x =   and y = .
3 3

170 Maxima-Minima
Concept Builders - 4

(i) Find the two positive numbers x and y whose sum is 35 and the product x2 y5 maximum.
(ii) A square piece of tin of side 18 cm is to be made into a box without top by cutting a square
from each corner and folding up the slops to form a box. What should be the side of the square
to be cut off such that volume of the box is maximum possible.
(iii) Prove that a right circular cylinder of given surface area and maximum volume is such that the
height is equal to the diameter of the base.
x2 y2
(iv) A normal is drawn to the ellipse + = 1. Find the maximum distance of this normal from
25 16
the centre.
(v) A line is drawn passing through point P(1, 2) to cut positive coordinate axes at A and B. Find
minimum area of PAB.

Important Note
(i) If the sum of two real numbers x and y is constant then their product is maximum if
they are equal.
1
i.e. xy = [(x + y)2 (x y)2]
4
(ii) If the product of two positive numbers is constant then their sum is least if they are
equal.
i.e. (x + y)2 = (x y)2 + 4xy

Least/Greatest Distance Between Two Curves


Least/Greatest distance between two non-intersecting curves always lies along the common normal.
(Wherever defined)

Note: Given a fixed point A(a, b) and a moving point P(x, f (x)) on the curve y = f (x). Then AP will be
maximum or minimum if it is normal to the curve at P.

Proof: F(x) = (x a)2 + (f (x) b))2  F '(x) = 2(x a) + 2(f (x) b) . f '(x)
 x−a  f(x) − b
f '(x) =   . Also mAP= . Hence f '(x) . mAP = − 1.
 f(x) − b  x−a

Maxima-Minima 171
Example 11:
Find the co-ordinates of the point on the curve x2 = 4y, which is at least distance from the line
y=x 4.
Solution:
Let P(x1, y1) be a point on the curve x2 = 4y at which normal is also a perpendicular to the line
y=x 4.
dy dy x
Slope of the tangent at (x1, y1) is 2x = 4  = 1
dx dx (x1 ,y1 ) 2
x1
 = 1  x1 = 2
2
 x21 = 4y1 y1 = 1
Hence required point is (2, 1)

Concept Builders - 5

(i) Find the coordinates of point on the curve y2 =8x, which is at minimum distance from the line
x + y = 2.

Some Special Points on a Curve


(a) Critical points
The points of domain for which f '(x) is equal to zero or doesn't exist are called critical
points.
(b) Stationary points
The stationary points are the points of domain where f '(x) = 0.
Note: Every stationary point is a critical point but vice-versa is not true.
(c) Point of inflection
A point where the graph of a function has a tangent line and where the concavity
changes is called a point of inflection. If function y = f(x) is double differentiable then
d2 y
the point at which = 0& changes its sign is the point of inflection.
dx2

d2 y d2 y
Note: If at any point does not exist but sign of changes about this point then it is also called
dx2 dx2
point of inflection.

172 Maxima-Minima
Example 12:
2
Find the critical point(s) & stationary point(s) of the function f ( x ) = ( x − 2) 3 (2x + 1)

Solution:
2
( ) (
f x = x−2 ) (2x + 1)
3

2 1 2
( ) ( ) 3 .2 + (2x + 1) 23 ( x − 2) ( ) 3 + 23 (2x + 1) 1

f' x − x − 2 3 = 2 x −2
1
( x−2 ) 3

 2  1 2(5 x− 5)
= 2(x − 2) + (2 x+ 1) =
 3 
1 1
(x− 2) 3
3(x− 2)3
f'(x) does not exist at x = 2 and f'(x) = 0 at x = 1
 x = 1, 2 are critical points and x = 1 is stationary point.

Example 13:
5
The point of inflection for the curve y = x 3 is -
(A) (1, 1) (B) (0, 0) (C) (1, 0) (D) (0, 1)
Solution:
d2 y 10
Here =
dx 2 1
9x 3
d2 y
From the given points we find that (0, 0) is the point of the curve where does not exist but
dx2
d2 y
sign of changes about this point.
dx2
 (0, 0) is the required point Ans. (B)

Example 14:
Find the inflection point of f(x) = 3x4 4x3. Also draw the graph of f(x) giving due importance to
maxima, minima and concavity.
Solution:

f(x) = 3x4 4x3

f'(x) = 12x3 12x2


f'(x) = 12x2(x 1)
f'(x) = 0  x = 0, 1
examining sign change of f'(x)
thus x = 1 is a point of local minima

Maxima-Minima 173
f''(x) = 12(3x2 2x)
f''(x) = 12x(3x 2)
f''(x) = 0

2
 x = 0,
3
2
Again, examining sign of f ''(x) thus x = 0,
3
are the inflection points Hence the graph of f(x) is

Concept Builders - 6

ex
(i) Find the critical points and stationary point of the function f (x) =
x
(ii) Find the point of inflection for the curve y = x3 6x2 + 12x + 5
x4 5x3
(iii) Find the intervals for f(x) = − + 3x2 + 7 in which it is
12 6
(a) Concave upward
(b) Concave downward.
(c) Hence find the points of inflection of f(x)

Miscellaneous Example

Example 15:
If a cuboid having square base has area 6. Then find its maximum volume.
Solution:
Let the base of cuboid = a × a; and height = h
Total area = 2a2 + 4ah = 6
(6 − 2a2 ) a2 (6 − 2a2 )
h=  V = a2 h =
4a 4a
dv 6 − 6a2
= =0
da 4
d2 v
a = 1 and = ve
da2
Minimum V = 1

174 Maxima-Minima
ANSWER KEY FOR CONCEPT BUILDER

1. (i) local max. at x = 1, local min. at x = 1 (ii) 121

1 1
2. (i) (ii)
e e2

3. (i) local minima at x = 3

4. (i) x = 25, y = 10. (ii) 3 cm (iv) 1 unit (v) 4 units

5. (i) (2, 4)

6. (i) x = 1 is a critical point as well as stationary point (Note x = 0 is not in the domain of f (x))

(ii) x = 2

(iii) (a) ( , 2) (3, ) (b) (2, 3) (c) x = 2 and x = 3

Maxima-Minima 175
Objective Exercise - I

 x − 1|
1. The function is monotonically decreasing at the point
x2
(A) x = 3 (B) x = 1 (C) x = 2 (D) none of these

2. If f(x) = 1 + 2 x2 + 4 x4 + 6 x6 +...... + 100 x100 is a polynomial in a real variable x, then f(x) has:
(A) neither a local maximum nor a local minimum
(B) only one local maximum
(C) only one local minimum
(D) one local maximum and one local minimum

3. Let f(x) = (1 + b2)x2 + 2bx + 1 and let m(b) be the minimum value of f(x). As b varies, the range
of m(b) is
 1 1 
(A) [0, 1] (B)  0,  (C)  , 1 (D) (0, 1]
 2 2 

4. Consider the function f(x) = x cos x sin x, then identify the statement which is correct.
(A) f is neither odd nor even (B) f is monotonic decreasing at x = 0
(C) f has a local maxima at x =  (D) f has a local minima at x = 

5. If f(x) = x3+ ax2+ bx + c has local minimum at x = 3 and local maximum at x = 1, then-
(A) a = 3, b = 9, c = 0 (B) a = 3, b = 9, c = 0
(C) a = 3, b = 9, c R (D) none of these

6. If (x a)2m(x b)2n+1, where m and n are positive integers and a > b, is the derivative of a function
f(x), then-
(A) x =a give neither a maximum, nor a minimum
(B) x = a gives a maximum
(C) x = b gives neither a maximum nor a minimum
(D) None of these

7. Consider the following behaviors of function in ( 1, 1)


I. Increasing II. Decreasing III. Continuity IV. Derivability
Which one of the following four functions exhibits atleast three of the four mentioned above?
(A) max{x, x3} (B) max{x, x2} (C) max{x, |x|} (D) max {x, [x]}
where [ ] denotes greatest integer function.

8. A minimum value of sinx cos2x is-


−2 6
(A) 1 (B) 1 (C) (D) None of these
3

176 Maxima-Minima
9. The rate of change of the function f(x) = 3x5 5x3+ 5x 7 is minimum when
1 1 1
(A) x = 0 (B) x = (C) x = − (D) x = 
2 2 2

10. Which one of the following statements does not hold good for the function f(x) = cos 1 (2x2 1) ?
(A) f is not differentiable at x = 0
(B) f is monotonic
(C) f is even
(D) f has an extremum

11. The radius of a right circular cylinder of greatest curved surface which can be inscribed in a
given right circular cone is
(A) one third that of the cone
1
(B) times that of the cone
2
2
(C) that of the cone
3
1
(D) that of the cone
2

b
12. If ax +  c for all positive x, where a, b > 0, then-
x
c2 c2
(A) ab< (B) ab
4 4
c
(C) ab (D) None of these
4

13. The maximum area of a right angled triangle with hypotenuse h is:-
h2 h2
(A) (B)
2 2

h2
h2
(C) (D)
4 2 2

2 2
14. Point 'A' lies on the curve y = e− x and has the coordinate (x, e− x ) where x > 0. Point B has the
coordinates (x, 0). If 'O' is the origin then the maximum area of the triangle AOB is
1 1
(A) (B)
2e 4e
1 1
(C) (D)
e 8e

Maxima-Minima 177
15. The sum of lengths of the hypotenuse and another side of a right angled triangle is given.
The area of the triangle will be maximum if the angle between them is:
   5
(A) (B) (C) (D)
6 4 3 12

16. If xy = c2 then the minimum value of ax + by (a > 0, b > 0, c > 0) is-


(A) c ab (B) c ab (C) 2c ab (D) 2c ab


17. The minimum value of asecx + bcosecx, 0 < a < b, 0 < x < is-
2
(A) a + b (B) a2/3+ b2/3 (C) (a2/3+ b2/3)3/2 (D) None of these

18. P is a point on positive x-axis, Q is a point on the positive y-axis and 'O' is the origin.
If the line passing through P and Q is tangent to the curve y = 3 x2 then the minimum area of
the triangle OPQ, is
(A) 2 (B) 4 (C) 8 (D) 9

19. The least area of a circle circumscribing any right triangle of area S is:
(A) S (B) 2S (C) 2 S (D) 4 S

ax3
20. The set of value(s) of 'a' for which the function f(x) = + (a + 2)x2 + (a 1)x + 2 possess a
3
negative point of inflection.
(A) ( , 2)  (0,) (B) { 4/5}
(C) ( 2, 0) (D) empty set

178 Maxima-Minima
Objective Exercise - II

Single Correct Type Questions

5
1. Minimum value of the function f(x) =  (x− k)
k=1
2
is at-

(A) x = 2 (B) x = 5/2


(C) x = 3 (D) x = 5

 b
2. The cost of running a bus from A to B, is Rs.  av +  , where v km/h is the average speed of the
 v
bus. When the bus travels at 30 km/h, the cost comes out to be Rs. 75 while at 40 km/h, it is
Rs. 65. Then the most economical speed (in km/h) of the bus is:
(A) 40 (B) 60
(C) 45 (D) 50

3. Two sides of a triangle are to have lengths 'a' cm & 'b' cm. If the triangle is to have the maximum
area, then the length of the median from the vertex containing the sides 'a' and 'b' is
1 2a + b
(A) a 2 + b2 (B)
2 3
a2 + b2 a + 2b
(C) (D)
2 3

4. A rectangle has one side on the positive y-axis and one side on the positive x - axis. The upper
nx
right-hand vertex of the rectangle lies on the curve y = . The maximum area of the rectangle
x2
is:
1 1

(A) e 1
(B) e 2
(C) 1 (D) e2

One or More Than One Correct Type Questions


5. If f(x) = |x| + |x 1| + |x 2|, then-
(A) f(x) has minima at x = 1
(B) f(x) has maxima at x = 0
(C) f(x) has neither maxima nor minima at x = 3
(D) None of these

ex
6. Let f(x) = and g(x) = f' (x) then
1 + x2
(A) g(x) has two local maxima and two local minima points
(B) g(x) has exactly one local maxima and one local minima point
(C) x = 1 is a point of local maxima for g(x)
(D) There is a point of local maxima for g(x) in the interval (−1, 0)

Maxima-Minima 179
7. Let F(x) = (f(x))2+ (f (x))2, F(0) = 6 where f(x) is a thrice differentiable function such that

|f(x)|  1  x  [−1, 1], then choose the correct statement(s)

(A) there is at least one point in each of the intervals (−1, 0) and (0, 1) where |f (x)|  2

(B) there is at least one point in each of the intervals (−1, 0) and (0, 1) where F(x)  5

(C) there is no point of local maxima of F(x) in (−1, 1)

(D) for some c  (−1, 1), F(c) 6, F(c) = 0 and F(c)  0

8. Let f(x)= (x 1)m (x 2)n, x  R, then each critical point of f (x) is either local maximum or

local minimum where

(A) m=2, n=3 (B) m=2, n=4 (C) m=3, n=4 (D) m=4, n=2

9. If f(x) = (logx)n x, then -

(A) f(x) = 0 has exactly two solutions for n = 5

(B) f(x) = 0 has exactly one solution for n = 5

(C) f(x) = 0 has no solutions if n (0, 1)

(D) f(x) = 0 has exactly two solutions if n (0, 1)

10. Let f '(x) = (x2 x + 2) (x2 x 2)(x2 x 6) (x2 x 12), x R, then

(A) the equation of normal to f(x) at x = 2 is x = 2.

(B) the sum of values of x of at which f(x) has local maximum is 1.

(C) the minimum number of stationary points of f '(x) is 5.

 
 f(x)  
(D) the value of lim  9  equals .
x → 2x
  2
 
 9 

11. If L im f(x) = Lim f(x) ('a' is a finite quantity), where [·] denotes greatest integer function and f(x)
x →a x →a

is a non-constant continuous function, then

(A) L im f(x) is an integer. (B) L im f(x) need not be an integer.


x →a x →a

(C) f(x) has a local minimum at x = a (D) f(x) has a local maximum at x = a.

180 Maxima-Minima
12. Let f(x) be a cubic polynomial such that it has point of inflection at x = 2 and local minima at
x = 4, then:
(A) f(x) has local minima at x = 0 (B) f(x) has local maxima at x = 0

(C) L im f(x) → (D) lim f(x) → 


x → x →

13. Let f(x) = 2x3 15x2 + 36x 48, then-

(A) f(x) is increasing function in ( ,2) (3,)


5 
(B) graph of y = f(x) is concave upwards in  ,  
2 

5
(C) Point of inflection of f(x) at x =
2
(D) Range of f(x) for x  [2,4] is [ 21, 16]

2
14. If g(x) = 7x2 . e− x x R, then g(x) has

(A) local maxima at x = 0

(B) local minima at x = 0


(C) local maxima at x = 1

(D) two local maxima and one local minima

Maxima-Minima 181
Subjective Exercise - I

 3−x 0  x 1
1. Let f(x) =  2 . Find the set of values of b such that f(x) has a local minima at
 x + nb x 1
x = 1.

2. Find the points of local maxima/minima of following functions


(i) f(x) = 2x3 21x2 + 36x 20
(ii) f(x) = (x 1)3 (x + 1)2
(iii) f(x) = x nx

3. Find points of local maxima / minima of


(i) f(x) = (2x 1)(2x 2)2
(ii) f(x) = x2e x

(iii) f(x) = 3cos4x + 10cos3x + 6cos2x 3, x  [0, ]


(iv) f(x) = 2x + 3x2/3
x2 – 2
(v) f(x) =
x2 – 1

4. Draw graph of f(x) = x|x 2| and, hence find points of local maxima/minima.

5. Let f(x) = x2 ; x  ( 1, 2). Then show that f(x) has exactly one point of local minima but global
maximum is not defined.

6. Suppose f(x) is a function satisfying the following conditions:


(i) f(0) = 2, f(1) = 1
5
(ii) f has a minimum value at x = and
2
2ax 2ax − 1 2ax + b + 1
(iii) for all x, f '(x) = b b+1 −1
2(ax+ b) 2ax + 2b + 1 2ax + b

Where a, b are some constants. Determine the constants a, b & the function f(x).

7. Let f(x) be a cubic polynomial which has local maximum at x = 1 and f '(x) has a local minimum
at x = 1. If f( 1) = 10 and f(3) = 22, then find the distance between its two horizontal tangents.

8. If the sum of the lengths of the hypotenuse and another side of a right-angled triangle is given,

show that the area of the triangle is a maximum when the angle between these sides is .
3

182 Maxima-Minima
9. Find the volume of the largest cylinder that can be inscribed in a sphere of radius ' r ' cm.

10. Show that the semi vertical angle of a right circular cone of maximum volume, of a given slant

height is tan−1 2 .

11. A running track of 440 m. is to be laid out enclosing a football field, the shape of which is a
rectangle with semi-circle at each end. If the area of the rectangular portion is to be maximum,
find the length of its sides.

12. Find the area of the largest rectangle with lower base on the x-axis and upper vertices on the
curve y = 12 − x².

13. Find the dimensions of the rectangle of perimeter 36 cm which will sweep out a volume as
large as possible when revolved around one of its side.

14. The graph of the derivative f ' of a continuous function f is shown with f (0) = 0. If
(i) f is monotonic increasing in the interval [a, b)  (c, d)  (e, f] and decreasing in
(p, q)  (r, s).

(ii) f has a local minima at x = x1 and x = x2.


(iii) f is concave up in (l, m)  (n, t]

(iv) f has inflection point at x = k


(v) number of critical points of y = f (x) is 'w'.

Find the value of (a + b + c + d + e) + (p + q + r + s) + ( + m + n) + (x1 + x2) + (k + w).

Maxima-Minima 183
Subjective Exercise - II

 p(x) 
1. Let P(x) be a polynomial of degree 5 having extremum at x = 1, 1 and lim  3 − 2  = 4.
x →0
 x 

If M and m are the maximum and minimum value of the function y = P'(x) on the set

m
A = {x|x2 + 6  5x} then find .
M

2. The length of three sides of a trapezium are equal, each being 10 cms. Find the maximum area

of such a trapezium.

6
3. Of all the lines tangent to the graph of the curve y = , find the equations of the tangent
x +3
2

lines of minimum and maximum slope.

4. A closed rectangular box with a square base is to be made to contain 1000 cubic feet. The cost

of the material per square foot for the bottom is 15 paise, for the top 25 paise and for the sides

20 paise. The labour charges for making the box are Rs. 3/-. Find the dimensions of the box

when the cost is minimum.

5. A given quantity of metal is to be casted into a half cylinder i.e. with a rectangular base and

semicircular ends. Show that in order that total surface area may be minimum, the ratio of the

height of the cylinder to the diameter of the semi circular ends is /(+ 2).

6. The value of 'a' for which f (x) = x3 + 3 (a 7)x2 + 3 (a2 9)x 1 have a positive point of maximum

lies in the interval (a1, a2)  (a3, a4). Find the value of a2 + 11a3 + 70a4.

7. The circle x2 + y2 = 1 cuts the x-axis at P & Q. Another circle with centre at Q and variable

radius intersects the first circle at R above the x-axis & the line segment PQ at S. Find the

maximum area of the triangle QSR.

184 Maxima-Minima
Match the List Type Question

8. List I List II

(P) A rectangle is inscribed in an equilateral triangle of side 4cm. (1) 65

Square of maximum area of such a rectangle is

(Q) The volume of a rectangular closed box is 72 and the base (2) 45

sides are in the ratio 1 : 2. The least total surface area is

(R) If x and y are two positive numbers such that x + y = 60 and x3y is (3) 12

maximum then value of x is

(S) The sides of a rectangle of greatest perimeter which is inscribed (4) 108

in a semicircle of radius 5 are a and b. Then a3 + b3 =

(A) (P)→(3); (Q)→(4); (R)→(2); (S)→(1)

(B) (P)→(1); (Q)→(3); (R)→(2); (S)→(4)

(C) (P)→(3); (Q)→(2); (R)→(1); (S)→(4)

(D) (P)→(1); (Q)→(4); (R)→(3); (S)→(1)

Maxima-Minima 185
JEE(Main)-(Previous Year Questions)

1. If x = 1 and x = 2 are extreme points of f(x) = log |x| + x2 + x then: [JEE(Main)-2014]
1 1 1 1
(1)  = 6 , = (2)  = 6 ,= (3)  = 2 ,  = (4)  = 2 ,  =
2 2 2 2

2. Let f (x) be a polynomial of degree four having extreme values at x = 1 and x = 2.


 f(x) 
If lim  1 + 2  = 3, then f (2) is equal to: [JEE(Main)-2015]
x →0
 x 
(1) 0 (2) 4 (3) 8 (4) 4

3. A wire of length 2 units is cut into two parts which are bent respectively to form a square of
side = x units and a circle of radius = r units. If the sum of the areas of the square and the
circle so formed is minimum, then: [JEE(Main)-2016]
(1) 2x = r (2) 2x = ( + 4)r (3) (4 )x = r (4) x = 2r

4. Twenty meters of wire is available for fencing off a flower-bed in the form of a circular sector.
Then the maximum area (in sq. m) of the flower-bed is: [JEE(Main)-2017]
(1) 30 (2) 12.5 (3) 10 (4) 25

1 1 f(x)
5. Let f(x) = x2 + and g(x) = x , x R { 1, 0, 1}. If h (x) = , then the local minimum
x2 x g(x)
value of h(x) is: [JEE(Main)-2018]
(1) 2 2 (2) 3 (3) 3 (4) 2 2

6. The maximum volume (in cu. m) of the right circular cone having slant height 3m is:
[JEE(Main)-2019]
4
(1) 3 3  (2) 6  (3) 2 3  (4) 
3

3 
7. The shortest distance between the point  ,0  and the curve y = x, (x > 0) is:
2 
[JEE(Main)-2019]
3 5 3 5
(1) (2) (3) (4)
2 4 2 2

8. If S1 and S2 are respectively the sets of local minimum and local maximum points of the function
f(x) = 9x4 + 12x3 36x2 + 25, x  R, then: [JEE(Main)-2019]
(1) S1 = { 1}; S2 = {0, 2} (2) S1 = { 2} ; S2 = {0, 1}
(3) S1 = { 2, 1} ; S2 = {0} (4) S1 = { 2, 0} ; S2 = {1}

186 Maxima-Minima
9. A water tank has the shape of an inverted right circular cone, whose semi-vertical angle is
 1
tan   . Water is poured into it at a constant rate of 5 cubic meter per minute. Then the rate
2
(in m/min.), at which the level of water is rising at the instant when the depth of water in the
tank is 10 m; is [JEE(Main)-2019]
1 1
(1) (2)
10 15
1 2
(3) (4)
5 

10. The maximum value of the function f(x) = 3x3 18x2 + 27x 40 on the set
S = {x  R: x + 30  11x} is:
2
[JEE(Main)-2019]
(1) 222 (2) 122
(3) 122 (4) 222

11. The maximum area (in sq. units) of a rectangle having its base on the x axis and its other two
vertices on the parabola, y = 12 x2 such that the rectangle lies inside the parabola is:
[JEE(Main)-2019]
(1) 32 (2) 20 2
(3) 36 (4) 18 3

12. A helicopter is flying along the curve given by y x3/2 = 7, (x  0). A soldier positioned at the
1 
point  , 7  wants to shoot down the helicopter when it is nearest to him. Then this nearest
2 
distance is: [JEE(Main)-2019]

5 1 7
(1) (2)
6 3 3

1 1 7
(3) (4)
2 6 3

13. Let f(x) be a polynomial of degree 3 such that f( 1) = 10, f(1) = 6, f(x) has a critical point at
x = 1 and f'(x) has a critical point at x = 1. Then f(x) has local minima at x = ________
[JEE(Main)-2020]

14. A spherical iron ball of 10 cm radius is coated with a layer of ice of uniform thickness that melts
at a rate of 50cm3/min. When the thickness of ice is 5 cm, then the rate (in cm/min.) at which
of the thickness of ice decreases, is: [JEE(Main)-2020]
1 5 1 1
(1) (2) (3) (4)
36 6 54 18

Maxima-Minima 187
15. Suppose f(x) is a polynomial of degree four, having critical points at 1, 0, 1. If
T = {x  R|f(x) =f(0)}, then the sum of squares of all the elements of T is: [JEE(Main)-2020]
(1) 2 (2) 6 (3) 8 (4) 4

  
16. The set of all real values of  for which the function f(x) = (1 cos2x) . ( + sinx), x   − ,  ,
 2 2
has exactly one maxima and exactly one minima, is: [JEE(Main)-2020]
 1 1  1 1  3 3  3 3
(1)  − ,  − {0} (2)  − ,  (3)  − ,  (4)  − ,  − {0}
 2 2  2 2  2 2  2 2

1 4
17. The maximum slope of the curve y = x − 5x3 + 18x2 − 19x occurs at the point :
2
[JEE(Main)-2021]
21
(1) (2, 9) (2) (2, 2) (3) (3, ) (4) (0, 0)
2

18. The curve y(x) = ax2 + bx2 + cx + 5 touches the x-


the local maximum value of y(x) is : [JEE(Main)-2021]
27 29 37 9
(1) (2) (3) (4)
4 4 4 2

19. The sum of the absolute minimum and the absolute maximum values of the function
f(x) = |3x x2 + 2| x in the interval [ 1, 2] is : [JEE(Main)-2022]
17 + 3 17 + 5 9 − 17
(1) (2) (3) 5 (4)
2 2 2

20. Let x = 2 be a local minima of the function f(x) = 2x4 − 18x2 + 8x + 12, x  (−4, 4) . If M is local
maximum value of the function f in (-4, 4), then M = [JEE (Main)-2023]
33 31 33 31
(1) 12 6 − (2) 12 6 − (3) 18 6 − (4) 18 6 −
2 2 2 2

21. A wire of length 20 m is to be cut into two pieces. A piece of length 1 is bent to make a square

of area A1 and the other piece of length 2 is made into a circle of area A2. If 2A1 + 3A2 is minimum
then ( 1) : 2 is equal to: [JEE (Main)-2023]

(1) 6 : 1 (2) 3 : 1 (3) 1 : 6 (4) 4 : 1

22. The sum of the absolute maximum and minimum values of the function
f(x) = x2 − 5x + 6 − 3x + 2 in the interval [-1, 3] is equal to [JEE (Main)-2023]

(1) 10 (2) 12 (3) 13 (4) 24

188 Maxima-Minima
23. A square piece of tin of side 30 cm is to be made into a box without top by cutting a square
from each corner and folding up the flaps to form a box. If the volume of the box is maximum,
then its surface area (in cm2) is equal to [JEE (Main)-2023]
(1) 800 (2) 675
(3) 1025 (4) 900
sin2 x
 3e    k
24. If the total maximum value of the function f(x) =   , x   0,  , is , then
 2 sin x   2  e
 
8
k  k8 8
  + 5 + k is equal to: [JEE (Main)-2023]
e
  e
(1) e3 + e6 + e11 (2) e5 + e6 + e11
(3) e3 + e6 + e10 (4) e3 + e5 + e11

Maxima-Minima 189
JEE-Advanced (Previous Year Questions)

1. A rectangular sheet of fixed perimeter with sides having their lengths in the ratio of 8: 15 is
converted into an open rectangular box by folding after removing squares of equal area from
all four corners. If the total area of removed squares is 100, the resulting box has maximum
volume. Then the lengths of the sides of the rectangular sheet are: [JEE(Advanced)-2013]
(A) 24 (B) 32 (C) 45 (D) 60

2. The function f(x) = 2|x| + |x + 2| ||x + 2| 2|x|| has a local minimum or a local maximum
at x = [JEE(Advanced)-2013]
−2 2
(A) 2 (B) (C) 2 (D)
3 3

3. A cylindrical container is to be made from certain solid material with the following constraints.
It has a fixed inner volume of V mm3, has a 2 mm thick solid wall and is open at the top. The
bottom of the container is a solid circular disc of thickness 2 mm and is of radius equal to the
outer radius of the container. If the volume of the material used to make the container is
V
minimum when the inner radius of the container is 10mm, then the value of is:
250
[JEE(Advanced)-2015]
cos(2 x) cos(2 x) sin(2 x)
4. If f (x) = − cos x cos x − sin x , then [JEE(Advanced)-2017]
sin x sin x cos x

(A) f '(x) = 0 at exactly three points in ( , )


(B) f '(x) = 0 at more than three points in ( , )
(C) f(x) attains its maximum at x = 0
(D) f(x) attains its minimum at x = 0

5. Consider all rectangles lying in the region


  
(x, y)  R  R : 0  x  x and 0  y  2 sin(2x) 
 2 
and having one side on the x-axis. The area of the rectangle which has the maximum perimeter
among all such rectangles, is: [JEE(Advanced)-2020]

3   3
(A) (B)  (C) (D)
2 2 3 2

6. Let the function: (0, ) → be defined by f() = (sin + cos)2 + (sin cos)4 Suppose the
function f has a local maximum at  precisely when  {1,...., r}, where 0 < 1 < ··· < r < 1.
Then the value of 1 + ··· + r is_____ [JEE(Advanced)-2020]

190 Maxima-Minima
7. For a polynomial g(x) with real coefficient, let mg denote the number of distinct real roots of
g(x). Suppose S is the set of polynomials with real coefficient defined by
S = {(x2 1)2 (a0 + a1x + a2x2 + a3x3): a0, a1, a2, a3  R}.
For a polynomial f, let f ' and f " denote its first and second order derivatives, respectively. Then
the minimum possible value of (mf ' + mf "), where f  S, is ______ [JEE(Advanced)-2020]

Comprehension Type Questions


Question Stem for Question Nos. 8 and 9
Let f1 : (0, ) → and f2 : (0, ) → be defined by [JEE(Advanced)-2021]
x 21

 and f2 (x) = 98(x − 1)50 − 600(x − 1)49 + 2450, x 0


j
1
= 
0 j= 1

where, for any positive integer n and real numbers a 1 ,a2 ,  ,an ,  ni=1 ai denotes the product of

a1 ,a2 , ,an . Let mi and ni , respectively, denote the number of points of local minima and the
number of points of local maxima of function fi ,i = 1,2 , in the interval (0, ) .

8. The value of 2m1 + 3n1 + m1n1 is

9. The value of 6m2 + 4n2 + 8m2n2 is

 
10. Let |M| denote the determinant of a square matrix M. Let g : 0,  → be the function
 2
 
defined by g() = f() − 1 + f  −   − 1 where
 2 
   
sin  cos   +  tan   − 
 4  4
1 sin  1
1    4
f() = − sin  1 sin  + sin   +  − cos log e   . [JEE(Advanced)-2023]
2  4 2 
−1 − sin  1
     
cot   +  loge   tan 
 4  4
 
Let p(x) be a quadratic polynomial whose roots are the maximum and minimum values of the
function g() , and p(2) = 2 − 2 . Then, which of the following is/are TRUE?
3+ 2   1+ 3 2   5 2 − 1 5− 2 
(A) p  0 (B) p  0 (C) p  0 (D) p  0
 4   4   4   4 
       

Maxima-Minima 191
ANSWER KEY

1. Limit
Objective Exercise - I
Que. 1 2 3 4 5 6 7 8 9 10 11 12 13 14 15 16 17 18 19 20
Ans. A C D C D B B A C D D C B D A A A D A B

Objective Exercise - II
Que. 1 2 3 4 5 6 7 8 9 10 11 12 13 14 15 16 17 18 19 20
Ans. C A D D D A C BCD ABD ACD AC BC AB AD AC AD BC ABC AD BCD
Que. 21 22 23 24 25
Ans. ABCD BCD A C D

Subjective Exercise - I

1. 2 2. 5050

1
3. 2 4.
16 2

1
5. 2 6. (i) a = 1, b = 1 (ii) a = 1, b =
2

7. 1 8. 8 2 (n3)2

9. 16 10. e 8

11. c = n2

Subjective Exercise - II

1. 1 2. 3

3. 1 4. 3

5. 1 6. 1

7. 4 8. (a) 2, (b) 0, (c) 0

192 Answer Key


JEE-Main (Previous Year Questions)
Que. 1 2 3 4 5 6 7 8 9 10 11 12 13 14 15 16 17 18 19 20
Ans. 4 1 1 1 2 3 3 4 1 2 36 4 40 3 4 1 3 3 4 4
Que. 21 22 23 24 25 26 27 28 29 30
Ans. 1 11 4 3 1 1 4 3 3 4

JEE-Advanced (Previous Year Questions)

1. 0 2. 7

3. (AD) 4. (AD)

5. 1 6. 8

7. (AB) 8. 0.50

9. 5

Answer Key 193


2. Continuity
Objective Exercise - I
Que. 1 2 3 4 5 6 7 8 9 10 11 12 13 14 15
Ans. A C A D A D B C C A D B C B D

Objective Exercise - II
Que. 1 2 3 4 5 6 7 8 9 10
Ans. C C B D B AB ACD ABCD AC ABCD

Subjective Exercise - I

1. 1 2. a=0;b= 1

3. (a) 2, 2, 3; (b) K = 5;

(c) even

4. g(x) = 2 + x for 0  x  1, 2 x for 1 < x  2, 4 x for 2 < x  3, g is discontinuous at x = 1 & x=2

5. a = 1/2, b = 4 6. a = 3/2, b  0, c = 1/2

7. A= 4, B = 5, f(0) = 1

8. locus (a, b) → x, y is y = x 3 excluding the points where y = 3 intersects it.

9. c = 1, a, b  R

Subjective Exercise - II

1. a = 0, b = 1 2. P not possible.

3. (a) 4 3 2 + a, (b) a=3

4. discontinuous at all integral values in [ 2, 2]

5. e2 + e 2
6. (P)→(2); (Q)→(3); (R)→(4); (S)→(1)

194 Answer Key


JEE-Main (Previous Year Questions)
Que. 1 2 3 4 5 6 7 8 9 10 11 12 13 14 15 16 17 18 19 20
Ans. 1 4 3 1 4 2 3 4 5 4 8 4 2 3 1 6 4 1 4 1
Que. 21 22
Ans. 4 2.00

JEE-Advanced (Previous Year Questions)

1. Discontinuous at x = 1 ; f(1+) = 1 and f(1 ) = 1

2. (BD) 3. (AD)

4. (ACD)

Answer Key 195


3. Differentiability
Objective Exercise - I
Que. 1 2 3 4 5 6 7 8 9 10 11 12 13 14 15 16 17
Ans. B C D A B D C A D C A D D D B D D

Objective Exercise - II
Que. 1 2 3 4 5 6 7 8 9 10 11 12 13 14 15
Ans. D B B A A D ABC ABC BC ABCD ABCD BCD BD ABD BD
Que. 16 17
Ans. (P)→(1,3,4), (Q)→(1,3,4), (R)→(2,3,4), (S)→(3,4) (P)→(1,2,3), (Q)→(1,3,4), (R)→(1,3,4), (S)→(1,3,4)

Subjective Exercise - I

1. f(x) is continuous but not derivable at x = 0

2. continuous x R, not differentiable at x = 0, 1 and 2

3. 0<n1

4. continuous but not differentiable at x = 0 ; differentiable and continuous at x = /2

5. continuous but not differentiable at x = 0 7. f is cont. but not differentiable at x = 0

8. a = 1/2, b = 3/2 9. f(x) = 2x + c

10. f' (1+) = 3, f '(1 ) = 1 11. not derivable at x = 0 & x = 1

Subjective Exercise - II

1. 74 2. 120

3. 3 4. 4

5. 7 6. 4

7. 7 8. 3

9. 11 10. 10

11. (C) 12. (C)

13. (C)

196 Answer Key


JEE-Main (Previous Year Questions)
1 2 3 4 5 6 7 8 9 10 11 12 13 14 15 16 17 18 19 20
2 3 4 4 2 3 1 1 10 3 2 1 2 2 4 5 3 309 4 4
21 22
3 25

JEE-Advanced (Previous Year Questions)

1. (D) 2. 3

3. (AB) 4. (BC)

5. 2 6. (D)

7. (AC) 8. (ABD)

9. (ACD)

Answer Key 197


4. Methods of Differentiation
Objective Exercise - I
Que. 1 2 3 4 5 6 7 8 9 10 11 12 13 14 15 16 17 18 19 20
Ans. A B B D B C C D A A C D A A B A C B B B
Que. 21 22 23 24 25 26 27 28 29 30 31 32 33
Ans. C C D B A C D B A A D C A

Objective Exercise - II
Que. 1 2 3 4 5 6 7 8 9 10 11 12 13 14 15 16 17
Ans. C B D A A C D C B BD CD ABC ACD A ABC BD (A) S; (B) P ; (C)R.

Subjective Exercise - I

1. 16 4. 8

 21 3
 −  + n  x if x  0
3  6 2 
6. f(x)=  1/x 8. 100
  1+ x 
   if x  0
 2+ x

 1 1 2x
9. (a) − ,  , ( , ); (b) f(x) = ;
 2 2 1 − 4x2

16 3
(c)
9

1
10. (a) 1/6 ; (b) ;
14

(c) 1

1  
 , x   0, 
1 1 dy  2  2
11. − 13. =
1 + (x+ n)2 1 + x2 dx  1  
− , x   , 
 2 2 

14. 3

15. (a) (i) y'(0) = 1; (ii) y"(0) = 2;

(b) a =1, 2

198 Answer Key


−3 −9
16. 17.
2 4

18. 2(1 + 2x).cos 2(x + x2) 19. 100

5 1
20. 21.
6 3

1
22. 23. 1000
2

24. 25 25. 3 (k =1, 1 or 0)

Subjective Exercise - II

1. f is continuous but not derivable at x = 0 2. 10

3. 6 4. 12

 x cos x − sinx
if x  0 ; f " (0) = 1
5. f ' (x)=  x2
0 if x = 0 3

1 − 2x
6. Zero 8.
2 1 − x2

10. 2 11. 1

JEE-Main (Previous Year Questions)


Que. 1 2 3 4 5 6 7 8 9 10 11 12 13 14 15 16 17 18 19 20
Ans. 2 4 3 2 2 4 2 1 2 1 2 2 4 1 3 1 481 2 1 248
Que. 21 22 23 24 25 26 27 28 29 30
Ans. 4 3 16 4 16 4 3 2 3 1

JEE-Advanced (Previous Year Questions)

1. (C) 2. (D)

3. (BC)

Answer Key 199


5. Rate Measure, Tangent and Normal
Objective Exercise - I
Que. 1 2 3 4 5 6 7 8 9 10 11 12 13 14 15 16 17 18 19 20
Ans. C D C B A B C C C C D D B B C D D C A D

Objective Exercise - II
Que. 1 2 3 4 5 6 7 8 9 10 11 12 13 14
Ans. C C C AC AC ABC CD AB AC AC ABD AD AB A

Subjective Exercise - I

1
1. m/min
9

2. (i) 6 km/h ; (ii) 2 km/hr

3. (4, 11) & ( 4, 31/3) 4. 3/8 cm/min

80
5. (i) (a) 6.05, (b) ; (ii) 9.72cm3
27

1
6. cm/s 7. x+y 1=0
48

8. x = 1 when t = 1, m→  ; 5x 4y = 1 if t 1, t = 1/3

−5 82.73
9. a = 1, b = 10. –
2 3

1 3
11. a= − ;b= − ;c=3 12. 3, 12
2 4

13. (0, 1)

200 Answer Key


Subjective Exercise - II

1. 8 2. 4

3. 37 4. 73

5. 2 6. 24

7. 17 8. 2

9. 9 10. 2

JEE-Main (Previous Year Questions)


Que. 1 2 3 4 5 6 7 8 9 10 11 12 13 14 15 16 17 18 19 20
Ans. 2 3 3 3 1 3 2 2 1 4 2 1 4 4 0.50 3 1 1 2 195
Que. 21 22 23 24 25 26
Ans. 4 4 3 292 5 11

JEE-Advanced (Previous Year Questions)

1. 2x+y 2 2 = 0 or 2x y 2 2=0 2. (D)

3. (D) 4. (D)

5. (A) 6. 8

Answer Key 201


6. Monotonicity
Objective Exercise - I
Que. 1 2 3 4 5 6 7 8 9 10 11 12 13 14 15 16 17 18 19 20
Ans. A B C A A A B C C C A C B C C C A D C A
Que. 21
Ans. A

Objective Exercise - II
Que. 1 2 3 4 5 6 7 8 9 10 11 12 13 14 15
Ans. C A D B D ABC BCD BC BC BC AC BD ABCD ABCD A

Subjective Exercise - I

1. (A) I in (2, ) and D in ( , 2)

(B) I in (1, ) and D in ( , 0)  (0, 1)

(C) I in (0, 2) and D in ( , 0)  (2,)

1 1 1 1
(D) I for x > or < x < 0 and D for x < or 0 < x <
2 2 2 2

2. (A) I in [0, 3/4)  (7/4, 2] and D in (3/4, 7 /4)

(B) I in [0, /6)  (/2, 5/6) (3/2 , 2] and D in (/6, /2)  (5/6, 3 /2)]

(C) I in [0, /2)  (3/2, 2] and D in (/2, 3/2)

3. (A) (/6) + (1/2)ln 3, (/3) (1/2)ln 3

(B) ( 9/4

(C) 2 and 10

mb + na
4. ( 1, 0)  (0,) 5. c = which lies between a and b
m+n

8. a = 3, b = 4 and m = 1 9. (0, 1/3)  (1, 5)

202 Answer Key


Subjective Exercise - II

1. ( 2, 0)  (2, ) 2. continuous but not diff. at x = 1

3. a 0 5. (6, )

6. [1, ) 7. [ 7, 1)  [2, 3]

11. (C) 12. (B)

13. (A)

JEE-Main (Previous Year Questions)


Que. 1 2 3 4 5 6 7 8 9 10 11 12 13 14 15
Ans. 4 1 4 2 4 2 1 3 2 2 2 4 3 2 3 (Bonus)

JEE-Advanced (Previous Year Questions)

1. (BC) 2. (A)

3. (D) 4. (D)

5. (D) 6. (ABD)

7. (AB) 8. (ABC)

9. (ABC)

Answer Key 203


7. Maxima-Minima
Objective Exercise - I
Que. 1 2 3 4 5 6 7 8 9 10 11 12 13 14 15 16 17 18 19 20
Ans. A C D B C A D B D B D B C D C C C B A A

Objective Exercise - II
Que. 1 2 3 4 5 6 7 8 9 10 11 12 13 14
Ans. C B A A AC BD ABD BD B ABCD AC BC ABCD BCD

Subjective Exercise - I

1. b  (0, e]

1
2. (i) local max at x = 1, local min at x = 6 (ii) local max. at x = , local min. at x = 1
5

1
(iii) local mini at x = , No local maxima
e

4
3. (i) local maxima at x = log2 and local minima at x = 1
3

2 
(ii) local min at 0, local max at 2 (iii) local max at x = 0, , local min at x = , 
3 2

(iv) local maxima at 1 and local minima at 0

(v) local minima at x = ± 2 , 0

1 5 1
4. local max at x = 1, local min at x = 2. 6. a= ;b= ; f(x) = (x2 5x + 8)
4 4 4

4  r3
7. 32 9.
3 3

220
11. 110 m, m 12. 32 sq. units

13. 12cm, 6 cm 14. 74

204 Answer Key


Subjective Exercise - II

1. 6 2. 75 3 sq. units

3. 3x + 4y 9 = 0 ; 3x 4y + 9 = 0. 4. side 10', height 10'

4
6. 320 7.
3 3

8. (A)

JEE-Main (Previous Year Questions)


Que. 1 2 3 4 5 6 7 8 9 10 11 12 13 14 15 16 17 18 19 20
Ans. 3 1 4 4 1 3 4 3 3 2 1 4 3.00 4 4 4 2 1 1 1
Que. 21 22 23 24
Ans. 1 1 1 1

JEE-Advanced (Previous Year Questions)

1. (AC) 2. (AB)

3. 4 4. (BC)

5. (C) 6. 0.50

7. 5.00 8. 57.00

9. 6.00 10. (AC)

Answer Key 205


Quick Revision Notes

--------------------------------------------------------------------------------------------------

--------------------------------------------------------------------------------------------------

--------------------------------------------------------------------------------------------------

--------------------------------------------------------------------------------------------------

--------------------------------------------------------------------------------------------------

--------------------------------------------------------------------------------------------------

--------------------------------------------------------------------------------------------------

--------------------------------------------------------------------------------------------------

--------------------------------------------------------------------------------------------------

--------------------------------------------------------------------------------------------------

--------------------------------------------------------------------------------------------------

--------------------------------------------------------------------------------------------------

--------------------------------------------------------------------------------------------------

--------------------------------------------------------------------------------------------------

--------------------------------------------------------------------------------------------------

--------------------------------------------------------------------------------------------------

--------------------------------------------------------------------------------------------------

--------------------------------------------------------------------------------------------------

--------------------------------------------------------------------------------------------------

--------------------------------------------------------------------------------------------------

--------------------------------------------------------------------------------------------------

--------------------------------------------------------------------------------------------------

You might also like